Tuyển chọn các bài toán hình học hay và khó bồi dưỡng HSG và luyện thi vào lớp 10 chuyên

140 246 0
Tuyển chọn các bài toán hình học hay và khó bồi dưỡng HSG và luyện thi vào lớp 10 chuyên

Đang tải... (xem toàn văn)

Tài liệu hạn chế xem trước, để xem đầy đủ mời bạn chọn Tải xuống

Thông tin tài liệu

Đến đây, chỉ cần chứng minh được thêm OI vuông góc với MN thì bài toán hoàn tất do có các điểm O, I, E cùng nằm trên một đường thẳng vuông góc với MN Do O, I lần lượt là các tâm đường t[r]

(1)



CÁC BÀI TỐN HÌNH HỌC HAY TRUNG HỌC CƠ SỞ

(2)

TUYỂN CHỌN CÁC BÀI HÌNH HAY VÀ KHĨ

BỒI DƯỠNG HỌC SINH GIỎI TOÁN 9VÀ LUYỆN THI VÀO LỚP 10 CHUYÊN

TOÁN

Hình học phẳng nội dung quan trọng chương trình mơn tốn trường THCS THPT chuyên toán Trong năm gần đầy tốn

về hình học phẳng xuất đề thi vào lớp 10 THPT, lớp 10 khiếu tốn kì thi học sinh giỏi cấp với độ khó ngày cao Với mong muốn tuyển chọn hình hay khó nhằm mục đích làm tài liệu học tập cho học

sinh tài liệu giảng dạy cho giáo viên, soạn tài liệu ”Tuyển chọn hình hay khó bồi dưỡng học sinh giỏi toán luyện thi vào lớp 10

chuyên toán” Nội dung tài liệu giới thiệu tốn hình học phẳng mà thân tác giả thấy hay khó, với lời giải trình bày cơng phu xác Với cách viết đặt bạn đọc vào vị trí người giải, lối suy nghĩ hình thành lời giải tốn cách tự nhiên đảm bảo tính khoa học, hy vọng tài liệu thực có ích cho bạn đọc chinh phục tốn hình học phẳng Mặc dù chúng tơi thực cố gắng dành nhiều tâm huyết để hoàn thiện sách với hiệu cao nhất, song sai sót điều khó tránh khỏi Chúng tơi mong đónggóp ý kiến bạn đọc để chúng tơi hồn thiện tài liệu tốt

Bài Đường tròn ngoại tiếp đường tròn nội tiếp tam giác ABC có bán kính R r Biết BAC ACB ABC BAC− = − Tính diện tích tam giác ABC theo R

và r

Lời giải

Gọi O I tâm đường tròn ngoại tiếp tâm đường tròn nội tiếp tam giác ABC gọi D, E, F tiếp điểm đường trịn ( )I với AB, AC, BC Khi ta có

( )

( )

ABC

1 1

S AB.r AC.r BC.r r AB AC BC

2 2

1

r AD AE BD BF CE CF

= + + = + +

= + + + + + M

F E D

H I O

C B

(3)

Mà ta có AD AE, BD BF,CE CF= = = nên SABC 1r 2.AD BF CF( ) r AD BC( )

2  

=  + + = +

Ta có BAC ACB ABC BAC− = − nên 2BAC ABC ACB= + Do

3BAC 180= nên ta

được BAC 60= suy DAI=300

Trong tam giác vuông DAI có AD DI.cot DAI r.cot 30= = =r 3

Kẻ đường kính AM đường trịn ( )O , ta OAB OBA=

Từ suy BOM=OAB OBA+ hay BOM=2.OAB Tương tự ta có COM=2.OAC Suy BOM COM+ =2 OAB OAC( + ) nên BOC=2.BAC 2.60= =1200

Kẻ OH vng góc với BC ta HB HC BC

= = HOC 1BOC 600

2

= =

Trong tam giác OHC có HC OC.sin HOC R.sin 600 R

= = =

Từ suy BC=2.HC R 3= nên ta SABC =r r R 3( + )= 3.r R r( + )

Bài Cho tam giác ABC nhọn có AB AC nội tiếp đường trịn ( )O Gọi H trực tâm tam giác AH vng góc với BC F Gọi M trung điểm BC Trên đường tròn ( )O lấy điểm Q K cho HQA HKQ 90= = (Các điểm A, B, C, K,

Q theo thứ tự đường trịn) Chứng minh đường tròn ngoại tiếp tam giác KHQ tiếp xúc với đường tròn ngoại tiếp tam giác MFK

Lời giải

Lời giải Do HQA HKQ 90= = 0 nên

đường tròn ngoại tiếp tam giác AHQ QHK nhận AH QH đường kính Gọi AD đường kính đường

tròn ( )O E điểm đối xứng với H qua BC Khi dễ thấy tứ giác HBDC hình

hình hành nên ba điểm H, M, D thẳng hàng Mà ta lại có HQA 90= nên suy

ba điểm Q, H, D thẳng hàng Từ ta điểm Q, H, M, D thẳng hàng

D E

F

Q

H A

B C

O

K

M X

(4)

tiếp tam giác QHK cắt X Khi ta có XKH XHK KQH= = Mà ta lại có

KQH KQD KAD= = Từ ta có biến đổi góc

( )

( )

0 0

0

KXH 180 2KHX 180 2KQH 90 KQH

2 90 KAD 2ADK 2KEH

= − = − = −

= − = =

Lại có XH=XK nên X tâm đường tròn ngoại tiếp tam giác KHE Do E H đối xứng qua BC nên BC đường trung trực DH, từ suy X thuộc đường thẳng BC Do XH tiếp tuyến đường trịn đường kính QH nên ta có XH vng góc với QH H Tam giác XHM vuông H có đường cao HF nên

2

XK =XH =XF.XM.Từ suy XK tiếp tuyến chung hai đường tròn ngại tiếp tam giác KQH KFM với tiếp điểm K giao điểm hai đường trịn Do suy hai đường trịn ngại tiếp tam giác KQH KFM tiếp xúc K

Lời giải 2. Do HQA HKQ 90= = nên đường tròn ngoại tiếp tam giác AHQ

QHK nhận AH QH đường kính Gọi AD đường kính đường trịn

( )O E điểm đối xứng với H qua BC Khi dễ thấy tứ giác HBDC hình hình hành nên ba điểm H, M, D thẳng hàng Mà ta lại có HQA 90= nên suy ba điểm Q,

H, D thẳng hàng Từ ta điểm Q, H, M, D thẳng hàng Kẻ đường kính PQ đường trịn ( )O , HKQ 90= nên suy ba điểm P, H, K thẳng hàng

Đến ta có biến đổi góc PKE PQE OQE 900 1QOE 900 QBE

= = = − = − Ta dễ dàng

chứng minh DE song song với BC nên ta lại có

( ) ( )

1 1

QMC EMC sdQC sdBD sdQC sdCE sdQE QBE

2 2

= = + = + = =

Để ý MHE 90= 0−QMC 90= 0−QBE, từ ta suy PKE=MHE nên DQ

là tiếp tuyến tại H đường tròn ngoại tiếp tam giác KHE Giả sử đường thẳng vng góc với HQ H cắt BC X, đường thẳng HX qua tâm đường tròn ngoại tiếp tam giác KHE Mặt khác dễ thấy BC đường trung trực HE nên

XE=XH, từ suy X tâm đường tròn ngoại tiếp tam giác KHE,

(5)

trịn ngoại tiếp tam giác KHQ nên suy XK tiếp tuyến K với đường tròn ngoại tiếp tam giác KHQ Áp dụng hệ thức lượng cho tam giác MHX có đường cao HF ta có

2

XK =XH =XF.XM nên XK tiếp tuyến K với đường tròn ngoại tiếp tam giác KMF Vậy đường tròn ngoại tiếp tam giác KHQ tiếp xúc với đường tròn ngoại tiếp tam giác MFK

Bài Cho tam giác ABC có ba đường cao AD, BE, CF Gọi G, P hình chiếu D AB AC Gọi I, K hình chiếu E AB BC Gọi M, N hình chiếu F AC BC Chứng minh sáu điểm G, P, I, K, M,

N nằm đường tròn

Lời giải Ta xét trường hợp sau

Trường hợp Tam giác ABC vng Khơng tính tổng qt ta giả sử tam giác ABC vng A, ba điểm A, E, F trùng Ba điểm N, D, K trùng Các điểm I M trùng với A Khi ta có tứ giác AGDP hình chữ nhật nên điểm A, G, P, D nằm đường tròn Từ suy G, P, I, K, M, N

Trường hợp 2. Tam giác ABC có ba góc nhọn Khi dễ thấy tứ giác AGDP nội tiếp đường tròn

Suy ta GAD GPD= nên

GPC GBC 180+ =

nên tứ giác BCPG nội tiếp đường tròn Dễ thấy tứ giác BCEF EFIM nội tiếp đường tròn nên ta FEC FBC 180+ = Do FEC GPC= nên EF

song song với GP Mặt khác

MIF MEF 180+ = nên suy MIF MPG 180+ =

P M I

G

K N

F

E

D C

B

A

Do tứ giác GPMI nội tiếp đường tròn ( )O1 Tương tự ta chứng minh tứ giác MNPK nội tiếp đường trịn ( )O2 Hồn tồn tương tự ta tứ giác AIKC nội tiếpđường tròn Từ tứ giác BCPG nội tiếp đường tròn ta

MPG=GBC kết hợp với MIF MPG 180+ = suy MIF GBC 180+ = nên IM song

song với BC Tương tự ta LP song song với AB Từ ta

0

(6)

KPMI nội tiếp đường tròn ( )O3 Qua ba điểm không thẳng hàng xác định đường tròn nên ba đường tròn ( )O1 , ( )O2 ( )O3 trùng Vậy sáu điểm G, P, I, K, M, N nằm đường trịn

Trường hợp 3. Tam giác ABC tam giác tù Chứng minh hoàn toàn tương tự ta sáu điểm G, P, I, K, M, N nằm đường tròn

Vậy sáu điểm G, P, I, K, M, N nằm đường tròn

Bài 4. Cho tam giác ABC có

BAC=30 Đường phân giác phân giác ngồi

của góc ABC cắt cạnh AC B , B1 2 Đường phân giác phân giác

ngoài góc ACB cắt cạnh AB C ,C1 2 Đường tròn ngoại tiếp tam giác

1

BB B cắt đường tròn ngoại tiếp tam giác CC C1 2 điểm P tam giác ABC Gọi O trung điểm B B1 2 Chứng minh CP vng góc với BP

Lời giải

Do BB ; BB1 2 đường phân giác phân giác ABC nên

0

B BB =90 Khi tam giác BB B1 tam

giác vuông B Do O trung điểm

1

B B nên O tâm đường tròn ngoại tiếp

tam giác BB B1 2

P

O

C2

B2

C1

B1 C

B

A

Do ta có OBC OBB= 1−CBB1 =BB O B BA1 − 1 =BAC Từ suy hai tam giác

OBA OCB đồng dạng với nhaunên ta OB OA

OC= OB hay

2

OA.OC OB= Do ta OA.OC OP= hay OA OP

OP =OC nên hai tam giác OPC OAP đồng dạng với nhau, từ suy OPC PAC= Ta có biến đổi góc sau

( 1 ) ( 1) 1

PBC PBA PBB B BC ABB PBB 2PBB POB

PCA OPC PCA PAC

− = + − − = =

= − = −

Do suy PAC PBC PBA PCA+ = + Tương tự ta PAB PCB PBA PCA+ = +

(7)

PAC PBC PAB PCB PBA PCA PBA PCA+ + + = + + +

Từ suy ( ) ( )

180 − PBA PCA+ =2 PBA PCA+ hay

PBA PCA+ =60 Mà theo giả thiết BAC=300 nên suy PBC PCB 90+ = hay PB PC vng góc với

Bài 5.Cho tam giác nhọn ABC khơng cân nội tiếp đường trịn tâm O Lấy điểm P cạnh AB cho BOP=ABC lấy điểm Q cạnh AC cho COQ ACB=

Chứng minh đường thẳng đối xứng với BC qua PQ tiếp tuyến đường tròn ngoại tiếp tam giác AQP

Lời giải Giả sử đường thẳng AO cắt BC D Do O tâm đường tròn nội tiếp tam giác ABC

nên OA OB= Từ ta OAB OBA=

Mà theo giả thiết ta có OAB ABC= nên ta

suy hai tam giác ABD BOP đồng dạng với

O D

Q P

M E

C B

A

Suy ta có AB AD

BO = BP hay AB.BP AD.BO= BPO=ADB Tương tự ta có hai

tam giác ACD COQ đồng dạng với nhau.Từ ta AC.CQ AD.CO= CQO ADC= Do ta có biến đổi góc

( )

0 0

APO AQO 180+ = −BPO 180+ −CQO=360 − ADB ADC+ =180

Suy tứ giác APOQ nội tiếp đường tròn hay điểm O nằm đường tròn nội tiếp tam giác APQ Gọi E giao điểm đường tròn ngoại tiếp tam giác APQ với đường tròn ngoại tiếp tam giác ABC Từ ta có OE OA= nên suy OAE=OEA tứ giác AEQO nội tiếp đường tròn nên AEO AQO= Kết hợp với từ giác AQOP nội tiếp đường tròn ta suy AEO AQO BPO ADB= = = Từ ta có EAO=ADB nên

AE song song với BC Mà ta lại có OB OC= nên AB.BP AC.CQ=

(8)

Nguyễn Công Lợi Website: tailieumontoan.com

Do EPQEAQ;EAQACB nên EPQ ACB Từ suy hai tam giác EPQ ACB đồng dạng với Điều dân đến hai tam giác ABC EBC nên ta APE APQ EPQ AOQ EOQ AOB= − = − =

Mặt khác ta lại có OAC 1(1800 AOC) 900 ABC

= − = − EAC=ACB

Từ suy 0 ( ) ( )

AOB 180= −2OAE 180= −2 90 −ABC ACB+ =2 ABC ACB−

Do ta APE=2 ABC ACB( − )

Ta lại có ABE=ABC EBC ABC ACB− = − nên suy APE 2ABE= hay PBE PEB=

Ta có AQP=AEP 180= 0−(BAC ACB+ ) (− ABC ACB− )=2ACB ABC−

Nên ta PMB ACB MQP ACB AQP ABC ACB= − = − = −

Suy PMB=BEP=ABC ACB− nên tứ giác BPEM nội tiếp đường tròn

Kết hợp với PBE PEB= ta BMP=PME nên MP phân giác góc BME hay

nọi cách khác hai đường thẳng ME BC đối xứng với qua PQ

Lại có EQM 180= 0−EQP 180= 0−ABC PEM 180= 0−ABC Từ dẫn đến hai tam giác MQE MPE đồng dạng với nhau, ta ME2 =MP.MQ Suy ME

tiếp tuyến đường tròn ngoại tiếp tam giác APQ

Bài 6.Cho tam giác ABC nội tiếp đường tròn (O; R) Kẻ đường cao BE CF tam giác ABC Gọi M trung điểm BC Gọi I tâm đường tròn nội tiếp tam giác ABC D giao điểm AI với đường tròn ( )O Tìm giá trị k để ID kOA=

biết tam giác MEF

(9)

D F

E M

I O

C B

A

M I

D O

F E

C B

A

Ta có I tâm đường trịn nội tiếp nên DB DC DI= = Do M trung điểm BC nên

MB MC ME= = Do BAC 90= EF, điều trái với giả thiết Vậy ta

được

BAC90 Ta xét trường hợp sau • Trường hợp 1. Nếu

BAC 90 Khi ta có ( )

EMF 180= − BMF CME + Tam giác BEC BFC vuông nên tam giác BMF MCE tam giác cân

Do

BMF 180= −2FBM

CME 180= −2ECM

Suy ta 2EMF FBM ECM 90= ( + − 0) (=2 1800−BAC 90− 0)=1800 −2BAC

Mà ta lại có

EMF 60= nên ta 0

180 −2BAC 60 =

Do

BAC 60= nên suy

BAD 30= Từ ta có

BOD 60= Do ta

BI R OA= = hay DI=R nên k 1= • Trường hợp 2.Nếu

BAC 90 Khi ta có

( ) ( )

( ) ( )

( ) ( ) ( )

0 0

0 0

0 0

EMF 180 BME CMF 180 180 2EBC 180 2FCB

2 EBC FCB 90 90 ECB 90 FBC 90

180 ECB FBC 180 180 BAC BAC 90

= − + = − − + −

= + − = − + − −

= − + = − − = −

Mà ta có EMF 60= nên BAC 90− =300suy BAC 120= Suy BAD 60= nên ta

được BOD 120= hay MBD 60= Từ ta BD=R hay ID=R nên suy k=

(10)

Bài 7. Cho tam giác nhọn ABC nội tiếp đường tròn ( )O Các đường cao AD, BE, CF cắt H Đường thẳng EF cắt đường tròn ( )O M N(F nằm M E) Chứng minh AM tiếp tuyến đường tròn ngoại tiếp tam giác MDH

Lời giải

Trong nửa mặt phẳng bờ AB không chứa điểm C vẽ tia tiếp tuyến Ax với đường tròn

( )O Khi ta có ACB=BAx Mặt khác

BFC=BEC 90= nên tứ giác BCEF nội tiếp đường trịn Suy ta BFE=ACB Do ta BAx BFE= nên suy Ax

song song với EF

Mà ta có OA vng góc với Ax nên ta suy OA vng góc với EF Từ suy AM=AN nên ta

được ABM=AMF Xét hai tam giác ABM AMF có BAM góc chung ABM=AMF nên hai tam giác ABM ÀM đồng dạng với Do tađược AB AM

AM= AF hay

2

AM =AB.AF

O x

H

N

M F

E

D C

B

A

Xét hai tam giác AFH ADB có FAH góc chung AFH ADB= nên hai tam giác

AFH ADB đồng dạng với nhau.Từ ta AF AH

AD= AB hay AB.AF AH.AD=

Kết hợp kết ta AM2=AH.AD nên AM AH

AD =AM Suy hai tam giác

AMH ADM đồng dạng với nhau, ta AMH=ADM Vẽ tia tiếp tuyến Mt đường tròn ngoại tiếp tam giác ADH, tia Mt nằm nửa mặt phẳng bờ MH

không chứa điểm D Khi ta có HMt=HDM nên ta suy HMt=HMA, điều dẫn đến hai tia Mt MA trùng hay MA là tiếp tuyến đường tròn ngoại tiếp tam giác MDH

Bài Cho tam tứ giác ABCD nội tiếp đường tròn ( )O có E giao điểm hai đường chéo AC BD Chứng minh ABD 60= AE 3CE= tứ giác

ABCD ln có AB CD AD BC+ = + AB BC AD CD+ = +

(11)

Vẽ DH vuông góc với AB H, DK vng góc với BC K

Ta có AH AD.cosHAD AD.cos 600 AD

2

= = =

0 3AD

DH AD.sin HAD AD.sin 60

2

= = =

Tam giác HBD vuông H nên theo định lí

Pitago ta có 2

BD =BH +DH nên suy

E K

H D

C

B A

2

2 AD 3AD

BD AB

2

 

 

= −  +  

   

Áp dụng định lí Cosin cho tam giác ABD ta

2 2 2

BD =AB +AD −2AB.AD.cosBAD AB= +AD −AB.AD

Trong tam giác DCK vng K có DCK=600 nên CK CD

= DK 3.CD

=

Từ ta

2

2 2 CD 3CD 2

BD BK KD BC BC CD BC.CD

2

 

 

= + = +  +  = + +

   

Do ta AB2+AD2−AB.AD BC= 2+CD2+BC.CD Mặt khác dễ thấy hai tam giác EAD EBC đồng dạng với nên ta AD EA

BC = EB Lại có hai tam giác

EBA ECD đồng dạng với nên AB EB

CD =EC Kết hợp với giả thiết AE 3CE= ta

được AD AB EA EB

BC CD = EB EC = hay AD.AC 3BC.CD= Từ ta

( ) ( )

( )

2

2 2

AB AD 2AB.AD BC CD 2BC.CD AB AD BC CD

AB BC AD CD

AB AD BC CD

AB CD AD BC

+ − = + −  − = −

 + = +

 − =  −  

+ = +

Vậy ta có điều phải chứng minh

Bài 9. Cho hình bình hành ABCD có A900 M trung điểm BC Đường thẳng AM cắt đường tròn ngoại tiếp tam giác ABC N Gọi H hình chiếu C cạnh AB Chứng minh tứ giác ADNH nội tiếp đường tròn

(12)

Theo giả thiết ta có MB MC= CH vng góc

với AB Từ ta MH MB= Đồng thời có BAN=BCN AMB CMN= nên ta

được hai tam giác AMB CMN đồng dạng với

nhau Từ suy MB AB

MN=CN Mà tứ giác ABCD hình bình hành nên AB CD=

O H

N

M C

D

B A

Do từ hệ thức ta có MN CD

MH= CN Mặt khác ta lại có HMB 2HCB= nên suy

( ) ( )

( o ) ( )

1

HMN HMB BMN HMB BN AC 2HCB BAN ANC

2

2 90 ABC BAN ANC

= + = + + = + +

= − + +

Mà ta lại có ABC=ANC Do ta suy

0

HMN 180= −ABC BAN+ =BAC BAN+ =BCD BCN+ =DCN

Đến suy hai tam giác MHN CDN đồng dạng với nên CDN MHN= Suy NDA CDA CDN= − =ABC MHN MHB MHN− = − =NHB nên tứ giác ADNH nội tiếp đường trịn

Bài 10.Cho tam giác nhọn ABC có AB AC hai đường cao BD, CE cắt H Gọi I trung điểm BC Hai đường tròn ngoại tiếp hai tam giác BEI CDI cắt K Gọi giao điểm DE với BC M Chứng minh tứ giác BKDM nội tiếp đường tròn

Lời giải Theo giả thiết ta tứ giác BEKI CDKI nội tiếp đường tròn Từ ta

0

ABC EKI IKD ACB 180+ = + = Mà ta lại có

0

BAC ABC ACB EKD EKI IKD+ + + + + =540

Do suy

BAC EKD 180+ = nên tứ giác EAKD nội tiếp đường trịn Từ ta có

ADE AKE=

M I

K H E

C D

(13)

tiếp Do ta ADE=ABC Kết hợp ADE AKE= hay ABI=AKE nên suy

0

EKI AKE EKI ABI 180+ = + = Vậy ta ba điểm A, K, I thẳng hàng Tam giác BCD vng D có I trung điểm cạnh huyền BC nên ta IKC IDC ICD= = Lại

có IKC KAC ACK= + ICD ICK KCD= + nên KAC ICK= Mà lại có KAD DEK= nên ICK=DEK tứ giác MEKC nội tiếpđường trịn Từ suy MEC MKC=

Theo kết ta có

IKC=AED MEB; MEC MEB 90 ; MKC MKI IKC= = + = + Suy MKI=900hay MK vuông góc với KI nên điểm A, E, H, I, K nằm đường trịn đường kính AH Do HK vng góc với AInên ba điểm M, H, K thẳng hàng Tứ giác DEHK nội tiếp nên ta HEK HDK= tứ giác MEKC nội tiếp nên ta KEC KMC= Do suy KMC HDK= hay KMB=BDK nên tứ giác BKDM

nội tiếp đường tròn

Bài 11 Cho tam giác ABC nội tiếp đường tròn ( )O Đường tròn ( )K tiếp xúc với AC, AB E, F tiếp xúc với đường tròn ( )O S Các đường thẳng SE,

SF cắt đường tròn ( )O M, N khác S Các đường tròn ngoại tiếp tam giác AEM, AFN cắt P khác A Gọi giao điểm EN, FM với đường tròn ( )K

lần lượt G, H Gọi giao điểm GH với MN T Chứng minh AP qua trung điểm MN tam giác AST cân

Lời giải

I H G

P O N

M

T

S K F

E

C B

A

(14)

Trước hết ta phát biểu chứng minh toán phụ Cho đường tròn ( )O và điểm T nằm ngồi đường trịn Từ T kẻ cát tuyến TNM với đường trịn ( )O Khi TA tiếp tuyến tại A đường tròn ( )O khi khi

2

TN AN

TM= AM Chứng minh

+ Điều kiện cần Giả sử TA tiếp tuyến A với đường trịn ( )O Khi đódễ thấy hai tam giác TAN TMA đồng dạng với nên ta có TA2=TN.TM Đồng thời từ hai tam giác đồng dạng ta có TN AN

TA= AM nên

2

2

TN AN

TA = AM Do ta được

2

TN AN

TM= AM + Điều kiện đủ Giả sử điểm A thuộc đường tròn ( )O thỏa mãn

2

TN AN

TM =AM Từ T kẻ tiếp tuyến TD đường tròn ( )O với D tiếp điểm nằm phía với A so với MN Khi đó theo chứng minh ta có

2

TN DN

TM = DM Mà ta lại có

2

TN AN

TM =AM nên

DN AN

DM=AM Điều dẫn đến hai điểm A D trùng nhau.Vậy TA tiếp tuyến đường trịn ( )O . Vậy tốn phụ chứng minh

Trở lại toán. Dễ thấy APF 180= −ANS 180= 0−APE nên ba điểm E, P, F thẳng

hàng Ta có APM=AEM AEM=SEC kết hợp với SEC EFS= nên EFS=PAN

Điều dẫn đến tứ giác ANFP nội tiếp đường tròn, suy APM=PAN nên AN

song song với PM Chứng minh hoàn toàn tương tự ta AM song song với PN Do tứ giác AMEN hình bình hành.Dễ thấy ta giác SKF SON đồng dạng với nên suy KF song song với ON Tương tự ta có KE song song với

OM Suy SF SK SE

SN= SO=SM nên MN song song với EF Từ HGE HFE HMN= =

nên tứ giác MNGH nội tiếp đường tròn Giả sử TS cắt đường tròn ( )O ( )K L J, ta TS.TL TM.TN TH.TG= = =TS.TJ Từ suy ba điểm S, L, J trùng nên TS tiếp tyến đường tròn ( )O Tứ giác AMPN hình bình

hành nên AP MN cắt trung điểm I đường

Ta có IAM PES= =FST=NAS AMI=AMN=ASN nên hai tam giác AIM ANS

(15)

ta AN.SM AI.SN AM.SN= = Do TS tiếp tuyến với đường trịn ( )O nên ta có

2

2

TM SM AM

TN = SN = AN nên theo toán phụ ta suy TA tiếp tuyến A

với đường trịn ( )O Do tam giác AST cân T

Bài 12.Cho tứ giác ABCD có hai đường chéo vng góc với O Gọi M, N, P, Q theo thứ tự đối xứng với O qua AB, BC, CD, DA Đường thẳng AN cắt đường tròn ngoại tiếp tam giác OMN E, đường thẳng AP cắt đường tròn ngoại tiếp tam giác OQP F Chứng minh rẳng bốn điểm M, E, F, Q thuộc đường tròn

Lời giải Vì O, M đối xứng qua AB O, N đối xứng qua BC nên tâm đường tròn ngoại tam giác OMN B Hơn AC vng góc với BD nên AO tiếp tuyến đường trịn ngoại tiếp OMN Vì hai tam giác AOB AMB nên AM tiếp tuyến đường trịn ngoại tiếp OMN.Do

AM AN= Tương tự ta có OC CN tiếp tuyến với đường tròn ngoại tiếp tam giác OMN Lại có AO AQ AM= = CO CN CP= = nên tam giác AMQ cân

F E

Q P

N M

O

D

C B

A

Do AMQ AQM= nên AME QME AQF MQF− = − Vì AM AQ tiếp tuyến đường tròn ngoại tiếp tam giác MNQ PQO nên suy AME MNE= AQF QPF= Do MNE QME QPF MQF− = − hay QME MQF MNE QPF− = − Vì

AM tiếp tuyến đường trịn ngoại tiếp tam giác MNO nên AM2=AE.AN Vì AQ

là tiếp tuyến đường tròn ngoại tiếp tam giác PQO nên AQ2 =AF.AP Khi ta có AE.AN AF.AP= nên tứ giác EFPN nội tiếp nên FEA FPN= EFA ENP= Chú ý

(16)

( ) ( )

( )

( ) ( ) ( )

( )

0

0

0

FEM EFQ FEA AEM EFA APQ FPN 180 MNE ENP 180 QFP

FPN ENP MNO QOP 180 MOA NOC

FPN ENP 180 MOA NOC 180 QOA POC

FPN ENP MOA NOC QOA POC

FPN ENP MNO ONC QPO OPC

FPN ENP MNO ONP PNC QPO OPN NPC FPN ENP

− = + − + = + − − − +

= − − + = − −

= − − − − + − −

= − + + − −

= − + + − −

= − + + + − − −

= −

( ) ( )

( ) ( )

MNO ONP QPO OPN

FPN ONP ENP ONP MNO OPQ

FPO ENO MNO QPO MON ENO QPO FPO MNE QPF

+ + − −

= − − − + −

= − + − = − − − = −

Kết hợp với QME MQF MNE QPF− = − ta FEM EFQ QME MQF− = − Từ suy

ra FEM MQF EFQ QME+ = + hay FEM MQF 180+ = nên tứ giác MEFQ nội tiếp đường tròn

Bài 13 Cho tứ giác ABCD có H, K, L chân đường vng góc hạ từ D xuống AB, BC, CA Chứng minh tứ giác ABCD nội tiếp CA AB BC

DK =DH+DL

Lời giải

Điều kiện cần.Giả sử tứ giác ABCD nội tiếp đường trịn Khi gọi ( )O ngoại tiếp tứ giác ABCD Trên

đường tròn ( )O lấy điểm N cho AN BC= suy AB NC= Từ ta ADN=BDC Gọi E giao điểm ND với AC Khi CAD CBD= Lại có

( ) ( )

1

AED sdAD sdCN sdAD sdAB BCD

2

= + = + =

N

E

L K

H

D

C B

A

Từ hai tam giác ADE BDC đồng dạng với Mà DK DL hai đường cao tương ứng nên DK AE

DL = BC hay

BC AE

DL = DK Lại có hai tam giác CDE BDA đồng dạng

với nhau, mà DK DH hai đường cao tương ứng nên CE BA

DK =DH Từ ta

BC BA AE CE AC

(17)

Điều kiện đủ Giả sử tứ giác ABCD thỏa mãn

CA AB BC

DK= DH+DL Gọi

'

D giao điểm DB với đường tròn ngoại tiếp tam giác ABC Khi tứ giác ABCD’ nội tiếp đường trịn Gọi '

H , '

K , '

L chân đường vng góc hạ từ D’ xuống AB, BC, CA Chứng minh hoàn toàn tương tự

ta có CA' ' AB' ' BC' ' D K =D H +D L

D'

L' K'

H' L

K

H

M

D

C B

A

Mặt khác theo định lí Talets ta lại có

' ' ' '

D H D B D L

DH = DB = DL

Từ ta suy

' ' ' D B.DH

D H

DB =

' ' ' D B.DL

D L

DB =

Từ ta thu

' ' '

AB BC AC.D B

DH+DL = DB.D K , mà ta lại có

CA AB BC

DK =DH+DL

Gọi M giao điểm BD AC, ta

' ' ' '

D B D K D M

DB = DK = DM Theo tính chất dãy tỉ số tađược

' '

DB D B DM D M

DB DM

− −

= nên

' '

DD DD

DB = DM

Mà ta thấy DB DM nên từ ta suy DD' =0 hay hai điểm D D' trùng

nhau Do tứ giác ABCD nội tiếp đường trịn

Bài 14.Cho hai đường tròn (O ; R1 1) (O ; R2 2) với R1 R2 cho tiếp tuyến chung

ngồi M M1 2 vng góc với tiếp tuyến chung N N1 2 điểm A Gọi P P1 2 tiếp tuyến chung thứ hai hai đường trịn điểm M ; N ; P1 1 1 thuộc đường tròn (O ; R1 1) M ; N ; P2 2 thuộc đường (O ; R2 2) Tính diện tích tam

giác AP P1 2 theo R1 R2

(18)

Dễ thấy đường thẳng O O ; N N ; P P1 2 1 2 1 2

đồng quy điểm, gọi điểm điểm

K Theo tính chất tiếp tuyến chung hai đường ta có N N1 2 =P P1 2 Lại có

ngồi M M1 2 vng góc với tiếp tuyến

chung N N1 điểm A nên ta

1 2

N N =P P =R −R

E K

A H

P2

P1 N2

N1

M2 M1

O2 O1

Mặt khác ta có O N1 1 song song vớiO N2 2 nên áp dụng định lí Thales ta có 2

1

N K R

N K = R

hay 2

1 2

N K R

N N =R +R Mà N N1 =P P1 =R1−R2 nên ta

( )

2

2

1

R R R

P K N K

R R

= =

+

Suy 2( 2)

2 2

1 2

R R R 2R R

AK AN N K R

R R R R

= + = + =

+ +

Gọi E giao điểm hai tiếp tuyến P P1 2 M M1 2

Khi ta có M M1 2 =EM1+EM2 =EM1+EP2 =EM1+EP1+P P1 2 =2EM1+P P1 2

Mặt khác ta lại có M M1 1 =R1+R1 =AN1+AN1 =2AN2 +N N1 2

Do kết hợp với N N1 2 =P P1 2 =R1−R2 ta EM1 =AN2 =R2

Từ suy AE=AM1−EM1 =R1−R2

Do ta EM2 =AE AM+ 2 =R1−R2+R2 =R1

Suy ( )

2

2 2

2 2

1 2

R R R R R

EK EP KP EM KP R

R R R R

− +

= − = − = − =

+ +

Tam giác AEK vng A có đường cao AH nên ta AH.EK AE.AK=

Từ ta tính 22( 2 2)

1

2R R R R

AE.AK AH

EK R R

= =

+

Vậy ta có ( )

1

2 2

AP P 2

1

R R R R

1

S AH.P P

2 R R

= =

+

(19)

Lời giải Gọi G giao điểm đường tròn ngoại tiếp tam giác CDF EF (F khác G) Ta

cần chứng minh hai tứ giác AOCG BOCG nội tiếp đường trịn, từ ta suy OG vng góc với EF Khi ta cần chứng minh ba điểm O, H G thẳng hàng toán chứng

minh

G

O H

E C

B

D A

F

Thật vậy, ta có tứ giác CDFG ACBD nội tiếp đường tròn nên CGE=CDA CDA=CBE Từ ta CGE CBE= , tứ giác CBGE nội tiếp đường trịn.Suy GBE GCE= , mà ta lại có GCE GFA= nên suy GBE=GFA Từ suy tứ giác ABGF nội tiếp đường tròn Do AGF ABF= kết hợp với ABF CDA= ta

AGF CDA= nên ta lại có tứ giác ADEG nội tiếp đường trịn, suy AGF CDA= Mà ta có CGE CDA= , ta AGF CGE+ =2CDA Mà ta lại có 2CDA COA= nên

AGF CGE+ =COA Suy

AGF CGE CGA COA CGA 180+ + = + = nên tứ giác AOCG nội tiếp đường trịn Ta có DGF=DCB BGE=DCB nên

DGF=BGE 2DCB+ Lại có 2DCF=BOD nên DGF BGE+ =DOB

Suy DGF BGE DGB+ + =BOD BGD 180+ = nên tứ giác BODG nội tiếp đường trịn Do OGA=OGC mà AGF CGE= , suy OGF OGE= Mà hai góc kề bù nên suy ta góc có số đo 900 Do ta OG vng góc với EF Hai đường trịn AOCG BODG có hai giao điểm O G Kết hợp với HA.HC HB.HD= suy

điểm H phải nằm đường thẳng OG hay ba điểm H, O, G thẳng hàng Vậy ta OH vng góc với EF

Bài 16. Cho tứ giác ABCD nội tiếp đường trịn tâm O có AC vng góc với BD Các tiếp tuyến A, B với đường tròn ( )O cắt X, tiếp tuyến B,C với đường

(20)

Lời giải

L O' O Q

P

N M

I

D

C B

A

I8

I7 I6

I5 I4

I3

I2 I1

O T

Y Z

X

D

E

C

B A

Trước hết ta phát biểu chứng minh bổ đề Cho tứ giác ABCD ngoại tiếp đường tròn ( )I với M, N, P, Q theo thứ tự tiếp điểm ( )I với AB, BC, CD, DA Gọi O giao điểm của AC BD Khi O giao điểm hai đoạn thẳng MP, NQ tứ giác ABCD nội tiếp khi MP vng góc với QM.

Chứng minh.Bài tốn có hai trường hợp

+ Trường hợp 1.Tứ giác ABCD có cặp cạnhđối diện song song với nhau, bổ để hiển nhiên

+ Trường hợp 2.Tứ giác ABCD khơng có cặp cạnh đối diện song song với

Khi gọi O ' là giao điểm AC MP Qua A kẻ AL song song với CD với L

thuộc MP

Ta có ALM DPM 1sdMQP AML

2

= = = nên ta AL AM= Theo định lí Talet ta có

' '

O A LA

CP

O C = hay

' '

O A MA

CP

O C =

Hoàn toàn tương tự ta gọi O " là giao điểm AC NQ, ta " "

O A QA

CN

O C =

Để ý MA AQ= CP CN= nên ta

' " ' "

O A O A

O C = O C , suy

'

O O trùng "

Chứng minh tâm đường tròn nội tiếp tam giác ETX, EXY, EYZ, EZT, TXY,

(21)

( )

( ) ( ) ( )

0

0 0

POQ 360 PDQ OQD OPD

360 180 QIP 90 NQI 90 MPI QIP NQI MPI

= − + +

= − − − − − − = + +

Hay ta QIP POQ= −(NQI MPI+ )

Mà ta có NQI MPI INQ PMI+ = + , ta QIP POQ= −(INQ PMI+ )

Mặt khác ta có MIB IMO INO MON= + +

Từ suy QIP MIN POQ+ = −(INQ PMI+ )+IMO INO MON+ + =2MON

Do

MON=90 hay QIP MIN 180+ = 0 nên PDQ MBN 180+ = 0 Do tứ giác

ABCD nội tiếp đường tròn Vậy bổ đề chứng minh xong

Trở lại toán.Gọi I ; I ; I ; I ; I ; I ; I ; I1 2 3 4 5 6 7 8 tâm đường tròn nội tiếp tam giác ETX, EXY, EYZ, EZT, TXY, XYZ, YZT, ZTX Theo bổ đề ta có E thuộc hai đường chéo XZ, YT tứ giác XYZT nội tiếp đường trịn Từ ta ZTY ZXY= Mà EDT EBX= nên ta TED XED= Kết hợp với AC vng góc với BD ta

AET=AEX, điểm I1 thuộc EA Hồn tồn tương tự ta chứng minh

điểm I2 thuộc EB Mà ta lại có XA=XB nên ta có

1

1

I A XA XB I B

I E = XE =XE =I E.Từ theo

định lí Talets ta suy I I1 2song song với AB Chứng minh hoàn toàn tương tự ta I I2 3song song với BC, I I3 4song song với CD, I I4 1song song với DA.Mà tứ giác ABCD nội tiếp đường tròn nên ta DAB DCB 180+ = Do ta

0 4

I I I +I I I =180 , suy tứ giác I I I I1 nội tiếp đường trịn.Lại có

0 0

1 5

XTY XYT 1

I I I I I I TI Y ACB 180 XAB 90 TXY 90 AXB 180

2 2

+

+ = + = − + = + + − =

Do tứ giác I I I I1 3 nội tiếp đường trịn Chứng minh hồn tồn tương tự ta tứ giác I I I I2 4, I I I I1 4, I I I I4 nội tiếp đường tròn Vậy điểm

1

I ; I ; I ; I ; I ; I ; I ; I nằm đường tròn

(22)

Bài 17.Cho đường tròn ( )I nội tiếp tam giác ABC, tiếp xúc với BC AC

D E Đường thẳng AD cắt đường tròn ( )I điểm thứ hai P Giả sử

BPC 90=

Chứng minh AE AP+ =PD

Lời giải Trước hết ta phát biểu chứng minh hai bổ đề sau

Bổ đề 1.Cho tam giác ABC điểm D, E đường thẳng BC cho DB EB

DC =EC

(với BD BC BE  DAE 90= 0) Khi AD AE theo thứ tự đường phân giác phân giác tam giác ABC

L K

E C

D B

A

Chứng minh. Qua D kẻ đường thẳng KL song song với AE(K AB; L AC  ) Do AD

vng góc với AE nên AD vng góc với KL Theo định lí Talets ý đến

DB EB

DC =EC, BD BC BE  ta có

KD KD AE BD CE BD CE

KD LD

LD = AE LD= BE CD=CD BE =  =

Từ suy tam giác KAL cân A nên AD đường phân giác góc BAC, kết

hợp với

DAE 90= ta suy AD AE theo thứ tự đường phân giác phân giác tam giác ABC.Vậy bổ đề chứng minh

Bổ đề 2.Cho tứ giác ABCD nội tiếp đường tròn

( )O Các tiếp tuyến với đường tròn ( )O A, C cắt S ba điểm S, B, D thẳng hàng AB.CD AD.BC=

Chứng minh. Giả sử ba điểm S, B, D thẳng hàng Khi ta có SAB SDA= SCB SDC= nên suy

SAB SDA

 ∽  SCB ∽ SDC

O

D'

S

D

C B

A

Kết hợp với SA SC= ta AB SA SC CB

(23)

Ngược lại, giả sử AB.CD AD.BC= Khi gọi '

D giao điểm tứ hai SB với đường tròn ( )O Chứng minh tương tự ta AB.CD' =AD BC'

Mà ta lại có AB.CD AD.BC= nên ta thu

' '

CD AD

CD = AD

Kết hợp với CD A CDA' = ta CD A' ∽ CDA Do suy CAD' =CAD, suy

ra hai tia AD AD’ trùng nên hai điểm D D’ trùng nhau.Vậy ba điểm S, B, D thẳng hàng Bổ đề chứng minh

I F

E

Q

K L

P

D C

B

A

Trở lại toán.Gọi F tiếp điểm của đường tròn ( )I với AB, K giao điểm BC EF Lấy đểm L thuộc đoạn EF cho CL song song với AB Gọi Q giao điểm thứ hai PC với đường tròn ( )I

Theo định lí Thales ý AE AF; BD BF; CD CE= = = ta có

KB BF BF AF BF AF BF BD CK BK

KD =CL= AF CL=AF CE =CE=CDCD=BD

Ta có PC vng góc với PB theo bổ đề ta QPK CPK CPD QPD= = =

Áp dụng bổ đề cho tứ giác EPFD ta EP.DF FP.DE= Áp dụng bổ đề cho tứ giác EPFD ta KP tiếp xúc với đường tròn ( )I Từ ta suy QPK QDP=

Do QPD QDP= Từ dẫn đến tam giác DPQ cân Q nên suy QD QP= Lại áp dụng bổ đề định lí Ptoleme cho tứ giác PEQD nội tiếp ta

2PE.PQ 2PE.DQ PE.DQ PD.EQ DE.QP= = + =

Tờ suy 2PE=DE Chú ý đến hai tam giác APE AED đồng dạng với ta 2AE AD; 2AP AE= =

Do ta có AE AP+ =2AE AE AP− + =AD 2AP AP− + =AD AP− =DP

(24)

Bài 18. Cho hình chữ nhật ABCD đường tròn ( )O tiếp xúc với cạnh AB, BC, DA Từ C kẻ tiếp tuyến với đường tròn ( )O cắt AD E Vẽ đường ( )I nội tiếp tam giác DEC đường tròn ( )J tiếp xúc với CB, CE tiếp xúc ngồi với đường trịn (O) Biết hai đường trịn ( )I ( )J có bán kính Tính tỉ số AB AD

Lời giải

Trước hết ta phát biểu chứng minh bổ đề. Cho hai đường tròn (O; R) ( )I; r tiếp xúc ngoài với Gọi BC tiếp tuyến chung ngồi hai đường trịn, ta có BC=2 R.r

Chứng minh. Gọi A giao điểm hai đường

tròn ( )O ( )I Vẽ tiếp tuyến chung A hai đường tròn, tiếp tuyến cắt BC M

C B

M

A I

O

Theo tính chất hai tiếp tuyến cắt ta có MA MB= MB MC= nên suy

được MA MB MC BC

2

= = = Do tam giác ABC vng A nên ta BAC 90=

Lại theo tính chất hai tiếp tuyến cắt ta có MO phân giác BMA MI phân giác AMC mà BMA AMC hai góc kề bù nên

OMI=90 Theo hệ thức lượng tam giác vng ta có MA2 =OA.IA R.r= nên suy MA= R.r Do ta BC=2MA=2 R.r

Trở lại tốn Ta kí hiệu d(X,YZ) khoảng cách từ điểm X đến đường thẳng YZ Gọi T tiếp điểm hai đường tròn ( )O ( )J Gọi F giao điểm BC tiếp tuyến với đường

tròn ( )J kể từ E (khác EC) Gọi G giao điểm TF AD Gọi M, N giao điểm đường tròn ( )J với EF, BC Gọi P giao điểm thứ hai đường tròn ( )O với TM

T P

S Q

N M E

J L O

K I

G

D C

(25)

tiếp điểm CE với đường tròn ( )O , ( )J Đặt R, r bán kính đường

tròn ( )O , ( )J CL CN x= = Ta thấy EF, CD, OQ, JN song song với GP

song song với EF nên ta GP song song với IN GP JN trùng nhau.Lại có T, N, Q thẳng hàng nên ta có ( )

( )

( ) ( )

T ,PG T ,JN

T ,EF T ,EF

d TP OP OQ TQ d

d =TM= JM = JN =TN =d

Kết hợp với OP OQ R

JM = JM = r  nên ta d(T,PG)=d(T,JN) d(T,EF)

Từ ý T, PG, JN thuộc nửa mặt phẳng bờ EF nên ta suy PG JN trùng Điều có nghĩa GN song song với QS Lại thấy tứ giác OPGQ hình vng kết hợp theo bổ đề ta thu R=OP=OQ SN= =2 R.r nên suy

R=4r Ta có tứ giác JNFM hình vng nên suy

EC EK KL LC EQ SN LC SF SN LC 2SN FN LC 2R r x= + + = + + = + + = − + = − +

Hay ta EC 2R 1R x 7R x

4

= − + = + Mà ta có

EFC 90= EF 2R; FC 1R x

= = +

Nên theo định lí Pitago ta ( )

2

2

7 1

R x 2R R x x R

4

 +  = + +   =

   

   

Do ta 6AD 6BC BN NC( ) 2R 1R 7.2R 7AB

 

= = + =  + = =

  hay

AB

AD=7

Bài 19.Cho tứ giác ABCD nội tiếp đường trịn có I giao điểm hai đường chéo Gọi I ; I ; I ; I1 2 3 4 theo thứ tự tâm đường tròn nội tiếp tam giác IAB, IBC, ICD, IAD Chứng minh tứ giác I I I I1 nội tiếp tứ giác ABCD ngoại tiếp

Lời giải

Trước hết ta phát biểu chứng minh bổ đề.Nếu I tâm đường trịn nội tiếp tam giác

ABC ta có AI2 AB.AC AB AC BC( )

AB BC CA + − =

+ +

(26)

Chứng minh Gọi D giao điểm AI với BC, E giao điểm thứ hai AI với đường tròn ngoại tiếp tam giác ABC Khi ta thấy AEB∽ ACD

DEB DCA

 ∽  nên ta AE AC

AB=AD

DE DC

DB= DA

Từ ta AE.AD AB.AC= DE.DA DB.DC=

Do (AE DE AD AB.AC DB.DC− ) = −

I

E

D C

B

A

Hay ta AD2 =AB.AC DB.DC− Ta có AD đường phân giác tam giác ABC

nên ta BD AB

CD= AC suy

BD AB

BD CD+ =AB AC+ hay

AB.BC DB

AB AC =

+ Hoàn toàn

tương tự ta CD AC.BC AB AC =

+ nên ta

( ) ( ) ( )( )

2

2

AB.AC.BC AB.AC

AD AB.AC AB BC CA AB AC BC

AB AC AB AC

= − = + + + −

+ +

Mà ta lại có AI phân giác tam giác ABD nên AI AB AB AB AC

AB.BC

DI BD BC

AB AC

+

= = =

+

Suy AI AB AC

AD AB BC CA

+ =

+ + Từ ta

( )

2 AB.AC AB AC BC

AI

AB BC CA + − =

+ +

Trở lại toán Do tứ giác ABCD nội tiếp đường tròn nên ta IAB∽ IDC

IBC IAD

 ∽  Do IA IB AB IC ID CD

IA IB AB IC ID CD

+ − + −

=

+ + + +

IB IC BC IA ID AD

IB IC BC IA ID AD

+ − + −

=

+ + + +

Theo bổ đề ta có

( )

2

IA.IB IA IB AB II

IA IB AB + − = + + ; ( )

IC.ID IC ID CD II

IC ID CD + − = + + ( ) 2

IB.IC IB IC BC II

IB IC BC + − = + + ; ( )

ID.IA ID IA DA II

ID IA DA + − =

+ +

+ Điều kiện cần.Giả sử tứ giác I I I I1 nội tiếp đường

tròn Khi ta có II II1 3 =II II2 4

I4 I3 I2 I1 I D C B A

Từ ta IA IB AB IC ID CD IB IC BC IA ID AD

IA IB AB IC ID CD IB IC BC IA ID AD

+ − + − = + − + −

+ + + + + + + +

Kết hợp với kết ta

IA IB AB IC ID CD IB IC BC IA ID AD

IA IB AB IC ID CD IB IC BC IA ID AD

+ − = + − = + − = + −

(27)

Do ta AB CD BC DA

IA IB+ =IC ID+ = IB IC+ =IA ID+

Nên suy AB CD BC DA

IA IB IC ID IB IC ID IA

+ +

=

+ + + + + + hay ta AB CD BC DA+ = +

Điều có nghĩa tứ giác ABCD ngoại tiếp đường tròn

+ Điều kiện đủ.Giả sử tứ giác ABCD ngoại tiếp đường tròn ta cần chứng minh tứ

giác I I I I1 4 nội tiếp đường tròn

Thật giả sử tứ giác I I I I1 4 khơng nội tiếp đường trịn Khi ta có II II1 3 II II2 4

Từ ta IA IB AB IC ID CD IB IC BC IA ID AD

IA IB AB IC ID CD IB IC BC IA ID AD

+ − + − + − + −

+ + + + + + + +

Kết hợp với kết quảtrên ta

IA IB AB IC ID CD IB IC BC IA ID AD

IA IB AB IC ID CD IB IC BC IA ID AD

+ − = + −  + − = + −

+ + + + + + + +

Do ta AB CD BC DA

IA IB+ =IC ID+  IB IC+ = IA ID+

Nên suy AB CD BC DA

IA IB IC ID IB IC ID IA

+  +

+ + + + + + hay ta AB CD BC DA+  +

Điều có nghĩa tứ giác ABCD khơng thể ngoại tiếp đường trịn Điều mâu thuẫn với giả thiết tứ giác ABCD ngoại tiếp đường tròn Vậy điều giả sử sai hay tứ

giác I I I I1 4 nội tiếp đường trịn Vậy tốn chứng minh

Bài 20. Cho tứ giác ABCD có

ADC=BCD 90= Lấy điểm E cạnh CD Các đường cao AM, BN tam giác ABE cắt H Gọi giao điểm DM CN K giao điểm KH CD L Chứng minh KH=KL

Lời giải

Trước hết ta phát biểu chứng minh bổ đề Cho tam giác ABC có đường cao AD, BE, CF cắt H Gọi I trung điểm AH Khi bốn điểm D, E, F, I thuộc đường tròn.

Chứng minh. Dễ thấy HIE=2HAE; HIF=2HAF nên

ta EIF=2BAC Do tứ giác BDHF nội tiếp đường

H I F

E

D C

B

(28)

tròn nên ta có HBF HDF=

Do tứ giác CDHE nội tiếp đường trịn nên ta có HDE=HCE Do tứ giác BCEF nội tiếp đường trịn nên ta có EBF FCE= Từ suy EDF 2FBH=

Do ta ( ) 0

EDF EIF FBH BAC+ = + =2.90 =180 Suy tứ giác DEIF nội tiếp đường tròn Bổ đề chứng minh

Trở lại toán.Gọi P giao điểm EH với

AB Q trung điểm EH Khi theo bổ đề ta bốn điểm M, N, P, Q thuộc đường trịn Do ta có

0

ADE=AME=APE 90= =BCE=BNE=BPE Suy đa giác ADEMP BCENP nội tiếp đường trịn Do ta

H

L

K

N Q

P M

E

D

C B

A

( ) ( )

0

0

0

0

0

MKN 180 KMN KNM 180 EMN EMK ENM ENK

180 EMN ENM EMK ENK MEN EMD ENC

AEB EAD EBC AEB 90 DEA 90 CEB

AEB 180 DEA CEB AEB BEA NEB MEA

APN BPN 180 MPN

= − − = − − − −

= − − + + = + +

= + + = + − + −

= + − − = + = +

= + = −

Từ ta K thuộc đường trịn ngoại tiếp tam giác MNP nên K thuộc đường tròn ngoại tiếp tam giác MPQ Do suy EDM EPM QPM QKM= = = nên ta DE

song song với KQ.Kết hợp với QH QE= ta KH=KL

Bài 21.Cho tứ giác ABCD điểm P, Q nằm tứ giác cho tứ giác ABPQ CDPQ nội tiếp đường tròn Giả sử tồn điểm E thuộc đoạn PQ thỏa mãn

EAQ EBP= EDQ ECP= Chứng minh tứ giác ABCD nội tiếp đường tròn Lời giải

(29)

E Q P

S' S

D C

B

A

A

B

C D

S

S' Q E P

+ Trường hợp Điểm S nằm tia đối tia BA Do góc BPS góc ngồi tam giác BPE Lại có tứ giác ABPQ nội tiếp đường trịn EAQ EBP= nên ta

BES BPS EBP SAQ EAQ SAE= − = − =

Từ suy SE tiếp tuyến đường tròn ngoại tiếp tam giác ABE

+ Trường hợp 2 Điểm S nằm tia đối tia AB.Khi Do góc BES góc ngồi

của tam giác BPE Lại có tứ giác ABPQ nội tiếp đường trịn EAQ EBP= nên ta có BES BPE EBP SAQ EAQ SAE= + = + = Từ BEP 180= 0−BES 180= 0−SAE=BAE Suy SE tiếp tuyến đường tròn ngoại tiếp tam giác ABE

Như hai trường hợp ta SE tiếp tuyến đường tròn ngoại tiếp tam giác ABE Từ dễ dàng chứng minh SA.SB SE= Do tứ giác ABPQ nội tiếp đường tròn nên ta có SA.SB SP.SQ= Từ suy SE2 =SP.SQ Chú ý

là xét trường hợp ta thu

+ Nếu S nằm tia đối tia BA ta

( )( ) ( )

2

SE =SP.SQ= SE EP SE EQ− + =SE + EQ EP SE EP.EQ− − Suy SE EQ EP

EP.EQ −

=

+ Nếu S nằm tia đối tia AB, lập luận tương tự ta thu SE EP EQ EP.EQ

− =

Như kết hợp lại ta SE EP EQ EP.EQ

=

Hoàn toàn tương tự gọi S’ giao điểm PQ CD Xét trường hợp ta chứng minh S' E EP EQ

EP.EQ −

(30)

SA.SBSC.SD , suy tứ giác ABCD nội tiếp đường tròn

Bài 22. Cho tam giác ABC có góc BAC 90 Giả sử P điểm thuộc miền tam giác ABC cho BAP=ACP CAP=ABP Gọi M N lầ lượt tâm đường tròn nội tiếp tam giác ABP ACP,R độ dài đường tròn ngoại tiếp tam

giác AMN Chứng minh 1 1

R = AB+AC+AP

Lời giải Gọi E giao điểm PM AB, F giao điểm PN AC Từ giả thiết ta có hai

tam giác ABP CAP đồng dạng với

Do ta APB APC 180= = −BAC Suy EPF 1(APB APC) 1800 BAC

2

= + = −

nên tứ giác AEPF nội tiếp đường tròn Mặt

khác APB=APC ta APE APF= ,

đó ta AE=AF

F

E

P N M

C B

A

Theo tính chất đường phân giác ta có PM AP AP PN

EM=AE = AF = PF

Do M, N tâm đường tròn nội tiếp tam giác ABP ACP nên AM AN phân giác góc EAP FAP Do ta MNsong song với EF nên

ta MN MP MP AP

EF = PE =ME MP+ =AE AP+ Mặt khác lại có

BAC EF 2AE.sin

2

= nên ta

được MN EF.PM EF.AP 2AE.AP.sinBAC

PE AP AE AP AE

= = =

+ + Áp dụng định lí sin cho tam giác

AMN ta có sin MAN sinBAC AP AE 1

R MN MN AE.AP AE AP

+

= = = = +

Mặt khác hai tam giác ABP CAP đồng dạng với nên ta có AB.AP AC.BP= AC.AP AB.CP= Do AB.AP AB.CP AC.BP AC.AP+ = + hay AB.AP AC.AP

BP AP+ = AP CP+

(31)

Đồng thời BP

AC= AB.AP

1 CP

AB= AC.CP

giác ta có AE BE AE BE AB

AP BP AP BP AP BP

+

= = =

+ + nên

AP.AB AE

AP BP =

+ Tương tự ta có AC.AP

AF

AP CP =

+ Do ta

AB.AP AC.AP

AE AF

BP AP AP CP

= = =

+ + Từ suy

1 BP AP AP CP 1 BP CP 1

AE AB.AP AC.AP AB AB AB.AP AC.CP AB AC

 + +   

=  + =  + + + = +

   

Do ta 1 1

R = AB+BC+CA

Bài 23 Cho tứ giác ABCD nội tiếp đường tròn tâm O Gọi I J trung điểm BD AC Chứng minh BD tia phân giác góc AIC AC tia phân giác góc BJD

Lời giải

Trước hết ta phát biểu chứng minh công thức. Cho tam giác ABC nội tiếp đường tròn (O; R) Khi ta

ln có SABC AB.BC.CA 4R

=

Chứng minh.Vẽ đường kính ADcủa đường trịn ( )O Khi ta có ACB ADB=

D O

C B

A

Mà ta có tam giác ABD vuông B nên sin ADB AB AB

AD 2R

= = Do sin C AB

2R =

Ta có SABC 1BC.CA.sin A 1BC.CA.AB AB.BC.CA

2 2R 4R

= = =

Trở lại toán.Kẻ CMsong song với BD với M nằm đường tròn O Khi ta có

BM CD= DBM=BDC Do I trung điểm BD nên IB ID= , từ ta hai

tam giác BIM DIC nên suy BIM=DIC

(32)

+ Điều kiên cần.Nếu BD phân giác góc AIC

thì ta AID=DIC Mà lại có BIM=AID nên

ba điểm A, I, M thẳng hàng Gọi R bán kính đường trịn ( )O , ta có SABM AB.BM.MB

4R

=

ADM

AD.DM.MB S

4R

= Vì I trung điểm AB nên

ta SAMB =SAMD Suy AB.BM AD.DM=

N

M O

J I

C D

B

A

Lại có CD BM, BC DM= = nên ta có AB.CD AD.BC= Kẻ DNsong song với AC với N nằm đường tròn ( )O ta AD CN= AN CD= Mặt khác ta lại có

ABN

AB.BN.NA S

4R

= SCBN BC.CN.NB

4R

= Do SABN =SCBN Điều chứng tỏ DN

đi qua trung điểm J BD Khi ta có AD NC= DAJ=NCJ nên suy AJD CJN AJB= = Do AD phân giác góc BJC

+ Điều kiện đủ.Nếu AD phân giác góc BJC Khi lặplại chứng minh ta BD phân giác góc AIC

Vậy toán chứng minh

Bài 24 Cho tứ giác ABCD có diện tích S nội tiếp đường trịn bán kính R Biết độ dài cạnh tứ giác AB a, BC b, CD c, DA d= = = = AC e= Giả sử tồn đường tròn tiếp xúc với tia đối BA, DA, CD, CB 2p a b c d= + + + Chứng minh R 2S.e 2

p e

=

2 2 8SR

a b c d 2p

e

+ + + + =

Lời giải

Trước hết ta nhắc lại định lí Ptoleme cơng thức diện tích tam giác sau

+ Định lí Ptoleme.Cho tứ giác ABCD nội tiếp đường trịn ( )O Khi ta ln có:

AD.BC AB.CD AC.BD+ =

+ Cơng thức diện tích tam giác.Cho tam giác ABC nội tiếp đường trịn (O; R)

đó ta ln có SABC AB.BC.CA 4R

(33)

O Q

P

N

M D

C

B A

Trở lại toán.Gọi ( )O đường tròn tiếp xúc với tia đối BA, DA, CD, CB theo thứ tự M, N, P, Q Khi theo tính chất hai tiếp tuyến cắt ta có

AM AQ; BM BP; CN CP; DN AQ.= = = = Khi ta thấy

AM AB BM AB BP AB BC CP

AQ AD DQ AD DN AD DC CN

= + = + = + +

= + = + = + +

Do ta có AM AQ; CN CP= = nên ta AB BC AD DC+ = +

Mà lại có 2p a b c d= + + + nên ta a b c d p+ = + =

Đặt BD f= Khi ta có SABCD =SABC+SADC =SABD+SBCD

Áp dụng cơng thức diệntích cho tam giác ABC, ADC, ABD, BCD nội tiếp đường tròn ta

ABC ACD

ABD BCD

AB.BC.CA abe AC.CD.DA cde

S ; S

4R 4R 4R 4R

AB.BD.DA adf BC.CD.DB bcf

S ; S

4R 4R 4R 4R

= = = =

= = = =

Do ta SANCD (ab cd e) 4R +

= SANCD (ad bc f)

4R + =

Nhân theo vế hai đẳng thức ta

( )( ) ( )( ) ( )

2 2

ABCD

ab cd ad bc ef

S 16S R ab cd ad bc ef *

16R

+ +

=  = + +

Áp dụng định lí Ptoleme cho tứ giác nội tiếp ABCD ta

AD.BC AB.CD AC.BD+ = hay ac bd ef+ =

Từ SANCD (ab cd e) 4R +

= ta ab cd 4R.S

e

+ = thay vào hệ thức ( )* ta

( )( ) ( ) ( ) ( ) ( )

( )

2 2 2

2

2

4 Re.S ac bd ad bc ab c d cd a b ab p 2cd cd p 2ab

4Rp S

p ab cd 4abcd 4S

2

= + + = + + + = − + −

(34)

Do ta Rp2 =Re2+e.S hay

2

S.e R

p e

= −

Mặt khác ta có a2 b2 c2 d2 (a b) (2 c d)2 2ab 2cd 2p2 4SR e  

+ + + = + + + − − = −  

 

Do ta a2 b2 c2 d2 8SR 2p2 e

+ + + + =

Bài 25.Cho tam giác ABC có E, F tuộc đoạn CA BA cho EF song song với BC

Đường trung trực đoạn thẳng BC cắt AC M, đường trung trực đoạn EF cắt cắt AB N Đường tròng ngoại tiếp tam giác BCM cắt CF P khác C, đường ngoại tiếp tam giác EFN cắt CF Q khác F Chứng minh đường trung trực PQ qua trung điểm MN

Lời giải Gọi H K theo thứ tự hình chiếu M N CF Gọi S, T, U, V theo thứ tự trung điểm MN, HK, BC, EF Dễ thấy tứ giác BCMP NEFQ nội tiếp đường trịn Do ta MPH MBU= NQK VEN=

Lại có MHP=MUB 90 ; NKQ= =NVE 90= Suy MHP ∽ MUB NKQ∽ NVE

nên ta HP UB KQ; VE

MP =MB NQ =NE

N

V

U S

T Q

P M

H K

F E

C B

A

Để ý U, V trung điểm BC EF nên ta có

HP UB BC MP

HP MP MP

MP MB MB

KQ VE EF NQ

KQ NQ NQ

NQ NE NE

= = =

 

 = = =



Do tứ giác BCMP NEFQ nội tiếp lại có EFsong song với BC nên ta

MBP MCP ECF; PMB PCB EFC NQE NFE CBF; QNE EFC BCF

 = = = =

 

= = = =

(35)

Do ta MBP∽ FCE NQE∽ CBF suy MP FE MB=FC

NQ CB

NE =CF Từ

đó ta HP BC.EF KQ 2CF

= = Kết hợp với TP TQ= suy TH TK= Mà ta có MH

song song với NK SM SN= nên suy ST, MH, NK song song với Điều có nghĩa SI đường trung trực QP Từ ta đường trung trực PQ qua trung điểm MN

Bài 26. Cho tam giác ABC nhọn nội tiếp đường tròn ( )O với đường cao AD Tiếp tuyến B C vớiđường tròn ( )O cắt T Trên đoạn thẳng AD lấy K cho

0

BKC=90 Gọi G trọng tâm tam giác ABC, KG cắt OT L Lấy điểm P Q

thuộc đoạn BC cho LPsong song với OB LQ song song với OC Các điểm E

F thuộc đoạn AC AB cho QE PF vng góc với BC Gọi ( )O

đường tròn tâm T qua B, C Chứng minh đường tròn ngoại tiếp tam giác AEF tiếp xúc với đường tròn ( )T

Lời giải Gọi H trực tâm tam giác ABC, gọi M I trung điểm BC EF Gọi N Y theo thứ tự giao điểm BH TO với AC, X giao điểm hai đường tròn ngoại tiếp hai tam giác BMF CME Ta thấy G giao điểm OH MA Điểm M I thuộc đoạn OT Xét

hai tam giác AHN YCM có

0

ANB CMY= =90 HAN MYC= nên

hai tam giác AHN YMC đồng dạng với

N O Y

Q P

F

E I

L G K

X

T H

D C

B A

Suy ta AH HN

CY =CM Lại có OM song song với HA ba điểm H, G, O thẳng

hàng nên theo định lí Talet ta KA LM

(36)

Mà LQ song song với CO nên theo định lí Talets ta có LM QM

LO = QC Mà QE song song

với MY nên ta QM EY

QC =EC Từ suy

KA LM QM EY

KH= LO = QC =EC Do ta

KA KH EY EC

KH EC

+ +

= nên AH CY

KH = CE hay

AH KH

CY = CE Do ta

AH HN KH

CY =CM= CE

nên suy hai tam giác KHN ECM đồng dạng với Mặt khác ta thấy tứ giác BKNC nội tiếp nên KCB KHN EMC= = Tương tự ta KBC=FMB

Do FME 180= 0−FMB EMC 180− = 0−KBC KCB− =BKC 90= Ta có biến đổi góc

( ) ( )

0 0

FXE=360 −FXM EXM− = 180 −FXM + 180 −EXM =FBM ECM 180+ = −FAE

Do tứ giác AFXE nội tiếp đường trịn Lại có

0 0

0

0 0

BXC 360 FXB EXC FXE 180 FMB EMC 180 FXE

180 2BAC BTC

FME BAC 90 BAC 180 180

2

= − − − = − − + −

= + = + = − = −

Do X thuộc đường trịn tâm T Từ kết suy X giao điểm đường tròn ngoại tiếp tam giác AEF với đường tròn ( )T Gọi X’ giao điểm thứ hai đường tròn ngoại tiếp tam giác AEF với đường trịn ( )T Khi ta lại có BXF=BXX ' FXX '+ =BCX ' FEX '+ Lại có tứ giác BFXM nội tiếp đường trịn nên

0

EMF EQM 90= = , EQ song song với IM IM IE IF= = Từ đóta có biến đổi góc

FXB FMB MEQ EMI IEM IEX MEX FEX MCX FEX BCX= = = = = + = + = +

Kết hợp kết lại ta BCX ' FEX ' FEX BCX+ = + , suy X X’ trùng

Vậy đường tròn ngoại tiếp tam giác AEF tiếp xúc với đường tròn ( )T

Bài 27.Cho tam giác ABC điểm D cạnh BC(D khác B C) Gọi E F tâm đường tròn nội tiếp tam giác ABD ACD Chứng minh bốn điểm B, C E, F nằm đường trịn ta có AD DB AB

AD CD AC

+ =

+

Lời giải

(37)

S R

M

N I

P

Q F

E

D C

B

A

Trước hết ta biểu diễn tỉ số có chứa AD BD+ AD CD+ Theo

tính chất đường phân giác tam giác ta

AD AM AD DB AM BM AB AD BD AD

BD BM AD AM AM AB AM

AD AN AD DC AN NC AC AD DC AD

CD CN AD AN AN AC AN

+ + +

=  = =  =

+ + +

=  = =  =

Như toán chứng minh ta AM AN=

Thật vậy, tứ giác BEFC nội tiếp nên ta APQ ABE PEB ABE BCF= + = +

Từ ta APQ 1(ABC ACB)

= + Chứng minh hoàn toàn tương tự ta

( )

1

AQP ABC ACB

2

= + Như tam giác APQ cân A nên AP AQ= Qua A kẻ đường thẳng song song với PQ cắt DM, DN lầnlượt R, S

Ta có EP AP AQ FQ

EI = AI = AI = IF nên ta

IE EP

IF =FQ

Theo định lí Thales ta có IE AR

IF = AS Từ ta

EP AR

FQ = AS nên suy

EP FQ

AR = AS

Mặt khác ta lại có EP MP

AR =MA

FQ QN

AS= AN, từ ta

MP NQ

MA=NA nên MN song

song với PQ Mà AP AQ= nên suy AM AN= Từ ta AD DB AD DC

AB AC

+ = +

Do ta suy AD DB AB

AD CD AC

+ =

+ Vậy ta có điều phải chứng minh

(38)

Lời giải Trước hết ta phát biểu hai bổ đề sau

+ Bổ đề 1.Cho tam giác ABC hai điểm M, N cho

2

BM.BN AB

CM.CN = AC Khi ta ln có BAM=CAN

+ Bổ đề Cho điểm A nằm ngồi đường trịn ( )O cát tuyến ABC Các tiếp tuyến B, C với đường tròn cắt K AE AF tiếp tuyến với đường tròn ( )O kẻ từ A (E, F là tiếp điểm) Khi ba điểm K, E, F thẳng hàng.

Trong hai bổ đề bổ đề thứ tính chất hai điểm đẳng giác bổ đề thứ hai toán quen thuộc hai tiếp tuyến cắt Trở lại toán. Gọi M, N, P, Q

các tiếp điểm đường tròn với cạnh AB, BC, CD, DA Lấy diểm D’ đối xứng với Aqua AC Gọi giao điểm BD’ với AC H’ Dựng OH vng góc với AC, ta cần chứng minh hai điểm H H’ trùng Gọi S giao điểm MP BD, kẻ BK song song với CD với K thuộc MP, ta

có BS BK

DS= DP Mà ta có BKM CPM= =BMK

nên ta BK BM=

I H'

K

H

E'

D'

Q

P

N M

F

E

D C

B A

Do BS BM

DS= DP Gọi S’ giao điểm NQ BD, tương tự ta có

BS' BN

DS'=QD Do BM BN= DP DQ= nên ta

BS BS'

DS= DS', suy hai điểm S S’

trùng Từ suy ba đường thẳng BD, MP, NQ đồng quy S.Chứng minh tương tự ta AC, MP NQ đồng quy S.Từ cách dựng điểm H’ ta H’C tia phân giác góc BH' D nên suy BH' BS BM

DH'= DS = DP

Mặt khác ta có BE.BF BM , DE.DF DP= = 2, suy

2

H' B BE.BF

(39)

Áp dụng bổ đề cho tam giác H’BD ta BH'E=DH'F Từ FH' C EH' C= Gọi I I’ giao điểm NM, PQ với AC Áp dụng định lí Menelaus cho tam giác ABC với ba điểm M, N, I thẳng hàng tam giác ACD với ba điểm P, Q, I’ thẳng

hàng ta AM BN CI AQ DP CI '

BM CN AI = DQ CP AI '= Từ suy

CI CI'

AI = AI', từ suy

hai điểm I I’ trùng Điều có nghĩa MN, AC, PQ đồng quy I Giả sử tiếp tuyến với đường tròn (O) E, F cắt I” theo bổ đề I” thuộc MN thuộc PQ Nên hai điểm I I” trùng Tứ giác HEIF nội tiếp đường

trịn đường kính OI nên ta EHC FHC=

Gọi E’ điểm đối xứng với E qua AC, ta có E'H'C EH'C FH'C

E'HC EHC FHC

 = =

 

= =



Từ suy ba điểm F, E’, H’ thẳng hàng F, E’ H thẳng hàng.Từ suy hai điểm H H’ thẳng hàng, kết hợp với BH'E=DH'F ta BHE DHF=

Vậy toán chứng minh

Bài 29. Cho tam giác ABC nội tiếp đường tròn ( )O Đường tròn ( )O ' tiếp xúc

với đường tròn ( )O tiếp xúc với cạnh AB, BC R, P, Q Gọi K tâm đường tròn nội tiếp tam giác ABC Chứng minh ARK=CRK

Lời giải Tại R vẽ tiếp tuyến với đường tròn

( )O , tiếp tuyến cắt BC D Khi

đó dễ thấy DR=DQ Từ ta

2

DC.DB DR= nên ta

DR DC DQ DQ DC QC

DB DR DB DB DR QB

= = = =

Các tam giác RDC BDR có BDR góc chung RBC=DRC nên đồng dạng với

E D

R

H K

Q P

N

M

C B

(40)

Từ ta RC DC QC

BR = DR = QB Điều chứng tỏ RM đường phân giác góc CRB Chứng minh hoàn toàn tương tự ta RN phân giác góc ARB Gọi AM, CN phân giác góc BAC; ACB, giao AM CN điểm K Xét tam giác BQR MCR có RBQ RBC RMC= = BRM=MRC

nên hai tam giác đồng dạng với nhau, suy BQ BR

MC= MR Mặt khác ta lại có

( )

1

KCM KCB BCM ACB BAC

2

= + = + CKM KAC KCA 1(BAC ACB)

2

= + = +

Do suy KCM CKM= nên tam giác KCM cân M Từ ta MK MC= ,

mà ta lại có BP BQ= nên kết hợp với kết ta BP BR

MK=MR Mặt khác ta lại có PBR=KMR nên hai tam giác BPR MKR đồng dạng với Từ dẫn đến PRB NRB KRM ERM= = = , nên suy BN=AN EM= Ta có BN BE EM BE+ = + hay EC EA= Điều chứng tỏ điểm E nằm cung ABC Do ta

ARK=CRK Bài toán chứng minh

Bài 30.Giả sử M N điểm nằm tam giác ABC cho MAB NAC=

MBA=NBC Chứng minh AM.AN BM.BN CM.CN

AB.AC + BA.BC + CA.CB =

Lời giải

Trên tia BN lấy điểm K cho BCK=BMA Khi ta ABM=KBC Xét hai tam giác BMA BCK có BCK=BMA ABM=KBC nên BMA ∽ BCK, ta

được AB BM AM

BK = BC = KC hay

AB BK

BM = BC Mặt khác ta lại có

ABK=ABM MBK; MBC MBK KBC+ = +

Nên ta ABK=MBC Hai tam giác ABK MBC có AB BK

BM = BC ABK=MBC

nên đồng dạng với

K

N M

C B

(41)

Từ suy AB BK AK

MB= BC =CM

Kết hợp với AB BM AM

BK = BC = KC ta thu

AM.BC AB.CM AB.BC

CK ; AK ; BK

BM MB BC

= = =

Ta lại có MAB NAC= MAB=NKC nên ta NAC=NKC, từ suy tứ giác AKCN nội tiếp đường trịn

Áp dụng định lí Ptoleme ta AC.NK AK.NC AN.CK= +

Hay ta AC BK BN( − )=AK.NC AN.CK+

Nên suy AC AB.BC BN AB.CM.CN AM.BC.AN

BM BM BM

 

− = +

 

 

Từ suy AB.BC.AC BN.BM.AC AB.CM.CN BC.AN.AN− = + Chia hai vế cho

AB.BC.CA chuyển vế ta thu AM.AN BM.BN CM.CN AB.AC + BA.BC + CA.CB =

Bài toán chứng minh xong

Bài 31. Cho tứ giác ABCD có

ABC CDA 90= = Gọi H hình chiếu A BD

Gọi S T điểm nằm AB, AD cho H nằm tam giác SCT

0

CHS CSB 90 ; THC DTC 90− = − = Chứng minh đường thẳng BD tiếp xúc với đường tròn ngoại tiếp tam giác TSH

Lời giải Gọi E F điểm đối xứng với

C qua B D Ta có SE SC= tam giác SCE cân S, suy ta

SEB SCB= Mà CHS CSB 90− = nên ta

được 0

CHS CSB 90= + =180 −SCB suy

tứ giác CHSE nội tiếp đường tròn Chứng minh tương tự ta tứ giác

CHTF nội tiếpđường tròn P

M

H T

S

F E

D

C B

A

(42)

ESC EHC= mà hai tam giác SEC HEP cân S H nên hai tam giác SEC

và HEP đồng dạng với nhau, ta SEH=CEP Từ ta lại có hai tam giác

SHE CEP đồng dạng với Do SHE SCE HPE HEP= = = nên SH song song

với EP Mà tứ giác CHTF nội tếp đường trịn nên ta có FTC CHF= suy hai tam giác

TCF HPE đồng dạng với Do SHE SCE= nên HS phân giác ngồi góc

CHE Tương tự ta có HT phân giác ngồi góc CHF Từ SHT=SCB TCD+ nên EPF SHT= Lại có HS HS HF CP FP FP

HT =HE HT= EP CP =EP nên hai tam giác SHT FPE

đồng dạng với nhau, suy STH FEP= Kết hợp với BD//EF SH//EP nên ta STH SHB= Vậy BD tiếp tuyến đường tròn ngoại tiếp tam giác HST hay đường thẳng BD tiếp xúc với đường ngoại tiếp tam giác HST

Bài 32. Cho tam giác ABC có trực tâm H Đường phân giác ngồi góc BHC cắt cạnh AB, AC D E Đường phân giác góc BAC cắt đường trịn ngoại tiếp tam giác ADE K Chứng minh KH qua trung điểm đoạn BC

Lời giải Trước hết ta chứng minh tam giác ADE cân A

Thật vậy, HD phân giác ngồi góc BHC nên ta có

( ) ( ) ( )

1 1

DHB HBC HCB 90 ABC 90 ACB BAC

2 2

 

= + =  − + − =

 

Do ta ADE DBH DHB 900 BAC 1BAC 900 1BAC

2

= + = − + = −

Tương tự ta có AED 900 1BAC

2

= − , suy

ADE AED= hay tam giác ADE cân A Mặt khác ta có AK phân giác góc DAE nên đường trung trực đoạn DE, từ suy AK đường kính đường trịn ngoại tiếp tam giác

ADE

K H

Q P

C' B'

D

E

C B

A

(43)

Từ ta có KD vng góc với AB Chứng minh tương tự ta KE vng góc với

AC Gọi P giao điểm KD vàHB, Q giao điểm KE HC Ta có KP HQ

cùng vng góc với AB nên KP song song với HQ Tương tự ta KQ song

song với HP, đo tứ giác KPKQ hình bình hành.Gọi BB’ CC’ đường cao tam giác ABC, ta có DPsong song với HC’ QE song song với HB’ Do theo

định lí Thales ta có PB DB ; QC EC

PH = DC' QH= EB' Theo tính chất đường phân giác tam giác ta có DB HB ; EC DB

DC'=HC' EB' =HB' Vì B, C, B’, C’ thuộc đường trịn đường kính BC nên hai tam giác BHC’ CHB’ đồng dạng với nhau, từ ta HB HC

HC' =HB' Kết hợp với kết ta PB QC

PH =QH, nên theo định Thales ta PQ

song song với BC Do HK qua trung điểm PQ nên HK qua trung điểm BC

Bài 33 Cho tam giác ABC có AB AC 3BC+ = Đường tròn ( )I nội tiếp tam giác ABC tiếp xúc với BC D Gọi T điểm đối xứng với D qua I Các đường thẳng BT CT cắt cạnh AC, AB M, N Chứng minh tâm đường tròn ngoại tiếp tam giác TMN nằm AI

Lời giải Gọi X trung điểm AI Ta chứng minh X tâm đường tròn ngoại tiếp tam giác TMN Gọi E, F tiếp điểm đường tròn ( )I với AC AB Gọi đường tròn ( )O ngoại tiếp tam giác ABC Gọi giao điểm AI với đường tròn ( )O J Gọi giao điểm đường tròn ( )I với đường tròn ngoại tiếp tam giác IBC K, L cho K nằm L C Do

các tứ giác BCIL TLDE nội tiếp đường tròn kết hợp với tính chất hai tiếp tuyến cắt ta có BLI 180= 0−ICB TLE TDE ICB= =

N

M A

B D C

I

J E F

K L

(44)

AB Ta có

MTN=BTC EDF 90= = −BAC Đặt AB c= p nửa chu vi tam giác ABC Ta lại có AM CE p AB= = − Ta có

( ) AC BC AB

NF AB 2BF AB AB BC CA AC BC p c

2 + −

= − = − + − = − = = −

Nên ta AM NF= , từ suy AXM= FXN FNX=AMX

Từ suy tứ gác AMXN nội tiếp Lại có AX phân giác góc MAN nên MAX=NAX hay AM=XN Do ta XM XN= nên X nằm đường trung

trực MN.Lại có

NXM 180= −BAC 2MTN= nên X tâm đường tròn ngoại tiếp

tam giác MNT

Bài 34.Cho tam giác ABC nội tiếp đường trịn ( )O có AB BC 2BC+ = Đường tròn ( )I

nội tiếp tam giác ABC tiếp xúc với cạnh BC, CA, AB D, E, F Gọi

b c

M , M trung tuyến CA AB Giao điểm M Mb c EF L

Chứng minh đường tròn (L; LI) tiếp xúc với đường tròn ( )I đường trịn đường kính AO

Lời giải

Gọi Ma trung điểm BC Gọi giao điểm

AI với đường tròn ( )I K Do đường tròn ( )I nội tiếp tam giác ABC nên ta suy AB, BC, CA tiếp tuyếncủa đường trịn ( )I Theo tính chất tiếp tuyến cắt kết hợp giả thiết ta

AB AC BC 2BC BC BC

AE AF

2 2

+ − −

= = = =

K D

L I O F

E Mc Mb

Ma C B

A

Từ suy AF=BMa Do AK phân giác góc BAC nên K nằm cung BC khơng chứa A đường trịn ( )O

(45)

Nên FAI=KBMavà KMa ⊥BC Từ dẫn đến AFI= BM Ka nên AI=BK

Ta lại có IBK IBC CBK 1(BAC ABC , BKI ACB)

= + = + = , ta

( ) ( )

( ) ( )

0

BIK 180 IBK BKI 180 BAC ABC ACB

2

1

BAC ABC BAC ABC BAC ABC IBK

2

= − + = − + −

= + − + = + =

Do ta tam giác BKI cân K, suy raBK=IK Khi ta AI=BK=DI,

suy I trung điểm AK Do ta AK vng góc với OI Do O tâm

đường tròn ngoại tiếp tam giác ABC nên suy OM , OM , OMa b c đường

trung trực BC, CA, AB Nên ta

c b

AM O=AM O 90= Từ suy M , Mb c

thuộc đường trịn đường kính AO Lại có

AIO 90= nên I thuộc đường tròn đường

kinh AO Do IE⊥AC, IF⊥AB nên E, F nằm đường thẳng Simson I

tam giác AM Mb c Từ suy IL⊥M Mb c nên ba điểm I, L, D thẳng hàng Điều dẫn đến ba điểm A, L, Ma thẳng hàng nên từ ta suy L trung điểm M Mb c

Gọi T tâm đường trịnđường kính OA, TI qua trung điểm L Từ hai đường trịn (L; LI) ( )T tiếp xúc với nhau.Từ IA=IK M Ka =ID ta

DI LI

2

= nên suy (L; LI) tiếp xúc với đường tròn ( )I

Bài 35.Cho tam giác nhọn ABC không cân nội tiếp đường trịn ( )O có đường cao AD, BE, CF cắt H Đường thẳng EF cắt BC P Qua D vẽ đường thẳng song song với EF cắt AC Q cắt AB M Chứng minh đường tròn ngoại tiếp tam giác PQM di qua trung điểm BC

(46)

Vẽ đường kính AA’ đường trịn ( )O

Gọi giao điểm A’H với BC S cắt đường tròn ( )O điểm thứ hai K Khi dễ thấy HB song song với A’C CF song song với A’B Từ suy tứ giác BHCA’ hình bình hành, nên S trung điểm BC

O

S A'

K

M

P Q

H' H F

E D C B

A

Như để chứng minh đường tròn ngoại tiếp tam giác PQM di qua trungđiểm BC Ta cần chứng minh tứ giác MSQP nội tiếpđường tròn Thật vậy, dễ thấy A' AC=A' KC CBA'= Suy ta có CAH HAA' AKE KEC+ = + =A' BC Dễ thấy tứ giác AKEH nội tiếp đường tròn nên ta HAE HKE= , suy A' AD EKC= mà tứ giác BFEC nội tiếp đường tròn nên ta có ABC FEA= , ABD FEA CEP= = Lại

ABC CBA' 90+ = nên ta

A' AH HAC ABC A' AH HAC FEA 90+ + = + + =

Do ta suy AA’ vng góc với EF Từ ta có

0

ACB=BPF FEA+ 90 −CAH=BPF 90+ −CAH A' AH− BPH=A' AH EKC=

Từ dẫn đến tứ giác EKPC nội tiếp đường tròn, nên ta FEA CEP= =CKP Do

ta có

FEA CAH A' AH FEA HKE EKC 90+ + = + + = nên

HKE EKC CKP+ + =90 Từ ta AKH PKH 180+ = nên ba điểm A, K, P thẳng hàng H trực tâm tam

giác APS Gọi giao điểm thứ hai đường tròn ( )O với AD H’ Đến ta thấy tứ giác ASH’P nội tiếp đường tròn nên suy DS.DP AD.DH' DB.DC= = Lại MQ song song với EF nên MQA FEA ABC= = nên tứ giác BMQC nội tiếp đường trịn.Từ ta DC.DB DQ.DM= Kết hợp hai kế ta DM.DQ DS.DP= nên tứ giác MSDP nội tiếp đường tròn Vậy đường tròn ngoại tiếp tam giác PQM di qua trung điểm BC

(47)

tại K

2

R AH KI.KJ

BC

=

Lời giải

Gọi R ,R1 bán kính đường

trịn nội tiếp tam giác ABH, ACH Đặt AB c, BC a,CA b,AH h= = = =

Gọi giao điểm AI, AJ với BC M, N.Gọi giao điểm BI với

AN E giao điểm CJ với AM

là F

K

D Q

N M

F E V T J I

H C

B

A

Từ giả thiết ta ABF HAF= =BCF ACF= ABE CBE HAE CAE= = =

Khi tam giác ABC đồng dạng với tam giác HBA nên AB R1 BH

BC= R = AB (1) tam

giác ABC đồng dạng với tam giác HAC nên AC R2 CH

BC = R = AC(2)

Từ (1) (2) ta

2 2

2 2

1

1

2

R R AB AC

1 R R R

R BC

+ +

= =  + = (3)

Ta có (R1+R2)2 2 R( 12+R22) nên suy R1+R2  2.R (4)

Cũng từ (1) (2) áp dụng bất đẳng thức Cauchy ta

2

1

1

R R BH CH BH.CH AH h h

2 2 R R 2R

R AB AC AB.AC BC.AH a a

+

= +  = =  +  (5)

Dễ thấy h AD a

 = hay x h

a

 =  nên x 2x( − )  0 x 2x2  x 2.x

Do ta suy h 2.h

a  a, nên ta

2 2.h.R

R R

a

+  (6)

Đẳng thức xẩy R1 =R2 hay tam giác ABC vuông cân A Từ (1) (2) ta có

2

1 2

R.AB R.AC R ah R h

R R

BC BC a a

= = = (7)

Từ tâm Q đường tròn nội tiếp tam giác ABC hà đường vng góc xuống cạnh tam giác ABC ta có hệ thức b c a 2R+ = + Áp dụng hệ thức vào tam giác

ABH ACH ta

(48)

( 2) (

2 R R R AH

Website:

tailieumontoan.com BH AB()AH CH AC()AB  AC BC)2AH

+ + =

Từ ta R R+ 1+R2 =h (8)

Kẻ TJ vng góc với AH T JP//AH, IP//CB cắt JP P cắt AH V Tam giác IJP vng P có 2

IJ =IP +JP hay ( ) (2 )2

1 2

IJ = R +R + R −R  =IJ 2R=AQ (9)

Ta có ( )

ABE BAE+ = ABE HAE+ +BAH=ABC ACB 90+ = BA BN= , AE EN=

Tương tự ta có CF⊥AM CA CM,AF FM= = Do EF đường trung bình

tam giác AMN nên EF//BC EFC FCB= (10) Từ suy điểm I, J, E, F nằm đường trịn đường kính IJ Từ IE⊥AJ, JF⊥AI IE cắt JF Q nên AK đường cao tam giác AIJ hay AQ⊥IJ K Từ ta A, J, K, F, T thuộc đường trịn đường kính AJ A, I, E, K, V thuộc đường trịn đường kính AI Từ ta EAK IAH= EIJ EFJ= , Kết hợp với (10) ta EAK IAH= , suy

IAV JAK

 ∽  nên ta có IV KJ R1 AI

AI= AJKJ =AJ (11)

Hoàn toàn tương tự ta FAK=JAH nên JAT∽ IAK suy R1 AJ

KI= AI (12)

Từ (11) (12) ta

R KI

KJ =R nên KI.KJ=R R1

Kết hợp với (7) ta

2

R AH KI.KJ

BC =

Bài 37 Cho hai đường tròn tâm O O’ cắt A B cho OA vng góc với O’A, OO’ cắt hai đường trịn C, E, D, F cho điểm C, O, E, D, E, O’, F nằm đường thẳng theo thứ tự BE cắt đường trịn (O) điểm thứ hai K cắt CA M BD cắt đường tròn (O’) điểm thứ hai L cắt AF N Chứng minh rằng: KE LN O'E

KM LD = OD

Lời giải

Ta có AOO' 2ACF= AO' O=2AFC Mà

OAO' 90= nên

(49)

Từ suy ACF AFC+ =450 Mà tam

giác O’AL cân nên ta lại có

( )

0 0

1

O' AL 180 AO' L 90 AO' L

2

90 ABL 90 ABD 90 ACF

= − = −

= − = − = −

Mặt khác CAL CAO OAO' O' AL= + +

Do đóta

F K

L

O' O

N M

E D C

B A

0 0

CAL=ACF 90+ +90 −ACF 180=

Từ suy ba điểm C, A, L thẳng hàng Chứng minh tương tự ta K, A, F thẳng

hàng Áp dụng định lí Menelaus cho tam giác CME vớiba điểm K, A, F thẳng hàng ta

có KE AM FC KM AC FE = hay

KE AC.EF

KM =AM.FC Áp dụng định lí Menelaus cho tam giác CME

với ba điểm K, A, F thẳng hàng ta có LD AN CF LN AF CD= hay

LN AN.CF

LD = AF.CD Mà ta lại có

EF 2EO' O'E

CD= 2OD = OD Do

KE LN AC.EF.AN.CF AC.AN.FE AC.AN.O'E

KM LD =AM.FC.AF.CD= AM.AF.CD= AM.AF.OD Do

hai tứ giác AEBF ACBD nội tiếp nên ta EBA EFA; ABD ACD= = Suy

( )

0 0

0

MAN CAF 180 ACF AFC 180 45 135

MBN EBA ABD EFA ACD CAF 45

 = = − + = − =

 

 = + = + = =

Từ ta MAN MBN 180+ = nên tứ giác AMBN nội tiếp đường trịn Từ ta AMN=ABN=ACD=ACF, suy MN song song với CF Theo định lí Thales ta AC AF

AM=AN hay

AC.AN AM.AF=

Do từ KE LN AC.AN.O'E

KM LD = AM.AF.OD ta thu

KE LN O'E

KM LD = OD

Bài 38.Cho tam giác ABC nhọn có đường cao AA’, BB’, CC’ Gọi D, E, F tâm đường trịn bàng tiếp góc B' AC', C' A' B, B' A' C tam giác AB’C’, BC’A’, CA’B’ tương ứng Đường tròn bàng tiếp góc BAC tam giác ABC tiếp xúc với BC, CA, AB M, N, P Chứng minh tâm đường tròn ngoại tiếp tam giác DEF trực tâm tam giác MNP

(50)

Gọi I tâm đường trịn bàng tiếp góc A tam giác ABC Gọi H tiếp điểm đường

tròn ( )E với đường thẳng BC Nhận thấy ba điểm E, B, I thẳng hàng ba điểm I, C, F thẳng hàng Dễ thấy tứ giác AC’A’C nội tiếp đường trịn,do ta có

1

EA'H BA'C' BAC IAP

2

= = = I

P

N M

E

F D

H

O

C' B'

A' C B

A

Xét hai tam giác vng EA’H IPA có EA'H IAP= nên suy hai tam giác EA’H IAP đồng dạng với nhau, ta HA' EH

PA = IP Mà ta lại có IM IP, MB PB= = HE song song với IM nên ta có EH HB

IM = BM

Từ suy raHA' EH EH HB HB

PA = IP = IM = BM= PB suy

HB PA

HA'= PB Do ta HP song

song với AA’ Nên theo tiên đề Ơclit ba điểm E, H, P thẳng hàng, IM song

song với EP Từ ta PEI PIB EHM= = nên ta có tam giác EPI cân P, suy

EP PI IM= = Tứ giác EPIM có EP song song với IM lại có EP PI IM= = nên hình thoi Hồn tồn tương tự ta PIND, FNIIM hình thoi Từ ta EM, IP, DN song song với EM PI= =DN; đồng thời ta có FM, NI, DP

song song với FM NI DP= = Suy tứ giác EMND, FMPD hình bình hành nên ta PMsong song với FD MN song song với DE Gọi O tâm đường tròn ngoại tiếp tam giác DEF, kho ta có OD OE OF= = Kết hợp với

PE PI= =PD nên ta PO đường trung trực DE hay vng góc với DE Mà ta có DE song song với MN nên ta có PO vng góc với MN Chứng minh tương tự ta đượcNO vng góc với MP Vậy O trực tâm tam giác MNP

Bài 39.Cho tam giác ABC vuông A Gọi I giao điểm ba đường phân giác tam giác Gọi D, E, F hình chiếu I cạnh BC, AC, AB Gọi M trung điểm cạnh AC Đường thẳng MI cắt AB N, đường thẳng DF cắt đường cao AH P Chứng minh tam giác ANP tam giác cân

(51)

Ta có IE AB vng góc với AC nên IE song song với AB Theo định lí

Thales ta có AN AM

EI = EM Từ ta suy

ra

( )

AM.EI AC.EI

AN

EM AM AE

= =

Vì BAC 90= nên tứ giác AEIF hình vng, suy AE=EI

K

H P

N I

M F

E

D C

B

A

Vì D, E, F hình chiếu I cạnh BC, AC, AB nên ta có

( )

1

AE CD BD AB BC CA

2

+ + = + + nên IE AE AB CA BC

2 + − = =

Lại có BC AB CD AF CE AE− = − = − =2 AM AE( − ) Kết hợp kết lại ta

( ) ( ( ) ) ( )

2

AC AB CA BC

AC.EI BC AB AB.AC AC.BC BC AB AC

AN

2

2 AM AE BC AB BC AB

+ − − + − + −

= = = =

− − −

Từ A kẻ đường thẳng song song với BC cắt DF K, tam giác APK vng

A Ta có BD=BFdo ta có AK=AF AE= AKF BDF 900 1B BIF

= = − = Trong

tam giác AKP có AP AK.tan AKF AK.tan 900 1B AK.cotB AE.cotB

2 2

 

= =  − = =

  Do

đó AP AC AB BC BD

2 ID

+ −

= Mà BD BC AB AC

2 + −

= DI AE AC AB BC

2 + −

= = Từ

đó suy AP AC AB BC BC AB AC AB BC AC

2 AB AC BC

+ − + − + −

= =

+ −

Như từ kết ta AN AP= nên tam giác ANP cân A

Bài 40. Cho tam giác nhọn ABC, lấy điểm X nằm tam giác điểm Y, Z nằm tam giác cho tam giác XBC, YCA, ZBA đồng dạng với Đường thẳng YZ cắt AB, AC M N Gọi P trung điểm BC Chứng minh YN ZM= PAB=XAC

Lời giảiTrước hết ta phát biểu chứng minh bổ đề Nếu P trung điểm cạnh BC

điểm S nằm tam giác ABC cho PAB SAC= ta ln có

2 SAB

2 SAC

S AB

(52)

S

P C

B

A

S V U

H

K N M

P Z

Y

X

C B

A

Chứng minh. Do PAB SAC= nên BAS CAP= Khi ta có

SAB PAC

S AB.AS.sinBAS AB.AS

S = AC.AP.sin CAP= AC.APvà

PAB SAC

S AB.AP.sin BAP AB.AP

S =AC.AS.sin SAC= AC.AS

Do AP đường trung tuyến nên ta SPAB=SPAC

Do ta

2

SAB PAB SAB

2

PAC SAC SAC

S S AB.AS AB.AP S AB

S S = AC.AP AC.AS S = AC

Trở lại toán Gọi S giao điểm AX YZ Gọi K H hình chiếu Y Z AB AC Ta chứng minh tứ giác AZXY hình bình hành

Thật vậy, theo giả thiết ta có hai tam giác XBC YCA đồng dạng với nên ta

được BCX=ACY, suy BCA=XCY Và ta có XC BC

YC =AC nên hai tam giác ABC

và YXC đồng dạng với Từ ta BAC CYX= ABC=YXC Chứng

minh hoàn toàn tương tự ta hai tam giác ABC ZBX đồng dạng với nên ta BZX=BAC BXZ=ACB Từ ta XYC XZB=

Mà ta có AZB AYC= nên ta AZX AYX= Mặt khác ZAY=ZAB BAC YAC+ + Ta có biến đổi góc

( )

0 0

0

ZXY 360 ZXB BXC CXY 360 ACB ABC 180 XBC XCB

180 ACB ABC XBC XCB BAC XBC XCB

= − − − = − − − − −

= − − + + = + +

Để ý BAZ=BCX; YAC CBX= nên ta ZAY ZXY= Như ta ta tứ giác AZXY hình bình hành

+ Điều kiện cần Nếu có YN ZM= , kết hợp với giả thiết ABZ∽ CAYta

AYN AZM

S

AC YK YK.AN AM AM YN AM AM

(53)

hình bình hành nên ta có SY SZ= , mà YN ZM= nên ta SN SM= Kết hợp với hai tam giác ABC ANM đồng dạng với ta hai tam giác ABP ANS

đồng dạng với Do ta PAB SAN XAC= =

+ Điều kiện đủ Nếu có PAB=XAC, với U V hình chiếu S AB AC, kết hợp với giả thiết ABZ CAY đồng dạng với nhauthì theo bổ đề

trên ta SAB

SAC

S SU.AB

SV.AC=S hay ta

2 SAB

2 SAC

S

SU AC AB AC AB ZH

SV =S AB = AC AB =AC = YK Do

đó ta SU SV

ZH =YK Do SU song song với ZH SV song song với YK nên theo

định lí thales ta có SM SU SV SN ZM= ZH= YK= YN

Kết hợp với SY SZ= ta ZM SM ZM SM SZ

YN SN YN SN SY

+

= = = =

+

Do ta YN ZM= Bài tốn chứng minh

Bài 41. Cho tam giác ABC không vng có đường cao BE CF cắt H Gọi M, N, P, Q, S theo thứ tự trung điểm BF, CE, BE, CF, EF Đường thẳng qua M vng góc với BS đường thẳng qua N vng góc với CS cắt K Đường thẳng qua P vng góc với BS đường thẳng qua Q vng góc với CS cắt L Chứng minh 2KL=AH

Lời giải Do M, N, P, Q, S theo thứ tự trung điểm BF, CE, BE, CF, EF nên ta

1 1

MB BE, MS BE, NS CF, CN CE

2 2

= = = =

Trong tam giác vuông BCE BCF có

2 2 2

BC =BE +CE =BF +CF

Gọi I giao điểm BS MK, ta có Km

vng góc với BS KN vng góc với CS nên

ta KB2 =KI2+SI ; KS2 2=KI2+BI2

Suy KB2−KS2 =BI2−SI2

F

R L

K E S

Q

P N

M

C

D B

A

H

(54)

Chứng minh tương tự ta MB2−MS2=BI2−SI2 vàNC2−NS2=KC2−KS2

Suy 2 2 2 2 2 2

KB KS MB MS BF BE CE CF NC NS KC KS

4 4

− = − = − = − = − = −

Từ ta KC KB= Gọi H trực tâm tam giác ABC, BF CE đường cao tam giác HBC.Tương tự ta chứng minh LB LC= Từ ta suy LK đường trung trực đoạn thẳng BC nên LK vng góc với BC

Dễ thấy LKN=BCS Lại thấy QLK SCB 180+ = suy QLK LKN 180+ = Do ta

được QLsong song với KN Lấy điểm D R cho tứ giác AHDF QRKL hình bình hành, suy R thuộc KN Ta có HE⊥NC HD//AF, CF//NS, AF⊥CF suy HD⊥NS Từ ta EHD CNS= Dễ thấy hai tam giác CHE CAF đồng

dạng nên ta có HE HE EC 2NC NC

HD = AF = HC= 2NS = NS Từ suy tam giác EHD CNS

đồng dạng với nên ta HED NCS= Ta có DE RN vng góc với

SN nên DE song song với KR, mà CF song song với RQ FE song song với QN nên

ta hai tam giác DEF RNQ đồng dạng với Từ KL RQ QN

HA = DF = FE =

nên ta 2KL=AH

Bài 42.Cho tam giác ABC có D chân đường cao hạ từ đỉnh C Trên cạnh AB lấy điểm E, F cho ACE=BCF 90= Lấy điểm X đoạn CD điểm K đoạn FX cho BK BC= Lấy điểm L đoạn EX cho AL AC= Gọi giao điểm AL BK M Chứng minh ML MK=

Lời giải + Trường hợp 1 Xét tam giác ABC vng C Khi ta hai điểm A F trùng nhau, hai điểm B E trùng

Vẽ AI vng góc với BX, đường thẳng AI cắt CD N Tam giác NAB cao ND BI đường cao nên X trực tâm, ta à vng

góc với BNtại S.Tam giác ACB vng C có

CD đường cao nên ta có AC2=AD.AB

D L K

S I

N

C X

B A

Theo ta có AL AC= nên ta AL2 =AD.AB Ta lại có AD AI

(55)

AIL900

Tam giác ALN vng L có LI đường cao nên ta LN2=NI.NA Tương tự ta có NK2=NS.NB Mà ta có NI NS

NB= NA nên ta

được NI.NA NS.NB= Do ta NK NL= Từ suy hai tam giác vng KMN LMN Từ ta suy ML MK=

+ Trường hợp 2. Xét tam giác ABC có góc

0

ACB 90 Kẻ đường thẳng qua A vng góc với XE cắt CD N Khi X trực

tâm tam giác NAE Suy à vng góc

với Từ ta AEX=AND

Ta có AL2=AC2=AD.AE nên AL tiếp xúc

với đường tròn ngoại tiếp tam giác EDL F D E

K

L X

M N

C

B A

Suy ALD LED AEX= = Từ ta AND ALD= nên bốn điểm A, N, L, D

nằm đường trịn Do

NLA=NDA 90= hay NL vng góc với AL

Lại có hai tam giác XED AND đồng dạng với nên ta XD ED

AD= ND Hai

tam giác ACE BCF vng góc C nên ta CD2 =AD.DE BD.FD= hay

AD FD

BD =ED Do ta

XD BD

FD =ND, nên hai tam giác BDN XDF đồng dạng với nhau, suy BND=DFX=BFK Mặt khác ta có BK2=BC2=BD.BF nên BK tiếp xúc với đường trịn ngoại tiếp tam giác DKF nên BKD BFK= Từ ta BND=BKD

nên bốn điểm B, D, K, N nằm đường trịn Do ta NK vng

góc với BK, áp dụng định lí Pitago ta

( ) ( )

( ) ( ) ( ) ( )

2 2 2 2

2 2 2 2

MK ML NL NK NA AL NB BK

NA NB CB CA CA CB CA CB

− = − = − − −

= − + − = − + − =

Từ ta MK2=ML2 hay MK ML=

+ Trường hợp 3. Xét tam giác ABC có

ACB 90 Chứng minh hoàn toàn tương tự ta ML MK=

(56)

Bài 43.Cho tam giác ABC có M trung điểm AC D điểm cạnh BC

cho DB DM= Biết 2BC2−AC2 =AB.AC Chứng minh

( )

2

AC AB BD.CD

2 AB AC =

+

Lời giải Lời giải 1.Một ý tưởng tự nhiên tìm cách biểu diễn BD CD theo cạnh tam giác ABC kết hợp với giả thiết cho Quan sát hình vẽ ta thấy biểu diễn BD theo BM

cosCBM I D M C B A

Để ý BM đường trung tuyến tam giác ABC nên ta biểu diễn BM

cosCBM theo cạnh tam giác ABC Từ kết hợp với 2BC2−AC2=AB.AC ta

có thể tính BD.CD theo cạnh tam giác ABC.Để đơn giản qua trình

biến đổi ta đặt AB c; BC a; CA b.= = = Để tính cosCBM ta cần tạo tam giác vuông cách vẽ DI vng góc với BM với I thuộc BM Khi ta DI đường trung tuyến tam giác cân BDM Do ta BD BI BM

cosCBM cosCBM

= =

Mặt khác theo định lí cosin ta có

2 2

2 2 BM a b

BM BC CM 4

2 cos CBM

BC.BM a.BM

+ −

+ −

= =

Từ ta

2 2 a.BM BD b BM a = + −

Mặt khác ta có ( )

2

2 2

2 2

b

a BM a

4 BD.CD BD a BD

b BM a   −     = − =   + −    

Lại có ( )

2

2 2 b

BM a c

2

= + − kết hợp với giả thiết 2a2−b2 =bc ta được

( ) (2 )2

2 2

2 2 2

2 b a c b 2c 2bc b bc b c 2c b

BM a a

4 16

+ +

 + −  = + − +  = + + +  =

     

     

Ta có ( ) ( )

2

2 2 2

2 2

2 2 b a 2a 2c b b bc b a bc 2c bc

a BM a

4 4 16

+ − +

   + 

− = − =

   

(57)

Từ ta ( )

( ) ( ) ( ) ( )

2

2 2

2

a bc 2c bc b c AC AB

BD.CD

2 b c AB AC

b c 2c b

+

= = =

+ +

+ +

Lời giải 2.Một ý tưởng khác tự nhiên ta tìm cách biến đổi giả thiết 2BC2−AC2 =AB.AC

cho xác định yếu tố phụ cánh hợp lí Chẳng hạn ta viết lại giả thiết toán

thành ( )

2BC =AC AB AC+ ta ý đến xác định điểm P AC cho PA=AB, ta CP AC AC= +

P

M D C B

A

Từ ta 2BC2 =AC.CP 2CM.CP= hay BC2=CP.CM Hệ thức làm ta tiên

tưởng đến tính chất tiếp tuyến cát tuyến đường trịn.Tức BC tiếp tuyến đường tròn ngoại tiếp tam giác BMP hay BD tiếp tuyến đường tròn ngoại tiếp tam giác BMP Mặt khác ta lại thấy BD DM= nên DM tiếp tuyến đường trịn ngoại tiếp tam giác BMP Khi dễ dàng chứng minh MDC=BAM

nên tứ giác ABDM nội tiếp, từ dẫn đến AD phân giác BAC Đến ta biểu diễn BD, CD qua cạnh tam giác ABC

Trên tia đối tia AC lấy điểm P cho PA=AB, từ giả thiết ta

2

2BC =AC.CP 2CM.CP= BC =CP.CM

Từ suy BC tiếp tuyếncủa đường trịn ngoại tiếp tam giác PBM với tiếp điểm

là B Ta lại có DB DM= với D thuộc BC nên DM tiếp tuyến đường tròn ngoại tiếp tam giác PBM

Từ ABP=MPB=DBM=DMB Lại có MDC=DBM BMD+ BAM=APB ABP+ Suy MDC=BAM nên tứ giác MDBA nội tiếp, DBM=DMB CAD= =BAD

Do ta AD phân giác góc BAC Áp dụng tính chất đường phân giác

trong tam giác ta BD AB BD AB BD AB.BC

CD= AC BC= AB AC+  =AB AC+

Từ suy DC AC.BC AB AC =

+ , kết hợp với giả thiết ta

( ) ( )

2

2

AB.AC.BC AC AB

BD.CD

2 AB AC AB AC

= =

(58)

Bài 43 Cho tam giác ABC điểm D, E, F tiếp điểm đường tròn bàng tiếp góc A, B, C với cạnh BC, CA, AB tam giác ABC Chứng minh tâm đường tròn ngoại tiếp tam giác DEF nằm đường tròn ngoại tiếp tam

giác ABC tam giác ABC vuông

Lời giải

Lời giải Trước hết ta phát biểu chứng minh toán phụ

Cho D điểm nằm cạnhAC tam giác ABC ta ln có

2 2

AB CD BC AD BD AC AC.CD.AD+ − =

Chứng minh.Gọi H chân đường cao hạ từ B xuống AC, giả sử D thuộc đoạn AH

Trong tam giác ABD có

( )2

2 2 2 2

BC =HC +HB = BD DH− +BD −AD =BD +CD −2DC.DH

Tương tự tam giác ABD ta có AB2=BD2+AD2+2AD.DH

Từ ta AB CD BC AD2 + =BD AD DC2( + )+DC AD AD DC2 + Suy AB CD BC AD BD AC AC.CD.AD2 + − =

Trở lại toán.Đặt AB c; BC a; CA b; p a b c + +

= = = =

Gọi S tâm đường tròn ngoại tiếp tam giác DEF Theo giả thiết S nằm đường trịn ngoại tiếp tam giác ABC Khơng tính tổng quát ta giả sử S nằm cung BC có chứa điểm A Khi ta có SE SF= Theo

tính chất đường trịn bàng tiếp tam

giác ta có CE BF p a= = −

Vì ABS=ACS nên ta SEC= SFB, suy SB SC= hay S nằm cung

BC Áp dụng đổ đề cho tam giác ABC với điểm D cạnh BC ta có

( )

2 2

SC DB AB DC BC SD+ = +DB.DC

S

O N M

K

J

I

F E

B D C

A

(59)

Để ý DB DC BC+ = nên ta SB2−SD2=DB.DC

Mà ta có BD p b; CD p c= − = − , ta ( )( ) ( )

2

2 a b c

SB SD p b p c

4 − −

− = − − =

Gọi N M hình chiếu vng góc S AC AB

Khi ta SBM= SCN suy AM AN= nên ta BM CN b c + = =

Mặt khác ta có

2

2 2 b c b c b c a

SB SF MB MC

2 2

 +   + + − 

− = − =  − − 

   

Do SD SF= nên ta có SB2−SD2 =SB2−SF2 Do ta

( )2 2 2

2

2 2

a b c b c b c b c a

a b c

4 2

− −  +   + + − 

=  − −   = +

   

Do tam giác ABC vuông A Từ ta có điều phải chứng minh Lời giải 2.Trước hết ta phát biểu chứng minh bổ đề sau

Bổ đề 1.Cho tam giác ABC nội tiếp đường trịn ( )O có đường cao AD, BE, CF Khi đó, đường thẳng qua A vng góc với EF, đường thẳng qua B vng góc với FD, qua C vng góc với DE đồng quy O.

Chứng minh. Gọi H trực tâm tam ABC I, J

theo thức tự giao BE, CF với ( )O Dễ thấy E, F theo thứ tự trung điểm HI, HJ nên EF

song song với IJ

J

H

I O

F

E C D

B

A

Lại có ABI=ACJ suy AI=AJ nên AO vng góc với EF Tương tự OB vng góc với DF OC vng góc với DE Từ ta có điều phải chứng minh

Bổ đề 2. Cho tam giác ABC nội tiếp đường tròn ( )O và M trung điểm BC Khi ta có BC

OM

= khi

BAC=45

Bổ đề 3. Cho tam giác ABC có J tâm đường trịn bàng tiếp góc A Khi ta ln có

0 BAC

(60)

Trở lại tốn.Khơng tính tổng qt

ta giả sử BAC góc lớn tam

giác ABC

+ Trường hợp AB AC= toán hiển nhiên

+ Xét trường hợp AB AC

Đặt BAC=A Gọi S điểm

cung BAC đường trịn ngoại tiếp ta

giác ABC Dễ thấy SBF= SCE nên ta có SE SF= Do suy BSC ESF= =A Gọi I, J, K theo thứ tự tâm đường trịn bàng tiếp góc A, B, C tam giác ABC

A

D C

B

E F

I

J

K

M O

S

Dễ thấy IA, JB, KC đường cao tam giác MNP S trung điểm JK Theo toán phụ ta có ID, JE, KF đồng quy điểm M tâm đường tròn ngoại tiếp tam giác IJK.Chú ý tứ giác BFDM, CEDM, AEMF nội tiếp đường tròn

nên BDF=BMF CDE CME= Lại có EMF 180= 0−A Khi ta có biến đổi góc

( ) ( ) 0

BMC EDF+ = BMF FME EMC+ + + 180 −BDF CDE− =EMF 180+ =360 −A Từ ta BMC EDF1 1800 A

2

 

+ =  − 

 

Tâm đường tròn ngoại tiếp tam giác DEF nằm đường tròn ngoại tiếp tam giác ABC S tâm đường tròn ngoại tiếp tam giác DEF, điều xẩy

( ) A A ( )

ESF 180 EDF EDF 180 BMC 180 BSC A 180 BMC

2

= −  = −  = −  = = −

Hay S tâm ngoại tiếp tam giác BMC Do ta

0 0

JK A

SM SE SC SM KIJ 45 90 45 A 90

2

= =  =  =  − =  =

Do ta có điều phải chứng minh

(61)

khi

AB AC 3BC+ =

Lời giải

Gọi D tiếp điểm đường tròn ( )I với BC Gọi ( )O đường tròn ngoại tiếp tam giác ABC Gọi J giao điểm khác AI với đường

tròn ( )O Trước hết ta chứng minh I tâm đường tròn nội tiếp tam giác DKL

Thật vậy, I tâm đường tròn nội tiếp tam

giác ABC nên ta có BIC 900 1BAC

2

= +

T

X Y L

K Q

P

J I

D C

B A

Mà AP AQ hai tiếp tuyến đường tròn ( )I nên tam giác APQ cân A Từ ta có BAP 900 1BAC

2

= + Do suy BQL BIC= nên tứ giác BQLI nội tiếp, nên ta LQI LBI= Mà ta có LQB LIB 180+ = 0LQI LIB 90+ = nên ta

0

LBI LIB 90+ = Từ suy BLI=900 hay BL vng góc với LC Chứng minh tương tự ta BK vng góc với KC

Gọi T giao điểm BL CK, I trực tâm tam giác TBC Do ta D, I, T thẳng hàng.Dễ thấy tứ giác BLKC, MLID CKID nội tiếp đường trịn nên ta có KBL KCL; LBI LDI; KCI KDI= = = Từ suy LDI KDI= nên DI phân giác

của góc LDK Hồn tồn tương tự ta LI phân giác góc KLD Do

I tâm đường tròn nội tiếp tam giác DKL.Đường tròn ngoại tiếp tam giác IKL tiếp xúc với đường tròn ( )I R( )IKL r

2

= với r bán kính đường tròn ( )I

( )IKL

R bán kính đường trịn ngoại tiếp tam giác ILK Kẻ IX vng góc với LD X, IY vng góc với PQ Y Dễ dàng chứng minh hai tam giác ILK IBC đồng dạng với nên ta ( )

( )

ILK IBC

R IY IX A

sin IDX sin

R =ID =ID = =

(62)

Hay R( )ILK IP r

IJ =IA=AI Từ suy ( )IKL r R

2

= IA=2IJJA=3JB 3JC=

Do tứ giác ABJC nội tiếp dường tròn (O) nên áp dụng định lí Ptoleme ta

( )

AI.BC=AB.JC AC.JB+ = AB AC JB+

Kết hợp với JA=3JB 3JC= ta AB AC 3BC+ = Vậy tốn chứng minh

Bài 45. Cho đường trịn (O; R) có đường kính AB điểm M nằm

( )O khơng nằm đường kính AB Gọ N giao điểm đường phân giác góc AMB với đường trịn ( )O Đường phân giác ngồi góc AMB cắt đường thẳng NA, NB P Q Đường thẳng MA cắt đường trịn đường kính NQ R, đường thẳng MB cắt đường trịn đường kính NP S R, S khác M Chứng minh đường trung tuyến ứng với đỉnh N tam giác NRS ln qua điểm có định M di động phía đường trịn

Lời giải

O I

S

R

Q

P

N M

D C

B A

Qua R kẻ đường thẳng song song với PQ cắt AN C, qua S kẻ đường thẳng song song với PQ cắt BN D Gọi I trung điểm CD Ta chứng minh CD song song với AB

Thật vậy, N nằm đường trịn đường kính AB nên ta có

ANB 90= suy AN

vng góc với BN, BN tiếp tuyến đường trịn đường kính PN Từ ta

có tam giác BMN đồng dạng với BNS Vì PQ đường phân giác ngồi tam giác

AMN nên ta có SMP AMP QMR BMQ= = = Mặt khác SMP SNP= QMR QNR=

(63)

Xét hai tam giác BNS RNC có CNR=SNB RCN MPN= =NSM=NSB nên hai

tam giác đồng dạng với Từ suy BNS∽ RNC∽ BMN Lập luận chứng tương tự ta có DSN∽ RAN∽ NAM

Ta thấy BNS RNC NB NS NB.NC NR.NS

NR NC

 ∽   =  =

NS ND

DSN RAN NA.ND NR.NS

NA NR

 ∽   =  =

Từ ta NB.NC NA.ND= hay NA NC

NB =ND, theo định lí Thales đao ta AB

song song với CD

Do trung điểm AB, trung điểm CD N thẳng hàng Tức N, O, I thẳng

hàng Lại có BMN RNC MN BN RC NB.NC

NC RC MN

 ∽   =  =

DSN NAM DN DS SD NA.ND

MN NA MN

 ∽   =  =

Kết hợp điều ta RC SD= Mà ta có RC song song với SD nên tứ giác

RCSD hình bình hình Do hai đường chéo CD SR cắt trung điểm đường, suy I trung điểm CD trung điểm SR Khi NI đường trung tuyến tam giác NSR Do ta đường trung tuyến NI qua điểm O cố định Vậy đường trung tuyến xuất phát từ N tam giác NRS qua điểm O cố định điểm M di động đường tròn ( )O

Bài 46. Cho tam giác nhọn ABC cố định khơng cân nội tiếp đường trịn ( )O , đường

phân giác AD Lấy điểm P di động đoạn thẳng AD điểm Q đoạn thẳng

AD cho PBC QBA= Gọi R hình chiếu Q BC Đường thẳng d qua R vng góc với OP Chứng minh đường thẳng d qua điểm cố định P di chuyển

(64)

Lời giải 1.Gọi giao điểm thứ AD với đường

tròn ( )O E nên E điểm cung BC Vẽ đường kính EF ( )O Gọi M trung điểm BC Khi ba điểm E, O, F thẳng hàng Lấy điểm N đối xứng với M qua AD H trung điểm MN Khi H thuộc AD Ta chứng minh đường thẳng d qua điểm N cố định Dễ thấy RMN OEP=

H F

D O

N K

E R M

P Q

C B

A

Do QR song song với MN nên ta có QD DE QD DE QE

DR DM DR DM RM

+

= = =

+ Dễ thấy hai tam giác vuông HDM MDE đồng dạng với nhai nên DE ME 2ME

DM=MH= MN Do ta

QE 2ME

RM= MN hay

MN 2ME

MR = QE Dễ thấy CBE QAC QAB= = theo giả thiết ta có PBE PBC CBE QBA QAB BQE= + = + = Trong tam giác FBE vng B có BM đường cao nên BE2=EM.EF Xét hai tam giác EBP EQB có PBE BQE= BEQ

chung nên hai tam giác EBP EQB đồng dạng với nhau, suy EP EB

EB =EQ Từ ta

được EP.EQ EB= =EM.EF 2EM.EO= nên 2EM EP

EQ =EO Từ ta

MN EP

MR =EO Xét hai tam giác OPE MNR có RMN OEP= MN EP

MR =EO nên hai tam giác đồng

dạng với nhau, suy MNR=EPO Gọi RN cắt OP K, dễ thấy tứ giác PHNK nội

tiếp nên ta

(65)

Lời giải 2.Dựng đường có AH tam giác ABC Qua H dựng đường thẳng vng góc với OD cắt đường thẳng qua D vng góc với OA X, từ ta X cô định Ta chứng minh đường thẳng d qua điểm X cố định

Thật vậy, gọi giao điểm OD với AH M, giao điểm OP với AH L Đường tròn ( )O cắt đường thẳng AD điểm thứ hai F

E L

X R

M H

O P Q

D

C B

A

Áp dụng định lí Menelaus cho tam giác ADM với ba điểmO, P, L thẳng hàng ta

LA PD OM

LM PA OD = , từ suy

LM PD OM

LA =PA OD Ta có AQB

PBD

S QA AB.BQ

S = PQ = BP.BD

PAB QBD

S PA BA.BP

S =QD = BQ.BD.Chú ý OF song song với AM,

kết hợp tỉ số ta

2

QA BA PD FA PD OM PD LM

QD = BD PA=FD PA= OD PA = LA Mà QR song song với AM nên ta RH QA LM

RD = QD = LA

Dễ thấy XDH∽ OAM nên suy XDR ∽ OAL dẫn đến XRD OLA= Gọi giao điểm XR OP E, tứ giác LERH nội tiếp nên ta suy

LER=90

Do đường thẳng qua R vng góc với OP qua điểm X cố định Từ ta có điều phải chứng minh

Bài 47.Cho tam giác ABC cố định Các điểm E F di động đoạn CA, AB

cho BF CE= Giao điểm BE CF D Gọi H, K trực tâm tam giác DEF DBC Chứng minh đường thẳng HK qua điểm cố định E F di động

(66)

Gọi AG phân giác góc BAC với G thuộc BC Đường tròn ngoại tiếp tam giác AGB AGC cắt AC AB M, N khác A

Dễ thấy tứ giác ANGC nội tiếp đường

trịn nên có BN.BA BG.BD= Tứ giác AMGB nội tếp nên đường trịn nên ta

có CM.CA CG.CB= Do AG phân

giác tam giác ABC nên AB GB

AC= GC

G D

K H

O

T P

F

E N

M A

C B

S

Từ ta BN BN.BA CA BG.BC CA BG CA AB AC CM=CM.CA AB = CG.CB AB =CG AB = AC AB =

Từ ta BN CM= , mà theo giả thiết ta có BF CE= nên ta NF ME.=

Từ ta có CNF CNF ABC BME ABC BME

S S S NF AC AC

S =S S =AB ME = AB

Lại có BM BM AD BC AB AB

CN = AD CN=AC BC = AC Từ ta

CNF BME

S CN

S = BM

Gọi ( )O đường tròn ngoại tiếp tam giác P điểm cung BC có chứa A đường tròn ( )O Ta chứng minh HK qua P

Thật vậy, gọi EH, FH cắt PB, PC S, T Do SE vng góc với FC nên ta có

0 0

0

1

ESB 360 SBC FCB 90 270 90 BAC FCB

2

180 FCB NCB 180 NCF

 

= − − − = − − −

 

= − − = −

Tương tự ta FTC 180= −MCE Dễ thấy SBE CNF

S SB.SE

S =CN.CF

TCF BME

S TC.TF

S = BM.BE

Mặt khác ý SEB TFC= nên ta

SBE SBE CNF BME TCF CNF BME TCF

S S S S

ES.EB SB.SE CN BM.BE SB ES EB

FT.FC=S =S S S =CN.CF BM TC.TF =TC FT FC

Từ ta suy SB

CT = hay SB TC= nên ta STsong song với BC Lại thấy H

(67)

CT đồng quy P.Vậy HK qua điểm cố định P Vậy ta có điều phải chứng minh

Bài 48 Cho điểm A nằm ngồi đường trịn ( )O Từ điểm A vẽ hai tiếp tuyến AB

AC (B, C hai tiếp điểm) cát tuyến ADE thay đổi với đường tròn ( )O cho tia

AD nằm hai tia AO AB Đường thẳng qua D song song với BE cắt BC, AB Q Gọi K đối xứng với B qua E Chứng minh đường thẳng PK qua điểm cố định

Lời giải

I

x

K Q

P H O

E D

C B

A

Gọi H I giao điểm BC với OA DE Ta có AB AC hai tiếp tuyến với đường tròn (O; R) nên AB AC= AO phân giác góc BAC Do AO đường cao tam giác ABC Hai tam giác ABD AEB có góc BAD chung

ABD AEB= nên hai tam giác đồng dạng với nhau, ta AB2 =AD.AE

Trong tam giác ABO vng có BH đường cao nên AB2=AH.AO Từ ta

AD.AE AH.AO= hay AH AD

AE =AO Từ suy hai tam giác HAD AEO đồng dạng

với nhau, ta có AHD=AEO Đến ta tứ giác OEDH nội tiếp đường

tròn nên OHE=ODE Tam giác ODE có OD OE= nên cân O, suy ODE=OED

Từ ta OHE=AHD Để ý OHE EHI+ =AHD IHD 90+ = nên ta

EHI IHD= , HI tia phân giác góc HED Gọi Hx tia đối tia HE, ta có xHA=AHD OHE= Do HA đường phân giác HED, từ ta suy

được ID AD

ED = AE Trong tam giác ABE có DQ song song với BE nên theo định lí Thales

vng góc với CF suy SH song song với BK Tương tự ta có CKsong song với

(68)

ta có DQ AD

BE = AE Trong tam giác IBE có BE song song với PD nên theo định lí Thales ta có DP ID

BE = IE Từ ta

DQ DP

BE = BE nên DQ DP= Trong tam giác ABE có DQ

song song với BE nên theo định lí Talets ta có AQ QD

AB = BE Do

AQ 2DQ PQ

AB = 2BE = BK Hai tam giác APQ AKB có AQ PQ

AB =BK AQP ABK= nên đồng dạng với Từ

đó ta QAP BAK= nên hai tia AP AK trùng Điều có nghĩa đường thẳng PK qua điểm cố định A

Bài 49. Cho đường trịn (O; R) đường kính AB cố định Lấy C điểm di động đường tròn ( )O cho BC AC Vẽ CH vng góc với AB H, HM vng góc với AC M Đường thẳng qua O song song với BC cắt tia tiếp tuyến A với đường trịn ( )O E Đường thẳng qua O vng góc với AB cắt BC D Gọi I giao điểm BE với AC Các tiếp tuyến O D đường tròn ngoại tiếp tam giác OBD cắt G Đường thẳng qua B vuông góc với IH cắt CH N Đường trịn

ngoại tiếp tam giác AMN cắt CH điểm K khác N Chứng minh đường thẳng GK qua điểm cố định

Lời giải Gọi F giao điểm FO AC, S giao điểm AE BC Khi dễ dàng thấy E F

lần lượt trung điểm AS AC Hai tam giác AHI CNB có HAI=NCB AHI=CNB nên hai tam giác đồng dạng với Do ta IA AH

CB= CN Tứ giác AMKN nội tiếp

đường trịn nên CK.CN CM.CA=

Lại có CH2 =CM.CA CH AH.BH 2

E S

H F

I M

O K G

N D C

B A

Do ta CK.CN AH.BH= hay AH CK

CN = BH Do

AI CK

CB= BH hay

BH CK

BC = AI Lại có CB2=BH.AB nên BH BC

BC =AB ta

CK BC

(69)

ta hai tam giác CBK ABI đồng dạng với nhau, từ suy CBK=ABI nên suy CBK=ABE Ta có hai tam giác BCA BAS đồng dạng với nhau, mà E F

lần lượt trung điểm AS AC nên suy hai tam giác BCF BAE đồng dạng với Do suy CBF=ABE, kết hợp với CBK=ABE ta CBK=CBF nên hai tia BK, BF trùng Do ba điểm B, K, F thẳng hàng Ta có hai tam giác ODG

và HCF đồng dạng với nên ta DG OD

CF =HC Hai tam giác BOD BHC đồng

dạng với nhaunên ta OD BD

HC= BC Do ta

DG BD

DF = BC mà lại có GDB FCB= nên hai tam giác DGB CFB đồng dạng với Từ ta suy DBG CBF= nên ta có CBG CBF= , hai tia BG BF trùng nhau, suy ba điểm B, G, F thẳng hàng Do ta đượcB, G, K, F thẳng hàng nên ba điểm B, G, K thẳng hàng Mà B cố định nên đường thẳng GK qua điểm B cố định

Bài 50. Cho tam giác ABC nội tiếp đường trịn ( )O có H trực tâm Đường trung tuyến AM cắt đường tròn ( )O N khác A Đường thẳng AH cắt đường tròn ( )O điểm K khác A Các đường thẳng KN, BC đường thẳng qua H vng góc với AN cắt theo đôi tạo thành tam giác XYZ Chứng minh đường tròn ngoại tiếp tam giác XYZ tiếp xúc với đường tròn ( )O

Lời giải

Giả sử điểm X, Y, Z có vị trí hình vẽ Gọi T giao điểm thứ hai đường trịn đường kính AH với đường trịn ( )O Gọi P hình chiếu H AM Vẽ đường kính AQ đường tròn ( )O Do H trực tam tam giác nên ta có CH vng góc với AB nên suy CH song song với BQ, tương tự BH song song với CQ, suy tứ giác BHCQ hình bình hành Mà M trung điểm BC nên ta có ba

(70)

Q L

P T

K

N H

M

Z Y

X

O

C B

A

Xét tam giác AMH có hai đường cao cắt Y nên Y trực tâm tam giác AHM, lại có AT vng góc với MH nên suy AT đường có cịn lại tam giác hay ba điểm A, T, Y thẳng hàng Đến ta có biến đổi góc

0 0

HXK 90= −PNK 90= −ANK 90= −AQK KAQ QTK HTK= = =

Suy tứ giác XTHK nội tiếp đường trịn.Do ta TXK THA TYB= = nên tứ giác XZTY nội tiếp đường tròn Như T giao điểm đường tròn ( )O với đường tròn ngoại tiếp tam giác XYZ Từ T kẻ tiếp tuyến TL với đường trịn ( )O ,

đó để ý đến tứ giác nội tiếp đường trịn ta có biến đổi góc

XTL XTK LTK XHK TAK TYH TZX= − = − = =

(71)

Bài 51.Cho ta giác ABC nội tiếp đường tròn (O; R) Quay tam giác ABC góc

0

90 quanh O ta thu tam giác A B C1 1 1 Tính diện tích phần chung hai tam

giác ABC A B C1 1 1

Lời giải

Giả sử đường tròn ( )O có chiều A→ → →B C A

chiều ngược với chiều kim đồng hồ Giả sử quay tam giác ABC góc 900quanh điểm O theo chiều

thuận kim đồng hồ thu tam giác A B C1 1 1

Khi điểm A ; B ; C1 1 1 thuộc cung

AC; AB; BC Do cạnh A B1 1 cắt AB, AC M, N

T K

E

C1 B1

A1 O

A' Q

P N M

C B

A

Tương tự A C1 1 cắt AC, BC P, Q B C1 1 cắt BC, BA T, K Như phần chung hai tam giác ABC A B C1 1 1 đa giác MNPQTK Gọi diện tích lục giác MNPQTK S, ta S S= ABC−SAMN−SBKT−SCPQ Mặt khác

ta có

2

ABC

BC 3 3R

S

4

= =

Khi quay góc 90 quanh điểm O ta OA1⊥OA; OB1⊥OB; OC1 ⊥OC Từ ta suy OA1song song với BC, OB1 song song với CA, OC1 song song với

AB Gọi giao điểm OA1 với AC E Do cung AA ; BB ; CC1 1 1 có sơ đo

0

90 nên ta AMN 90= 0, mà ta lại có MAN=600 nên ta suy AN 2AM=

Dễ thấy NEA1 cân E Lại OE song song với BD AO 2AA'

= nên ta

2 2R

AE AC

3

= = OE BC R

3

= = Từ ta EN EA1 OA1 OE R R

3

= = − = −

Đến suy AN AE EN R R R ( R)

3

 

= − = − − = −

  Do ta

( ) ( )

2 AMN

3 R 3 R

MA.MN AN AN 3AN

S

2 2 8

− −

(72)

Hoàn toàn tương tự ta ( )

2 BTK CPQ

2 3 R

S S

4 −

= =

Do ta ( )

2

9 3 R S

4 −

=

Bài 52.Cho tứ giác lồi nội tiếp ABCD có đường trịn nội tiếp tâm I Gọi O giao điểm AC BD Chứng minh

2

AO AI

CO = CI

Lời giải

Gọi r bán kính đường tròn tâm I Giả sử

M,N,P,Q tiếp điểm đường tròn tâm I với cạnh AB, BC,CD, DA Đặt

AM AQ x; BM BN y; CN CP z; DP DQ t= = = = = = = =

Ta có AI r ,CI r

A C sin sin 2 = = O Q P N M I C D B A

Do ta suy

2 2 C sin

AI 2 cos A

A cos C

CI

sin

= =

− Sử dụng định lý cosin cho tam giác

BAD ta có ( ) ( )

( )( )

2 2

2 2 x y x t BD

AB AD BD

cos A

2AB.AD x y x t

+ + + −

+ −

= =

+ +

Suy ( ) (( )( ) ) ( ()( ))

2 2 2

x y x t BD BD y t

1 cos A

2 x y x t x y x t

+ + + − − −

− = − =

+ + + +

Tương tự ta chứng minh

( ) ( )

( )( ) ( ()( ))

2 2 2

z y z t BD BD y t

1 cos C

2 z y z t z y z t

+ + + − − −

− = − =

+ + + +

Do ( )( )

( )( ) CBD ABD ABD CBD

x y x t 2S 2S S

AI AB.AD AO

:

CB.CD S CO

z y z t

CI sin BAD sin BCD

+ +

= = = = =

+ +

(73)

minh tam giác ABC vuông B diện tích hai tam giác BDE HDE

Lời giải

Do AC 2AB= nên ta AB AC , ta

được ACB ABC nên ta ACB 90

Từ A900, ACB 90 AB AC suy điểm K nằm tam giác

Do DB AB

DC =AC= nên ta

BD CD

,

BD=3 BC =

H K E D C B A

+ Điều kiện cần.Giả sử tam giác ABC vng B Khi dễ thấy

BAC 60= Từ ta AH 1AB 1AC

2

= = AH 1CH

= Áp dụng định lí Menelaus cho tam giác

ABC với ba đường thẳng AD, BH, CE đồng quy ta EA DB HC

EB DC HA= nên ta

AE

EB = 3,

BE EA

;

BA =5 AB=

Khi ta AEH ABC

S AE.AH

S = AB.BC =10

CDH ABC

S CD.CH

S = CB.CA =2

Cộng theo vế ta SAEH SCDH 2SABC

+ = nên SBEHD 2SABC = Mà ta có BDE

ABC

S BD.BE

S = BC.BA =5 nên ta BDE BEDH

1

S S

2

= Từ suy SBDE =SHDE

+ Điều kiện đủ.Giả sử SBDE =SHDE

Theo định lí Menelaus ta có EA DB HC

EB DC HA= nên ta

AE 2AH

BE = HC

Suy AE 2AH

AB =AC AH+

EB AC AH

AB AC AH

− =

+

Do BDE EAH CDH

ABC ABC ABC

S

2S S

1

S +S + S = hay

2BD.BE AE.AH CD.CH

1

BC.BA +AB.AC + BC.CA = Từ ta

( )

( ) ( ) ( )

( )( )

2

2

2 AC AH 2AH AC AH

1 3AC

3 AC AH AC AH AC

AC 5AH.AC 4AH AC 4AH AC AH AC 4AH

− −

+ + =

+ +

(74)

Từ AC 4AH= ta

2 AC

AB AC.AH

4

= = nên tam giác ABC vng B

Vậy ta có điều phải chứng minh

Bài 54. Cho tam giác nhọn ABC có ba đường cao AA’, BB’, CC’ Gọi D, E F tâm đường tròn nội tiếp tam giác AB’C’, BC’A’ CA’B’ Tính bán kính đường trịn ngoại tiếp tam giác DEF theođộ dài cạnh tam giác ABC

Lời giải

Trước hết ta phát biểu chứng minh bổ đề. Cho

tam giác ABC vng A có C=  450 Khi ta ln có sin2 =2sin cos  

Chứng minh.Gọi AH đường cao AM a=

đường trung tuyến tam giác ABC B H M C

A

Khi tam giác AHC có AHC 90 ; C= = nên AH CH

sin ; cos

AC AC

=  Vì AM trung

tuyến ứng với BC nên AMB 2=  Xét tam giác AHM có AHM 90 ; AMH 2= =  suy sin AMH AH

AM

= hay sin AH

a

=

Do sin cos 2.CH 2AH.CH2 2AH.CH AH

AC AC BC.CH a

 = = = =

Do ta sin2 =2sin cos 

Trở lại tốn.Gọi I tâm đường trịn nội tiếp tam giác ABC Gọi r, p, S bán kính đường trịn nội tiếp, nửa chu vi diện tích tam giác ABC Khi dễ dàng chứng minh S pr= Đặt

AB c; BC a; CA b= = = BAC ; CBA ; ACB 2=  =  = 

F E

D

I C'

B'

A' C

B

A

Khi ta 0  ; ; 900   + + =900 Dễ dàng chứng minh

C' A' B CA' B' ; A' B' C= =  =AB' C' ; B' C' A=  =BC' A' 2= 

(75)

Do ta A' EB∽ A' FB'∽ AIB Suy ta có A'E A'F AI A' B= A' B'=AB

Do ta lại A' EF∽ A' BB' nên ta EF A'E

BB' =A' B'

Từ ta EF AI EF AI BB' BB'.AI AI.sin

BB' =AB = AB = AB = 

Cũng từ A' EF∽ A' BB' ta có EFA' BB' A 90= = 0−CBA 90= 0−2

Do ta IFE IFA' EFA'= − =( + )−(900−2)= + + + −    900 = = CBE

Tương tự ta IDE IFE= = IDF IEF= = nên ta EDF= + 

Hoàn toàn tương tự ta FED= + ; DFE= + 

Gọi R bán kính đường trịn nội tiếp tam giác DEF Áp dụng định lí sin cho tam giác DEF áp dụng đổ đề ta ta

( )

EF EF EF AI.sin

R IA.sin r

2cos 2cos

2 sin sin EDF

 

 

 

= = = = = =

+

Từ ta R r S p p a p b p c( )( )( ) (p a p b p c)( )( )

p p p

− − − − − −

= = = =

Vậy bán kính đường tròn ngoại tiếp tam giác DEF R (p a p b p c)( )( ) p

− − −

=

Bài 55. Cho tam giác ABC nội tiếp đường trịn (O; R) có đường cao AD, BE, CF Tìm cơng thức liên hệ diện tíchtam giác ABC với chu vi tam giác DEF

Lời giải

Ta xét trường hợp sau

+ Trường hợp 1.Tam giác ABC vuông, khơng tính tổng qt ta giả sử A=900 Khi điểm E, F trùng với A nên khơng tồn tam giác DEF

+ Trường hợp 2.Tam giác ABC nhọn Gọi H trực tâm tam giác ABC Trước hết ta chứng minh điểm O nằm

trong tam giác ABC

Thật vậy, vẽ đường kính AT cắt CB V Giả sử O nằm

R P

O H

F E

D C

B

(76)

ngoài tam giác ABC thuộc đường thẳng VT, ta

được

180 BOT COT+ =2BAO 2CAO+ =2BAC

suy BAC 90 0, mâu thuẫn với tam giác ABC nhọn

Như tam giác ABC nhọn O nằm tam giác Gọi K giao điểm AO FE Dễ thấy tứ giác AEHF BDHF nội tiếpđường trịn Kẻ OP vng góc với AC

P, theo tính chất góc nội tiếp ta có

AEF=AHF=ABC=AOP=90 −OAE Do ta AKE 90= hay OA vng góc với EF Chứng minh tương tự ta OB vng

góc với DF OC vng góc với DE Từ ta SABC =SAFOE+SBFOD+SCDOE Suy 2SABC =OA.EF OB.FD OC.DE R EF FD DE+ + = ( + + )=2R.pDEF

Với pDEF nửa chu vi tam giác DEF

+ Trường hợp 3. Tam giác ABC tù, không tính tổng

quát ta giả sử

BAC 90 Khi ta có H O nằm tam giác ABC Chứng minh tương tự ta OA

vng góc với EF Xét đường tâm Q đối xứng với đường tròn tâm O qua BC Dễ thấy ( )Q đường tròn ngoại tiếp tam giác HBC hai đường tròn ( )O đường

trịn ( )Q có bán kính Đoạn thẳng OQ vng

góc với BC M Xét tam giác HBC có ba đường cao HD,

CE BF

M

N Q

O H

F E

D C B

A

Khi áp dụng kết trường hợp ta SHBC =R DE EF FD( + + ) Mặt khác ta có

( )

HBC ABC ABH ACH ABC ABC

2S =2 S +S +S =2S +AH.BD AH.CD 2S+ = +AH.BC

Từ ta 2SABC+AH.BC R DE EF FD= ( + + )2SABC =R DE EF FD( + + )−AH.BC

Kẻ BQ cắt đường tròn tâm Q N, ta thấy AH//CN//OQ AH CN 2MQ OQ= = = Trong tam giác vng BQM có MQ2+MB2 =R2 4MQ2+BC2 =4R2

Từ ta ( ) 2

ABC DEF

(77)

Bài 56.Cho tam giác ABC có diện tích S ngoại tiếp đường trịn ( )O; r Kẻba tiếp tuyến đường tròn ( )O; r gồm tiếp tuyến song song với BC cắt CA, AB M, N; tiếp tuyến song song với CA cắt AB, BC P, Q; tiếp tuyến song song với AB cắt BC, AC R, S Chứng minh ta ln có SMNPQRS 2S

3 

Lời giải Gọi diện tích tam giác AMN, BPQ, CRS

là S ; S ; S1 Gọi chiều cao hại từ A tam giác

AMN ABC h1 ha ta thu

1 a

h =h −2r Dễ thấy hai tam giác ANM ABC

đồng dạng với nhaunên ta

2 2

a

1

a a a

h 2r

S h 2r

1

S h h h

   −    =  =  = −        S R Q P N M C B A

Tương tự gọi h ; hb c đường cao hạ từ B, C tam giác ABC Khi ta

có 2 b S 2r S h   = −   

2 c S 2r S h   = − 

  Dễ thấy a b c ( )

1 1

S a.h b.h b.h r a b c

2 2

= = = = + +

Suy

a b c

1 a b c a b c 1

r 2S 2S 2S 2S h h h

+ +

= = + + = + + Từ ta

2 2

1

a b c a b c

2 2

a b c

S S S 2r 2r 2r 2r 2r 2r

1 1 1

S h h h h h h

1 1 1 1

3 2r 2r

3 h h h r

        + + = −  + −  + −    − + − + −               =  −  + +  =  −  =       

Do suy S1 S2 S3 1S

+ +  nên ta SMNPQRS 2S

 Dấu xẩy rakhi

khi a b c

a b c

2r 2r 2r

1 1 h h h

h h h

− = − = −  = = hay tam giác ABC

Bài 57.Cho tam giác ABC nội tiếp đường tròn (O; R) ba đường trung tuyến AM, BN, CP Chứng minh

a b c

1 1

m +m +m R

(78)

A'

P N

M O

C B

A

N' P'

A' P

M N

C B

A

Ta xét hai trường hợp sau đây

+ Trường hợp 1. Xét tam giác ABC khơng tù Ta có R OA AM OM=  − , dấu xẩy O thuộc đoạn AM Do ta

a

R OA AM OM OM

1

m MA MA AM

=  = − Gọi AA’

đường cao tam giác ABC Khi ta lại có OBC ABC

S

OM OM

AM  AA'=S , đẳng thức xẩy

và hai điểm M, A’ trùng nhau.Do ta OBC

a ABC

S R

1

m  −S , đẳng thức xẩy

khi O thuộc đoạn AM đồng thời hai điểm M, A’ trùng nhau, điều có nghĩa tam giác ABC cân A Hoàn toàn tương tự ta OAC

b ABC

S R

1

m  −S

OAB

c ABC

S R

1

m  −S Dấu xẩy B, C Do O nằm tam giác ABC nên ta có SOAB+SOBC+SOCA =SABC Từ ta

OAB OBC OCA

a b c ABC

S S S

R R R

3

m m m S

+ +

+ +  − = − =

Hay ta

a b c

1 1

m +m +m R Dấu xẩy tam giác ABC

+ Trường hợp 2. Xét tam giác ABC tù Khơng tính tổng qt ta giả sử A900

Qua A kẻ đường thẳng vng góc với BC cắt đường tròn (O; R) A’ Gọi N’ P’ trung điểm A’C A’B

Khi thấy ' ' '

a a b b c c

m =AM A'M m ; m = =BN BN' m ; m = =CP CP' m =

Do ta ' ' '

a b c a b c

1 1 1

m +m +m  m +m +m ta có tam giác A’BC tam giác

nhọn Chứng minh tương tự trường hợp ta ' ' '

a b c

1 1

R

(79)

Do ta

a b c

1 1

m +m +m R

Vậy ta ln có

a b c

1 1

m +m +m R Dấu xẩy tam giác ABC

Bài 58.Cho tam giác nhọn ABC có diện tích S BC a= Trên cạnh BC lấy điểm D

cho DB k

DC= Tính diện tích tam giác có đỉnh tâm đường trịn ngoại tiếp tam giác ABC, ABC, ACD theo a, k, S

Lời giải Gọi O; O ; O1 2 theo thứ tự tâm đường tròn ngoại tiếp tam giác ABC, ABD, ACD Khi ta suy OO1vng góc với ABtại M

MA MB= , OO2vng góc với AC N

NA NC= Kẻ O E1 vng góc với BC O E1

vng góc với OF Các điểm M E nằm đường trịn đường kính BO1 nên OO F1 =ABH

F Q M R N K H E P D G O2 O1 O C B A

Từ ta OO F1 ∽ ABH nên ta OO1 AB

OF =AH hay

AB

OO OF

AH

= (1)

Ta lại có OF PE PE EB( ) 2EB BC BD CD

2 2

+ − −

= = = =

Từ giả thiết suy BC k CD= + nên

a CD

k =

+ , ta ( )

a OF

2 k =

+

Kết hợp với (1) ta

( )

1

a AB

OO

AH k =

+ (2)

Tương tự kẻ O G2 ⊥BC OK⊥O G2 ta

( )

2

ak AC

OO

AH k =

+ (3)

Kẻ BR⊥AC O Q2 ⊥OO1 Xét hai tam giác O OQ2 BAR ta có O OQ2 =BAR nên

suy O OQ2 ∽ BAR Từ ta

2

2

O Q BR BR

O Q O O

O O =AB = AB (4)

(80)

( ) ( )

1

2

OO O 2 2

1 BR ka 2S k.a

S O Q.OO OO OO

2 AB 8 k 1 2S 16S k 1

a

= = = =

+   +

   

Bài 59. Cho tam giác ABC có r ; r ; ra b clần lượt bán kính đường trịn bàng tiếp góc A, B, C Gọi R, r bán kính đường tròn ngoại tiếp nội tiếp tam giác

Chứng minh ra+ + =rb rc 4R r+

Lời giải Gọi D, E, F tâm đường trịn bàng tiếp góc A, B, C S diện tích tam giác ABC Đặt BC a, CA b, AB c= = = p a b c

2 + +

=

Ta thấy SDAB+SDAC−SDBC = S r b c aa( + − )=2S

Hay ta r p aa( − )=S

Tương tự ta có r p bb( − ) (=r p cc − )=S

O I F

E

D C B

A

Cộng theo vế đẳng thức ta có p r( a+ +rb rc) (− r a r b r ca + b + c )=3S Do suy

ra p r( a + +rb rc) (=2 SBDC+SECA+SFAB)+3S nên p r( a+ +rb rc)=2SDEF+S

Gọi O tâm đường trịn ngoại tiếp tam giác DEF, dễ thấy A, B, C chân đường cao tam giác DEF nên bán kính đường trịn ngoại tiếp ABC

2 bán

kính đường trịn ngoại tiếp tam giác DEF Dễthấy OD vng góc với BC nên ta

OBDC

1

S OD.BC R.a

2

= = Tương tự ta có SOCEA =R.b; SOAFB =R.c

Cộng theo vế đẳng thức ta có SDEF =R a b c( + + )=2Rp nên DEF

2S

4R

p =

Từ kết ta ra+ + =rb rc 4R r+

Bài 60.Cho tam giác ABC nội tiếp đường tròn tâm O Chứng minh tổng bình phương khoảng cánh từ điểm đường trịn đến cạnh tam giác ABC bình phương đường cao tam giác

(81)

Giả sử K điểm đường trịn ( )O Khi điểm K trùng với đỉnh tam giác ABC kết luận hiển nhiên Xét điểm không trùng với đỉnh tam giác ABC Khơng tính tổng qt ta giả sử K nằm cung nhỏ AC đường tròn ( )O

Gọi A ; B ; C1 1 1 hình chiếu K cạnh BC, CA, AB Gọi AH đường cao tam giác ABC

H K

O

C1

B1 A1

C B

A

Ta cần chứng minh 2 2

1 1

KA +KB +KC =AH

Trước hết ta chứng minh ba điểm A ; B ; C1 1 1thẳng hàng

Thật vậy, dễ thấy AKC C KA= 1 1=1200 nên tứ giác AC B K; KB CA1 1 1 1 nôi tiếp đường trịn có đường kính tương ứng AK CK Từ ta AKC1 =AB C1 1

1 1

CKA =CB A Từ

1

AKC C KA= =120 ta AKC1+C KC C KC CKA1 = 1 + 1 Do ta AKC1 =CKA1 Do AB C1 1=CB A1 1 suy ba điểm A ; B ; C1 1 1thẳng

hàng

Để ý

1 1 1

A KC C KB B KA

S =S +S CKB=BKA 60= nên KC KA1 1 =KC KB1 1+KB KA1 1

Mặt khác ta lại có SAKB+SBKC−SAKC =SABC

Do ta AB.KC1+BC.KA1−AC.KB1=BC.AH hay KC1+KA1−KB1=AH

Từ ta suy 2 2

1 1 1 1 1

KC +KA +KB +2KC KA −2KC KB −2KB KA =AH

Mà ta có KC KA1 1=KC KB1 1+KB KA1 1 nên 2KC KA1 1 =2KC KB1 1−2KB KA1 1=0

Do ta 2 2

1 1

KA +KB +KC =AH Vậy toán chứng minh

Bài 61. Cho tam giác ABC có m ,l ,la b c p theo thứ tự độ dài đường trung tuyến hạ từ đỉnh A, độ dài đường phân giác hạ tứ đỉnh B, C nửa chu vi tam giác Chứng minh ma+ + lb lc p

(82)

Trước hết ta phát biểu chứng minh bổ đề.Với

mọi 0  450 ta ln có cos

cos

2

 = +

Chứng minh. Xét tam giác ABC vuông A có

C= đường cao AH, đường trung tuyến AM B H M C

A

Trong tam giác AHM có AHM 90 ; AMH 2= =  cos2 HM

AM

=

Do cos2 HM AM HM CM HM HC

AM AM AM AM

 + +

+ = + = = =

Ta có

2 2

2 CH 2CH 2CH 2CH CH

2 cos

AC BC.CH BC 2AM AM

=  = = = =

 

Từ ta cos2 cos

2

 = + , bổ đề chứng minh

Trở lại toán.Đặt AB c; BC a; CA b= = = , theo cơng thức đường phân

giác ta có

b 2b B

2cacos B B

2

l ca.cos l ac.cos

c a 2

=   

+

Áp dụng bổ đề ta có cos2 B cos B

2

+

= , từ ta b

1 cos B l ac

2  + 

  

 

Mà theo định lí cosin ta có

2 2

c a b

cos B

2ca + −

=

Suy ( ) ( )

2 2

2

b b

ac c a b

l p p b l p p a

2 2ca

 + − 

  + = −   −

  Tương tự lc  p p c( − )

Cũng theo công thức đường trung tuyến ta có

( ) ( )

( )( ) ( )( )

2

2 2 2

a

4m 2b 2c a b c a b c

b c p b p c b c p b p c

 

= + − = + − − − 

 

   

= + + − −    + − − − 

Mặt khác ta lại có

( )( )

( )( ) ( )2

b c p b p c b c 2p b c 2p

b c p b p c 2p p b p c

+ + − −  + + − − =

(83)

Do 4ma2 2p 2p −( p a− − p b− )2 p p b( − )+ p p c( − ) p( 2−ma2)

 

Suy lb+ lc p( 2−m2a)

Do ta ( 2) ( )( 2 2)

a b c a a a a

m + + l l m + p −m  m+ +p −m =p

Dấu bẳng xẩy tam giác ABC

Bài 62 Cho tam giác nhọn ABC có h , h , ha b c l ,l ,la b c tương ứng đường cao đường phân giác hạ từ đỉnh A, B, C Gọi r R bán kính đường tròn nội tiếp đường tròn ngoại tiếp tam giác ABC Chứng minh rằng:

a b c

a b c

h A h B h C r

sin sin sin

l l l 4R

   

− − − 

   

   

Lời giải

Bổ đề.Trong tam giác nhọn ABC ta ln có sinAsin sinB C

2 2 8

Chứng minh Vẽ đường phân giác AD ta có BD CD BD BD CD BC

AB AC AB AB AC AB AC

+

=  = =

+ +

Vẽ BI⊥BCBI BD Tam giác ABI có

A BI BD BC BC

sin

2 = AB AB= AB AC+ 2 AB.AC

Chứng minh tương tự ta có

B AC C AB

sin ; sin

2 2 AB.BC 2 AC.BC

I A

B D C

Nhân vế với vế bất đẳng thức ta sinA.sin sinB C

2 2 8

Trở lại toán.Gọi AA’ đường phân giác hạ từ đỉnh A, gọi p nửa chu vi tam giác ABC Đặt

AB c; BC a; CA b= = = Ta có SABC =SABA'+SACA' Mà ta lại có

ABC a ABA' a ACA' a

1 A A

S a.h ; S b.l sin ; S c.l sin

2 2 2

= = =

la ha

A' C

B

(84)

Do ( ) a

a a

a

h

1 A b c A

a.h b c l sin sin

2 2 l a

+

= +  =

Suy a ( )

a

2 p a

h A b c A b c a A A

sin sin sin sin

l a a a

 +  + −

− = −  = =

 

Hoàn toàn tương tự ta b ( ) c ( )

b c

2 p b p c

h B B h C C

sin sin ; sin sin

l b l c

− −

− = − =

Do a b c ( )( )( )

a b c

8 p a p b p c

h A h B h C A B C

sin sin sin sin sin sin

l l l abc 2

− − −

   

− − − =

   

   

Mà theo bổ đề sinAsin sinB C

2 2 8 theo công thức diện tích ABC abc S

4R =

Và công thức Heron SABC = p p a p b p c( − )( − )( − ) ta

( )( )( )

8 p a p b p c A B C S r

.sin sin sin

abc 2 4Rp 4R

− − −

 =

Do ta a b c

a b c

h A h B h C r

sin sin sin

l l l 4R

   

− − − 

   

   

Bài 63.Cho hình vng ABCD có cạnh a hai điểm M, N thay đổi BC,

CD cho góc

MAN=45 Tìm giá trị lớn nhỏ diện tích tam giác

AMN

Lời giải

Đặt BM=x; DN=y x; y a(   ) Khi ta có

( )

AMN ABCD ABM ADN CMN

S =S − S +S +S

Hay ta

( )( ) ( )

2

AMN

1

S a ax ay a x a y a xy

2 

= −  + + − − = −

Trên tia đối tiaBM lấy điểm K cho BK y=

K y y x N M D C B A

Khi ta ABK= ADN Từ AN AK= BAK=DAN

Để ý

BAM DAN 45+ = nên ta

BAK BAM KAM+ = =45

Dễ thấy AKM= AMN nên ta MN MK x y= = + Mặt khác từ tam giác vuông

(85)

( )2 2 2 2 2 2 ( ) ( ) 2

x y+ =a −2ax x+ +a −2ay y+ xy a= −a x y+ a x y+ =a −xy

Do SAMN 1a x y( ) 1at

2

= + = với t= +x y Đến ta nhận thấy t lớn diện tích tam giác AMN lớn ngược lại Như ta cần tìm giá trị lớn nhỏ t

Để ý ta có x y a+ = x.y a= 2−at Khi thaeo hệ thức Vi – et ta có x, y

nghiệm phương trình bậc hai X2−tX a+ 2− =at 0 Để phương trình có hai

nghiệm x, y ta cần có

( ) ( )2 ( )

2 2

t a at t 2a 8a t 2a 2a t 2a

 = − −   + −   +    −

Khi t 2a= ( 1− ) phương trình bậc hai có nghiệm kép

( ) ( )

1

2a

t

X X a

2

= = = = −

Điều có nghĩa x y a= = ( 1− ) Mint 2a= ( 1− )

Vậy ta ( ) ( )

AMN

2 S

1

Min a.2a a

2

= − = −

Lại có xy a= 2− at at a= 2−xy nên suy at a 2 t a

Điều có nghĩa Maxt a= ,

AMN

2 S

1 a

Max a.a

2

= =

Trong trường hợp ta x a; y 0= = x 0; y a= = hay M B; N C 

M C; N D 

Bài 64. Cho hình chữ nhật ABCD có AB BC Vẽ nửa đường trịn đường kính AB

trên nửa mặt phẳng chứa CD có bờ đường thẳng AB Gọi M điểm nửa đường trịn(MA, B) Các đường thẳng MA MB cắt CD P Q Các đường thẳng MC, MD cắt đường thẳng AB E F

(86)

Đặt AB CD a, BC= = =b a( b) Kẻ MN vng góc

với BC N, theo định lí Talet ta có

PQ QM CN

AB = BM = BN

EF EM BN

CD=MC =CN Suy PQ AB.CN a.CN

BN BN

= = EF CD.BN a.BN

CN CN

= =

Do ta PQ EF a CN BN

BN CN

 

+ =  +   

Q C P

M F E D N B A

Đặt S CN BN

BN CN

= + , ta có

( )2

2 CN BN 2CN.BN

CN BN CN BN b

S

BN CN CN.BN CN.BN CN.BN

+ −

+

= + = = = −

Do ta CN.BN2

S 2+ = b nên

( )2 ( )2

2

CN BN 4CN.BN CN BN

4

S b b

+ − −

− = =

+

Do M nằm đường trịn đường kính AB nên NB AB a

2

 = , suy CN 2b a

− 

Do ta ( )

2

2

2

b a

4 2ab a

1

S b S b

− −

−   

+ + , nên ta có

2

2

4b 4ab 2a

S

2ab a − + 

Từ ta suy

2

4b 4ab 2a

EF PQ

2b a − + + 

− , đẳng thức xẩy

a NB

2 = hay

M nằm nửa đường trịn đường kính AB Vậy giá trị nhỏ PQ EF+

2

4b 4ab 2a

2b a − +

− , xẩy M nằm nửa

đường trịn đường kính AB

Bài 65.Trong tam giác nội tiếpđường trịn (O; R) cho trước, tìm tam giác có chu vi lớn nhât

(87)

Xét tam giác ABC nội tiếp đường tròn (O; R) Giả sử điểm M điểm cung BAC Kẻ đường kính MN đường trịn (O; R), MN vng góc với BC trung điểm H BC MBN 90= Đặt

MH h.= Trong tam giác vng MBN có BH đường

cao nên ta có

H O

N M

C B

A

2

MB =MN.MH 2R.h= ( )

BH =MH.MH=h 2R h−

Gọi p p1 nửa chu vi tam giác ABC MBC, theo ta có p p 1

và dấu xẩy A M trùng

Ta có ( )

2 2 2 2

2

2 2

1

MB MB MB MB MB

p MB BH BH BH BH

2 4

   

 

= + = + +    + + =  + 

     

Dấu xẩy MB BH

2 = hay MB BC=

Theo ta có ( ) ( )

2

2

2

MB 3R 27R

3 BH Rh h 2R h 3h 3R h

2

 + =  + − = −    =

     

 

 

Dấu xẩy h 3R h h 3R = −  =

Do 2

1

27

p R

4

 hay p1 3R

 , dấu xẩy

MB BC

MH R

2

 =

 

=

 hay tam

giác MBC

Từ ta 2p 3R , dấu xẩy A M tam giác MBC

đều hay tam giác ABC Vậy tam giác nội tiếp đường trịn (O; R) tam

giác có chu vi lớn chu vi lớn 3R

Bài 66 Cho tứ giác ABCD nội tiếp đường tròn Chứng minh rằng: AB CD−  AC BD−

(88)

Gọi E, F trung điểm AC, BD Khi ta có áp dụng cơng thức đường trung tuyến tam giác ta

2 2

2 2

2 2

1

AB AD 2AE BD

2

BC CD 2CE BD

2

EA EC 2EF AC

2

+ = +

+ = +

+ = +

M

F E

D B

C

A

Cộng theo vế đẳng thức ta

( )

2 2 2 2 2

AB +BC +CD +DA =2 AE +EF +BD =BD +AC +4EF

Do tứ giác ABCD nội tiếp nên theo định lí Ptoleme ta AC.BD AB.CD AD.BC= +

Từ ta (AB CD− ) (2+ AD BC− ) (2 = AC BD− )2+4EF2

Gọi M trung điểm AB, ta AD 2ME; BC 2MF= =

Từ suy ME MF− = AD BC−

Mà tam giác MEF ta có EF ME MF− 2EF AD BC− 4EF2 (AD BC− )2

Do kết hợp với đẳng thức ta

( ) (2 ) (2 ) (2 )2

AB CD− + AD BC−  AC BD− + AD BC− Suy (AB CD− ) (2  AC BD− )2 AB CD−  AC BD−

Dấu xẩy EF= ME MF−  ba điểm M, E, F thẳng hàng, điều dẫn đến tứ giác ABCD hình thang hình chữ nhật

Bài 67 Cho tam giác ABC đường tròn ( )I nội tiếp tam giác tiếp xúc với cạnh AB, BC, CA F, D, E Gọi M giao điểm BC với đường phân giác góc BIC N giao điểm EF với đường phân giác góc EDF

Chứng minh rằngba điểm A, M, N thẳng hàng Lời giải

(89)

+ Xét trường hợp tam giác ABC khơng cân A Khơng tính tổng qt ta giả sử AB AC

Gọi P, Q theo thứ tự giao điểm AI với ( )I

và EF Do P giao điểm AI với đường tròn

( )I nên P nằm cung nhỏ EF đường trịn ( )I Khi dễ thấy ba điểm D, N, P thẳng hàng nên ta có

P Q N

M D I F

E

C B

A

( )

( ) ( )

( )

0

0

0 0

0

0

1

IMC MIB MBI BIC MBI 180 IBC ICB MBI

2

1 ABC ACB ABC ABC ACB

180 90

2 2 4

1

NDC NDE EDC FDE EDC 180 90 IBD 90 ICD

2

1

180 FDB EDC EDC

2

IBD ICD ABC ACB

90 90

2 4

• = + = + = − − +

 

=  − − + = + −

 

 

• = + = + =  − − − − 

 

= − − +

= + − = + −

Do ta IMC=NDC ta IM song song với ND Do ta suy IM song song với Để ý ID IP= nên ta MID IDP QPN= = Do BC tiếp xúc với đường trịn ( )I D P điểm cung EF nên ta

1 1

IDM PDM IDP sdPED IDP sdPE sdED IDP

2 2

1

sdPF sdED IDP PNF QPN PQN

2

= − = − = + −

= + − = − =

Do ta IDM∽ PQN, suy IM DI

PN =QP mà ta lại có ID IP=

IM PI

PN =QP

Mặt khác ta có IAE=900, EQ⊥IP IE=IP nên ta IQ.IA IE= =IP2

Do ta có QP IQ IP PA IP IA

IP = − IP = −IA= IA PQ= AP nên ta

IM AP

PN= IA

(90)

Bài 68. Gọi O tâm đường tròn nội tiếp tam giác nhọn ABC Đường tròn ( )O tiếp xúc với cạnh AB, BC, CA theo thứ tự F, E, D Đường phân giác góc

BOC cắt BC I AI cắt EF K Chứng minh KD 4DE.DE EF2

 −

Lời giải

Ta định nghĩa lại điểm K giao điểm đường phân giác góc EDF Khi ta ta chứng minh ba điểm A, K, I thẳng hàng Để chứng minh ba điểm A, K, I thẳng hàng ta trình bày tương tự lời giải tốn 67 Ở ta trình bày lời giải theo hướng khác sau

Ta có biến đổi góc sau

0 o ABC o ACB ABC ACB

EDF 180 FDB EDC 180 90 90

2 2

    +

= − − = − −   − − =

   

Mà ta lại có KDC EDF FDC ABC ACB 900 ACB 900 ABC ACB

2 4 4

= + = + + − = + −

Mặt khác OI phân giác góc BOC nên ta có

0

ABC ABC BOC ABC ACB ABC ACB

OIC BOI 180 90

2 2 2 4

 

= + = + =  − − = + −

 

Kết hợp hai kết ta suy KDC OIC= nên OI song song với DK

Gọi giao điểm DK với đường trịn ( )O P, dễ thấy ba điểm A, P, O thẳng hàng Giả sử AO cắt EF Q Do DK song song với OI nên ta có KPO KDO= =DOI,

điều dẫn đến hai tam giác KQP DIO đồng dạng với Từ ta

KP PQ PQ

OI =OD =OF Mặt khác ta lại có P điểm cung nhỏ EF nên FP phân

giác tam giác AFQ, theo tích chất đường phân giác kết hợp hai tam giác AFO AQF ta suy PA FA AO

PQ =FQ = FO , suy

PA PQ

OA= FO

Kết hợp hai kết ta AP KP

AO= OI , mà ta có KP song song với IO nên theo

định lí Talets ta suy ba điểm A, K, I thẳng hàng

(91)

( )( )

( )

( )

( )

( )

2

2

3

2

DE.DF DE DF EF

DE.DF DE DF EF DE DF EF DK

DE DF DE DF

DE.DF.EF EF

DE.DF DE.DF

4 DE DF

 + − 

+ + + −  

= =

+ +

= −  −

+

Hay ta KD 4DE.DE EF2

2

 − Đẳng thức xẩy DE=DF

đó AB AC= hay tam giác ABC cân A

Bài 69.Cho tam giác ABC với cạnh AB c, BC a, CA b= = = ngoại tiếp đường trịn

tâm I bán kính r Gọi A , B , C1 1 1 tiếp điểm đường tròn I với cạnh BC, CA, AB Các tia AI, AI, CI cắt đường tròn tâm I A’, B’, C’ Đặt

i i i 1 1 1

A B =c , B C =a , C A =b với i 1, 2= Chứng minh

3

2 2 2 1

a b c 216r abc

a b c  , dấu

đẳng thức xẩy nào?

Lời giải

Bổ đề Trong tam giác nhọn ABC ta ln có cosA p p a( )

2 bc

− =

Chứng minh. Xét ta giác nhọn ABC có AD đường phân giác trong, ta có

( ) ( ) ( )

A

2bc.cos 2 bcp p a A 2 bcp p a b c p p a

2

AD cos

b c b c b c bc bc

− − + −

= =  = =

+ + +

Ngoài ta ý đến nhận xét: Trong tam giác ABC sinA B sin A sin B

2

+  +

Trở lại toán.Gọi A, B, C số đo

góc BAC; ABC; ACB, A ; B ; C1 1 1lần lượt số đo

góc B A C ; A B C ; A C B1 1 1 1 1 A ; B ; C2 2 lần

lượt số đo góc B A C ; A B C ; A C B2 2 2 2 2 2 2 2 2 Gọi p S nửa chu vi diện tích tam giác ABC

C2 A2

B1 C1

B2

A1 I

C B

(92)

Dễ dàng tính 1 1 1

2 2

B C C A A B

A ; B ; C

2 2

+ + +

= = = Khi áp dụng nhận

xét ta

( )( )( )

2 1 1 1

2 2 2

3

1 1 1

3

1 1 1

3

1 1 1

B C C A A B

a b c 8r sin A sin B sin C 8r sin sin sin

2 2

r sin B sin C sin C sin A sin A sin B

8r sin B sin C sin C sin A sin A sin B 8r sin A sin B sin C a b c

+ + +

= =

 + + +

= =

Từ ta suy

3

3 2

1 1 1

2 2 1

a b c

a b c 8r sin A sin B sin C

a b c  =

Ta lại có A1 B C; B1 C A; C1 A B

2 2

+ + +

= = = để ý sin A1 sinB C cosA

2

+

= =

Nên ta

3

3 2

1 1 2

1 1

a b c A B C

a b c 8r cos co s co s

2 2

a b c  = Áp dụng bổ đề ta

( ) ( ) ( )

3 3

3 2 2 2 1

p p a p p b p p c

a b c 8r p.S 8r p

8r

bc ca ab abc abc

a b c

− − −

 = =

Để ý ta ln có p 3r , ta

3

2 2 2 1

a b c 216r abc

a b c 

Bất đẳng thức chứng minh Dấu đẳng thức xẩy tam giác ABC

Bài 70. Cho đường tròn tâm O nội tiếp tam giác ABC Các tiếp tuyên với (O) song song với cá cạnh ram giác ABC với sáu điểm M, N, P, Q, R, S cho M,S AB; N,P AC; Q,R BC   Gọi l , l , l1 2 3 đường phân giác xuất phất từ đỉnh A, B, C tam giác AMN, BSR, CPQ Gọi p nửa chu vi tam giác ABC Chứng minh 2 2 2 2

1

1 1 81

l +l +l p

(93)

Gọi l , l , la b c theo thứ tự độ dài đường phân giác xuất phát từ đỉnh A, B, C tam giác ABC Áp dụng công thức đường phân giác cho

các tam giác ABC AMN ta có

a A AB.AC.cos l AB AC =

+

A AM.AN.cos l AM AN = +

Gọi p , p , p1 3lầ lượt nửa chu vi tam giác

AMN, BSR, CPQ

l3 l2 l1 S R Q P N M O C B A

Do NM//BC nên theo định lí Talet ta có AM AN MN AM AN MN p1

AB AC BC AB AC BC p

+ + = = = = + + Suy 1 p p

AB AM ; AC AN

p p = = Do ( ) 1 a 1 1

p p A A

2 AM AN.cos AM.AN.cos

p p p 2 p

l l

p p AM AN p

AM AN p             = = = + +

Hoàn toàn tương tự ta b 2 c 3

2

p p

l l ; l l

p p

= = Do a b c

1 3

l l l 1

p

l l l p p p

 

+ + =  + + 

 

Mà theo tính chất tiếp tuyến cắt ta p p= 1+p2+p3

Và lại có ( 1 2 3)

1

1 1

p p p

p p p

 

+ +  + + 

  ,

a b c

1

l l l

9 l +l +l 

Áp dụng bất đẳng thức Bunhiacopxki ta ( )

2

2 2

a b c

a b c 2

1 3

l l l 1

l l l

l l l l l l

 

 

+ +  + +  + + 

   

   

Với AB c, BC a, CA b= = = , theo công thức đường phân giác tam giác ta có

( )

a

p p a 2bc

l

b c bc

− =

+ Do ( ) ( ) ( )

2

a

4bc

l p p a p p a

b c

= −  −

+ Hồn tồn tương tự ta

cũng có l2bp p b ; l( − ) 2c p p c( − ) Do la2 + + l2b lc2 p p a p b p c( − + − + − )=p2 Suy

2

a b c

2 2

1 3

l l l 1

p

l l l l l l

 

 

+ +   + + 

   

    nên ta

2

2 2

1

1 1

81 p

l l l

 

  + + 

 

Do ta suy 2 2 2 2

1

1 1 81

l +l +l  p

(94)

bàng tiếp góc A, B, C Gọi r bán kính đường trịn nội tiếp tam giác ABC Chứng minh

( OI)(a ) ( OI)(b ) ( OI)(c )

1

2R a b a c+ + + b c a b+ + + c a b c+ + 4r

Lời giải

Gọi AA ; BB ; CC1 1 1 đường phân giác tam giác ABC Dựng EIa vuông góc

với AB E, FIa vng góc với AC F, EIa vng góc với OM M, FIavng góc

với ON N, OP vng góc với AB P Khi ta B AC1 1=BAC MON= (1)

Dễ thấy E F tiếp điểm đường trịn bàng tiếp góc A với AB AC nên ta

được AE AF p; OM PE p c a b; ON p b a c

2 2

+ +

= = = = − = = − = với p a b c + +

=

Khi ta OM a b

ON a c

+ =

+ (2)

Theo tính chất đường phân giác ta có AB1 bc ; AC1 bc

c a a b

= =

+ + Do

1

AB a b

AC a c

+ =

+ (3)

Từ (1), (2) (3) ta AB C1 1∽ OMN nên suy B C1 AB1 ( 2bc)( )

MN = OM= a b a c+ + Suy

( )( ) ( a )( ) ( a)( ) ( )(a )

1

2bc.OI sin MON 2bc.OI sin BAC abcOI 2bc.MN

B C

a b a c a b a c a b a c R a b a c

= = = =

+ + + + + + + +

Do ta OIa R a b a c B C( )( ) 1 abc

+ +

= Hoàn toàn tương tự

( )( ) 1 ( )( ) 1

b c

R b c a b A C R b c c a A B

OI ; OI

abc abc

+ + + +

= =

Do ( )(a ) ( )(b ) ( )(c ) ( )

1 1 1

OI OI OI R

Q A B B C A C

abc

a b a c b c a b c a b c

= + + = + +

(95)

+ Trước hết ta chứng minh Q 2R 

Trường hợp 1.Tam giác ABC khơng tù Gọi giao điểm OA B C1 1 D, ta

1

1 1 OB AC

R.B C =OA.B C 2S

Hoàn toàn tương tự ta

1 1

1 OC BA 1 OB CA

R.C A 2S ; R.A B 2S

Do ta R A B( 1 1+B C1 1+C A1 1)2SABC

F

E N

M

P D C1

B1 A1

Ia

I O

C B

A

Mà lại có SABC abc 4R

= Từ ta Q 2R  • Trường hợp Tam giác ABC tù, khơng tính tổng qt ta giả sử

BAC 90 Khi gọi

2

C C3 điểm đối xứng với C1 qua BC

và AB Từ A B1 1+B C1 1+C A1 1C C2 3 Dựng

2

AH⊥C C Do

ACB 90 C CC2 3 =2ACB nên suy CC1 =CC2 =CC2 CA

C3 C2

B1 A1 C1 H

C B

A

Từ suy C C2 3 =2CC sin ACB 2b sin C3  Do A B1 1+B C1 1+C A1 1 2b sin C

Tương tự ta có A B1 1+B C1 1+C A1 1 2c sin B

Nên ABC

1 1 1

2S

bc bc

A B B C C A b sin C c sin B

2R 2R R

+ +  + = +  Từ ta Q

2R 

Kết hợp hai trường hợp ta Q 2R 

+ Chứng minh Q

4r 

Theo định lí cosin ta 2

1 1 1

(96)

( )( )

( )( )

( )( ) (( ) ( )( ) ) ( )( )

2 2 2 2 2 2

2 1

2 2

2

2

2

2

bc bc b c b c a

B C

a c a b a b a c 2bc

b c c b bc b c b c

c a c a b a b a b a c a b a c a

abc a b c b c

a bc a bc

a b a c a b a c a b a c

a bc ab ac ab ac 2a b c

4 36

4 ab ac

    + − =  + − + + + +         =  − +  − + +  + +  +  + +  + + + + − = −  + + + + + +  +   + +  =           

Do suy B C1 1 2a b c + +

 Tương tự ta C A1 1 2b c a; A B1 1 2a b c

8

+ + + +

 

Từ suy A B1 1 B C1 1 C A1 1 a b c p

+ +

+ +  = Do ta Q Rp

abc 4r  =

Như ta ( OI)(a ) ( OI)(b ) ( OI)(c )

2R a b a c+ + + b c a b+ + + c a b c+ +  4r

Dấu đẳng thức xẩy tam giác ABC

Bài 72. Cho đường trịn tâm O bán kính R dây cung BC(với BC R ) Điểm A di động cung lớn BC điểm D di động cung nhỏ BC Xác định vị trí A

D để 1

DA+DB+DC đạt giá trị nhỏ

Lời giải

Với A, D ta ln có AD 2R Với điểm D cung nhỏ BC ta ln tìm điểm A cung lớn BC cho AD 2R= để 1

AD =2R có giá trị bé

Kẻ DH vng góc với BC H Kẻ đường kính EF vng góc với BC K Khi điểm E, F, K

điểm cố định Do

ABD CHD 90= = DAB DCB= nên

ta ABD∽ CHD.Từ suy

O H K F E D C B A BD DH

DB.DC AD.DH DB.DC 2R.DH

DA= DC  =  =

(97)

Áp dụng bất đẳng thức Cauchy ta 1 2 DB+DC  CB.CD = 2R.DH  2R.EK

Từ ta 1 1 2

DA+DB+DC 2R+ 2R.EK = 2R+BE Dễ thấy

1

2R+BE

hằng số Do 1

DA+DB+DC đạt giá trị nhỏ

1

2R+BE

Dấu xẩy DA trùng với đường kính EF

Nhận xét Có nhiều cách để tìm giá nhị nhỏ T 1 BD CD

= + như:

Ta có DH EK nên EH.BC EK.BC SDBC SEBC, điều dẫn đến

( ) ( )

DB.DC.sin 180 −BDC EB.EC.sin 180 −BEC

Mà ta có BEC=BDC EB EC= nên ta BD.CD BE

Kéo dài BD đoạn DG DC= , ta

BD DC BD DG BE EG BE EC 2EB+ = +  + = + = Do ta 1

BD+DC BD CD+ = BE

Áp dụng định lí Ptoleme cho tứ giác nội tiếp BFCD ta

( )

BD.CF CD.BF+ =BC.DF DB CD BF BC.2R+  Từ suy 1 2BF

BD+DC BD CD+ =R.BC

Bài 73 Cho đường tròn (O; R) điểm I nằm bên đường tròn Gọi AC BD hai dây cung qua I Xác định vị trí AC BD để AB.AD BC.CD

AB.BC DA.CD +

+

đạt giá trị lớn giá trị nhỏ

(98)

Xét hai tam giác IDC IAB có DIC=AIB IDC IAB=

nên ta IDC∽ IAB Từ ta ID IC CD

IA= IB =AD

Chứng minh tương tự ta IAD∽ IBC nên

IA ID AD

IB = IC = BC Từ

ID ID IA AD.DC

IB =IA IB = AB.BC Suy ta

được ID IB AB.BC DA.CD

IB AB.BC

+ = +

hay AB.BC DA.CD

BD IB

AB.BC + =

O

D

C B A

Mặt khác ta lại có IC IC IA: BC.CD IA= IB IB = AB.DA Suy IC IA AB.DA BC.CD

IA AB.DA

+ = +

hay AC AB.DA BC.CD.AI AB.DA

+ =

Từ kết ta

AB.DA BC.CD AI

AC AB.DA

AB.BC DA.CD BD

.IB AB.BC

+ =

+

Chú ý IA AD

IB = BC ta thu

AB.AD BC.CD AC

AB.BC DA.CD BD

+ =

+ Đến ta

+ AB.AD BC.CD AB.BC DA.CD

+

+ đạt giá trị lớn AC lớn đồng thời BD nhỏ

nhất, điều tương đương với AC qua O BD vuông góc với OI

+ AB.AD BC.CD AB.BC DA.CD

+

+ đạt giá trị nhỏ AC nhỏ đồng thời BD lớn

nhất nhất, điều tương đương với BD qua O AC vng góc với OI

Bài 74.Cho đường tròn ( )O điểm P cố định nằm đường trịn ( )O (P khơng

trùng với O) Hai dây cung AC BD thay đổi đường trịn ( )O vng góc với P Tìm vị trí dây cung AC BD cho diện tích tứ giác ABCD có

giá trị lớnnhất, nhỏ

(99)

Gọi E, F, G, H theo thứ tự trung điểm AB, AD, CD, CB Khi dễ dàng chứng minh tứ giác EFGH hình nhật có tâm trung điểm đoạn OP

Ta cần chứng minh AB2+CD2 =AD2+BC2=4R2

Thật AC vng góc với BC nên dễ dàng suy

ra OEA = DGO, suy OG EA AB = =

H G

F E

N M

P K

I O

D

C B

A

Ta có

2

2 2

AB CD

OG DG R

2

   

+ = + =

   

    nên ta

2 2

AB +CD =4R Hoàn toàn

tương tự ta chứng minh AD2+BC2=4R2 Hạ OK vuông góc với AC OI

vng góc với BD Khi ta có ID2 =R2−OI2 Do ta BD2 =4 R( 2−OI2)

Chứng minh tương tự ta có AC2 =4 R( −OK2)

Từ suy AC2+BD2 =4R2−4 OI( +OK2)=8R2−4OP2 khơng đổi Từ ta EF2 FG2 EG2 EG2 1(AC2 BD2)

4

+ =  = + không đổi

Suy hình chữ nhật EFGH hình chữ nhật thay đổi đường tròn cố định có tâm trung điểm OP Để ý SEFGH 1SABCD

2

= Do ta có

( ) ( )

2

2 2

EFGH

EF GH 1

S EF.GH AC BD 8R 4OP

2 8

+

=  = + = −

Dấu xẩy tứ giác EFGH hình vng nên AC BD=

Lại có 2 ( )2 ( 2) ( )2

EFGH

1 1

S EF.FG EF FG EF FG 8R 4OP EF FG

2

 

= =  + − − = − − −

 

+ Để SEFGH đạt giá trị nhỏ EF FG− phải đạt giá trị lớn

Mà ta có AC 2R FG R BD MN với MN qua P Do EF 1MN

2

 nên EF FG R MN

2 −  −

(100)

Vậy SABCD đạt giá trịlớn AC BD= SABCD đạt giá trị nhỏ

AC 2R=

Bài 75.Cho đường tròn ( )O điểm P cố định nằm đường trịn ( )O (P khơng

trùng với O) Hai dây cung AC BD thay đổi đường trịn ( )O vng góc với P Tìm vị trí dây cung AC BD cho chu vi tứ giác ABCD có giá

trị lớn nhất, nhỏ

Lời giải

Chứng minh tương tự ta AB2+CD2+AD2+BC2 =4R2

Đặt m AB BC CD DA= + + + Khi ta

( )

2 2 2

m =AB +BC +CD +AD +2 AB.BC BC.CD CD.DA DA.AB AB.BC BC.AD+ + + + + Mà ta có AB2+BC2+CD2+AD2 có giá trị khơng

đổi.Áp dụng định lí Ptoleme cho tứ giác AEOF nội tiếpđường trịn ta có AE.OF OE.AF R.EF+ = hay ta

được AB.BC CD.DA R.BD

4 + = Tương tự ta

có AB.AD CD.BC R.AC

4 + = Do ta

( )

2R BD AC+ =AB.BC CD.DA AB.AD CD.BC+ + +

H G

F E

N M

P K

I O

D

C B

A

Như việc tìm giá trị lớn nhỏ m tương đương với tìm giá trị lớn nhỏ biểu thức S=AB.CD AD.BC 2R AC BD+ + ( + ) Cũng theo định lí Ptoleme tứ giác ABCD nội tiếp đường trịn ta có AB.CD AD.BC AC.BD+ =

Do suy S=AC.BD 2R AC BD+ ( + )

Ta có ( ) ( )

2

2

AC BD

S AC.BD 2R AC BD 2R AC BD

2 +

= + +  + + không đổi

Dấu xẩy AC BD=

Lại có S AC2 BD2 (AC BD)2 2R AC( BD2) (AC BD)2

2

 

=  + − − + + − −

 

Mà ta có AC 2R; BD MN   AC BD− 2R MN−

Suy S AC2 BD2 (2R MN)2 2R AC( BD2) (2R MN)2

 

  + − − + + − −

(101)

Dấu xẩy AC 2R=

Vậy m AB BC CD DA= + + + đạt giá trị lớn AC BD= đạt giá trị nhỏ

khi AC 2R=

Nhận xét Ta tìm giá trị nhỏ chu vi tứ giác ABCD theo cách khác sau

Kẻ đường kính BE ( )O Hai tam giác vng ABE PAD có ADB AEB= nên đồng dạng với nhau, ta AB.AD BE.PA 2R.PA= = Hoàn toàn tương tự ta BC.CD 2R.AC.= Từ ta

( )

AB.AD CB.CD+ =2R PA PC+ =2R.AC

Từ giả thiết AC⊥BD suy AE CD; AD CE= =

E

O N

M P

B

D

C A

Ta có AB2+CD2=AB2+AE2=4R ; AD2 2+BC2 =CE2+BC2=4R2 Gọi M, N lần

lượt trung điểm AC BD, suy OM⊥AC; ON⊥BD Từ suy

( ) ( )

( )

2 2 2 2

2 2 2

AC BD 4AM 4BN R OM R ON

8R OM ON 8R OP

+ = + = − + −

= − + = −

Đặt d OP= Ta có AC BD2 =16 R( 2−OM2)(R2−ON2) (=16 R4−R d2 +OM ON2 2)

Từ p AB BC CD DA= + + + ta

( ) ( )

( ) ( )

2

2 2 2

2 2

p AB BC CD DA AB BC CD DA AB.AD BC.CD

2 AB.BC AD.CD AB.CD AD.BC

8R 2AC.BD 4R 8R 4d 2AC.BD

= + + + = + + + + +

+ + + +

= + + − +

Thay AC.BD R= 4−R d2 2+OM ON2 2ta

2 2 2 2 2 2

p =8R +8 R −R d +OM ON +4R 8R −4d +4 R −R d +OM ON

Như p2 8R2+8 R4−R d2 +4R 8R2−4d2 +4 R4−R d2 =16 R( 2+R R2−d2)

Nên ta 2

p R +R R −d , dấu xẩy OM.ON 0= tương đương với AC BD đường kính đường tròn ( )O

(102)

Bài 76. Cho tam giác ABC có BC a,CA b,AB c= = = Gọi r r ,r ,ra b c bán kính đường nội tiếp bấn kính đường bàng tiếp góc A, B, C tam

giác ABC

Chứng minh

3 3

a b c

abc a b c

r  r + r + r

Lời giải

Với kí hiệu hình bên ta có AK p a; AH p= − = ,

trong 2p a b c= + + Ta có hai tam giác AKI

AHQ đồng dạng với nhaunên ta

a a

p a rp

r IK AK

r

r HQ AH p p a

= = =  =

Hoàn toàn tương tự ta rb pr ; rc pr

p b p c

= =

− −

Xét hiệu

3 3

a b c

abc a b c

T

r r r r

 

= − + + 

  ta

ra r K Q I C B A ( ) ( ) ( ) ( ) ( ) ( )

3 3

3 3

a b c

3 3

a p a b p b c p c

abc a b c abc

T

r r r r r rp rp rp

1

pabc a p a b p b c p c

rp − − −   = − + + = − − −     =  − − − − − − 

Ta cần chứng minh T 0 pabc a p a− 3( − −) b p b3( − )−c p c3( − )

Thật vậy, vai trị a, b, c nên khơng tính tổng qt ta giả sử

a b c 0   Đặt P pabc a p a=  − 3( − −) b p b3( − )−c p c3( − ) Khi ta có

( ) ( ) ( ) ( )

( ) ( ) ( )

( ) ( ) ( ) ( ) ( ) ( )

( )( ) ( )( ) ( )( )

( )( )( ) ( ) ( )( )

3 3

2 2 3 3 3

2 3

2 2

2

2 2

P a b c abc a b c a b c a b c a b c

a bc b ca c ab a b a c a b c b a b c a c b c

a b c a a c a b c a b b a b c a b c c b c

a c a b a b a b c b c b c a c

c b b a a b b a b c b c a c

= + + − + − − + − − + −

= + + − − − − − − − − −

= − − − + − − − + − − −

= − − + − − + − −

= − − − + − + − −

Dễ thấy với a b c  P 0 nên T 0

Từ suy

3 3

a b c

abc a b c

(103)

Bài 77. Cho tam giác ABC nội tiếp đường tròn ( )O ngoại tiếp đường trịn có bán kính r Gọi O ,R ;1 1 O ,R ; O ,R2 2 3 3 theo thứ tự tâm bán kính đường trịn tiếp xúc ngồi với đường trịn (O) đồng thời tiếp xúc với AB, AC; BC, BA; CA, CB tương ứng Chứng minh R1+R2+R312r

Lời giải

Giả sử đường trịn (O ; R1 1) tiếp xúc ngồi với đường tròn (O; R) D tiếp xúc với hai tia AB, AC M, N Giả sử tia AD cắt đường tròn

(O ; R1 1) điểm thứ hai E Khi ta suy

được OO1 =OD O D R R+ 1 = + 1 Lại OA song

song với EO1 AM2=AN2=AD.AE Từ suy

ra

2

1

AD AD OD

AE OO

AM = = hay

2

1

AD R

R R

AM = + l E

I L K N M O1 P O D C B A

Chứng minh tương tự ta

2

2

1

BD CD R

R R

BM = CN = +

Từ ta thu AD BD CD

AM= BM=CN Mặt khác tứ giác ABDC nội tiếp nên theo định lí Ptoleme ta có AB.CD AC.BD AD.BC+ = Do ta AB.CN AC.BM BC.AM+ =

Đặt BC a; CA b; AB c= = = ta BM AM c; CN AN b AM b= − = − = − Khi từ hệ thức ta AM AN 2bc

b c a = =

+ − Gọi L tiếp điểm AB với đường tròn ( )I

nội tiếp tam giác ABC.Khi O M1 AM

IL = AL hay

1

R AM

r = AL Thay ( )

1

AL b c a

2

= + − 2bc

AM

b c a =

+ − vào tỉ lệ thức ta ( )2

R 4bc

r = b c a+ − Chứng minh tương tự ta

được

( ) ( )

2

2

R

R 4ac 4ab

;

r = c a b+ − r = a b c+ −

Từ ta

( ) ( ) ( )

1

2 2

R R R 4bc 4ac 4ab

r b c a c a b a b c

+ +

= + +

(104)

Theo bất đẳng thức AM – GM ta có

( ) ( ) ( ) ( ) ( ) ( )

2 2

2 2 2

4bc 4ac 4ab a b c

12

b c a c a b a b c b c a c a b a b c

+ + 

+ − + − + − + − + − + −

Chú ý (a b c b c a c a b+ − )( + − )( + − )abc, ta R1 R2 R3 12

r + +

Hay ta R1+R2+R3 12r, dấu xẩy R1 =R2 =R3 a b c= = hay tam giác ABC

Bài 78.Cho tam giác ABC khơng cân có AD BE đường phân giác Chứng minh góc nhọn tạo hai đường thẳng AB DE không vượt qua

A B −

Lời giải Khơng tính tổng qt ta giả sử

tam giác ABC có AB Gọi M giao điểm AB DE, góc nhọn tạo hai đường thẳng AB DE góc BMD

Ta cần chứng minh BMD 1(BAC ABC)

 −

Thật vậy, áp dụng định lí Menelaus cho tam giác ABC với ba điểm M, D, E thẳng hàng ta có

M

E

D C

B

A

DB EC MA MA DC EA

DC EA MB =  MB = DB EC

Để ý AD BE đường phân giác tam giác ABC nên DC AC EA; BA DB = AB EC = BC

Từ ta MA AC AB CA

MB = AB BC = BC Từ suy CM đường phân giác

ngồi đỉnh C tam giác ABC Do 1( )

BMC 180 MCB MBC BAC ABC

2

= − − = −

Do suy BMD CMD 1(BAC ABC)

+ = −

Giả sử BMD 1(BAC ABC)

 − , rõ ràng ta có

( ) ( )

1 1

CMD BAC ABC BMD BAC ABC BMD

2

(105)

Từ ta sin BMD sin 2CMD sin CMD.cosCMD cos CMD

sin CMD sin CMD sin CMD

 = =

Mặt khác áp dụng định lí sin cho tam giác BMD CMD ta

BD sin ABC

sin BMD MD BD sin ABC

sin CMD CD sin MCD CD sin MCD MD

= =

Lai có DB AB sin ACB

CD=AC=sin ABC nên ta

sin BMD sin ACB sin ABC ACB

sin

2 sin CMD=sin ABC sin MCD=

Từ ta suy sinACB cos CMD sin 90( CMD)

2  = −

Nên ta ACB 900 CMD CMD 900 ACB 1(BAC ABC)

2  −   − =2 + , điều vơ lí

Do điều ta giả sử sai Nên ta BMD 1(BAC ABC)

 −

Vậy góc nhọn tạo hai đường thẳng AB DE không vượt qua

A B −

Bài 79 Cho tam giác ABC đường tròn ( )O nội tiếp tam giác ABC tiếp xúc với AB, BC, CA E, D, F Đường bàng tiếp tâm Q tiếp xúc với BC, AB, AC K, H, P Đường thẳng EF cắt tia BO CO M N Đường thẳng HP cắt tia BQ CQ R S Chứng minh DMN= KRS

và SP SR RH

AB= BC=CA

Lời giải

Xét trường hợp điểm M nằm điểm N nằm tam giác ABC (Các trường hợp cịn lại tương tự) Ta có ONC=BMC 90= Gọi Q giao điểm hai tia phân giác

(106)

Khi Q tâm đường trịn bàng tiếp ứng với

góc BAC tam giác ABC Đường tròn ( )Q

tiếp xúc với AB, BC, CA H, K, P Gọi R S giao điểm HP với OB OC Khi dễ thấy CPR CQB 1HAP

2

= = nên tứ

giác CRQP nội tiếp đường tròn nên ta

0

CRQ CPQ 90= = Lại có MBQ 90= nên tứ giác MBRC hình chữ nhật

K O

Q

P S

R H

I N

M F

E

D C

B A

Gọi I trung điểm BC Ta có M R đối xứng với qua I Tương tự N S đối xứng với qua S Từ tính chất tiếp tuyến với đường tròn ta

( )

BH=AH AB p− = − AE BE+ =CD p nửa chu vi tam giác ABC Từ

suy BD CK= Do ta ID IK= hay K F đối xứng với qua I Từ tứ giác MDRK, NDSK, MNRS hình bình hành Do ta

DMN KRS

 = 

+ Lại thấy BKR=BHR=AHP=APH CPS= nên tứ giác RKCP nội tiếp đường tròn Do ta KRS=ACB Hồn tồn tương tự ta KSR=ABC Từ suy hai

tam giác KRS ACB đồng dạng nên ta KS SR KR

AB= BC= AC Mà ta lại có KS PS= KR=HR nên suy PS SR RH

AB= BC=CA

Bài 80 Cho đường tròn (O; R) nội tiếp hình thang ABCD (AB//CD) Gọi E, F, G, H theo thứ tự tiếp điểm đường tròn (O; R) với cạnh AB, BC, CD, DA Trên

cạnh CD lấy điểm M nằm hai điểmD C cho chân đường vng góc kẻ từ M đến DO điểm K nằm đường tròn (O; R) Đường thẳng HK cắt đường tròn

(O; R) điểm thứ hai T.Tính tỷ số EB

EA biết

4R

AB ;BC 3R

3

= = chứng minh

rằng MT MG=

(107)

+ Do tứ giác ABCD hình thang nên ta

CDA DAB 180+ = Do DO, AO theo thứ tự đường phân giác góc CDA

DAB Do ta

ODA OAD 90+ = , từ ta suy AOD 90= nên ta giác

AOD vuông O

O

M K

T H

G

F E

D C

B A

Hoàn toàn tương tự tam giác BOC vuông O Nên theo hệ thức lượng tam giác vuông ta HA.HD FB.FC R= = 2

Mặt khác ta có HA EA; BE FB; CF CG; DG HD= = = = nên EA.GD EB.GC= , từ ta lại suy EB GD

EA= GC Đặt

EB GD

k

EA = GC= Ta có OE vng góc với AB, OG vng góc

với CD nên ba điểm E, O, G thẳng hàng Từ ta EG 2R=

( )2

2

ED =BC − GC EB−

Ta lại có EB k EB kAB (4kR )

EB EA+ =k 1+  = k 1+ =3 k 1+ nên ( )

4kR BF

3 k =

+

Từ ta

(4kR )

CF BC BF 3R

3 k

= − = −

+ nên ( )

4kR

CG 3R

3 k

= −

+

Thay kết vào ED2 =BC2−(GC EB− )2 ta

( ) (( ))

2

2

2

k

4kR

4R 9R 3R k

11

3 k 9 k 1

  + − +

= − −   =  =

+ +

 

 

Vậy EB

EA 11

− +

=

+ Ta có DH DG= nên tam giác DGH cân D Mà OD đường phân giác góc

HDG nên suy GH vng góc với DO Lại có MK vng góc với DO nên MK song

song với HG Từ ta suy KMG HGC= , mà GC tiếp tuyến đường tròn

( )O góc HTG góc nội tiếp đường trịn ( )O Do ta HTG HGC= suy KMG=HTG nên tứ giác KTGM nội tiếp đường trịn Lại có OKM OGM 90= =

(108)

K, G, M, T thuộc đường trịn đường kính MO Do MIO 90= nên OT vng góc

với MT Do MT tiếp tuyến đường trịn ( )O Ta có MT, MG tiếp tuyến đường tròn ( )O nên MT MG=

Bài 81. Cho tam giác ABC có B, C cố định điểm A thay đổi cho tam giác ABC nhọn không cân Gọi D trung điểm BC E, F tương ứng hình chiếu D

trên AC, AB Gọi O tâm đường tròn ngoại tiếp tam giác ABC Đường thẳng EF cắt AO BC theo thứ tự M N Các tiếp tuyếntại E, F đường tròn ngoại tiếp tam giác AEF cắt T Chứng minh đường tròn ngoại tiếp tam giác AMN qua điểm cố định T thuộc đường thẳng cố định

Lời giải

G A

B D C

E F

O

M

N

K

P

Q

T

I J

+ Chứng minh đường trịn ngoại tiếp tam giác AMN ln quamột điểm cố định

Giả sử tam giác ABC có AB AC , trường hợp cịn lại chứng minh hồn tồn tương tự Khi điểm N nằm tia đối tia BC Tứ giác AEDF có

0

AED AFE 180+ = nên nội tiếp đường trịn đường kính AD, suy

MEA FEA FDA= = Do O tâm đường trịn ngoại tiếp tam giác ABC nên ta có

0

OAC 90= −ABC Từ có biến đổi góc

0

(109)

Từ suy tứ giác AMDN nội tiếp đường tròn Mà BC cố định nên điểm D cố định Vậy đường tròn ngoại tiếp tam giác AMN qua điểm D cố định

+ Chứng minh T thuộc đường thẳng cố định.

Gọi K tâm đường trịn ngoại tiếp tam giác AEF, K trung điểm AD Gọi giao điểm AE với DF P giao điểm AF với DE Q.Khi tam giác APQ có hai đường cao QE PF cắt D nên D trực tâm Do AD vng góc với PQ G Tam giác AED vng E có EK đường trung tuyến nên tam giác AKE DKE cân Mặt khác TE tiếp tuyến đường tròn ngoại tiếp tam

giác AEF nên ta có TEQ TED EAD EQG= = = nên ET qua trung điểm PQ, tương tự FT qua trung điểm PQ Từ suy T trung điểm PQ Dễ thấy tứ giác PEFQ nội tiếp đường trịn đường kính PQ có tâm T Gọi I, J giao điểm BC với đường tròn ( )T Áp dụng định lý Menelaus cho tam giác ABC với ba điểm Q, D, E P, D, F thẳng hàng ta có BF AP CD

AF CP BD =

BQ AE CD

AQ CE BD =

Từ ta

2

CD BF.AP.AE.BQ

AF.CP.AQ.CE

BD = hay

2

CD AF.CP.AQ.CE

BF.AP.AE.BQ

BD =

Mặt khác tứ giác QFEP nội tiếp đường tròn nên ta có AF.AQ AE.AP= Mặt khác tứ giác IEJP JFIQ nội tiếp đường tròn ( )T nên ta lại có CP.CE=CJ.CI

BF.BQ=BJ.CJ Từ ta suy

2

CD CJ.CI

BJ.CJ

BD = Mà CD BD= nên ta có

( )( ) ( )( ) 2 2

BI.BJ=CI.CJ ID DB ID DB− + = JD DC JD DC− + ID −BD =JD −CD

Mà lại CD BD= nên suy ID=JD hay D trung điểm IJ Từ dẫn đến TD

vng góc với BC D, suy T thuộc đường trung trực BC Mà BC cố định nên đường trung trực BC cố định Vậy T thuộc đường thẳng cố định A thay đổi

(110)

KM KA; LN LA= = Chứng minh tâm đường tròn ngoại tiếp tam giác MND thuộc đường thẳng cố định P di động đoạn thẳng AP

Lời giải

I

L

K Q

P

N M

F E

D

T

S

R

C B

A

Qua K kẻ đường thẳng vng góc với CP cắt AD N Khi P trực tâm tam giác RKC Suy KP vng góc với RC Từ ta KCP=KRD Chú ý tam giác AKC vng A có AD đường cao nên ta có KM2 =KA2 =KD.KC nên KM tiếp xúc với đường tròn ngoại tiếp tam giác DMC Suy KMD MCD KCP= = Từ ta KRD KMD= , nên bốn điểm K, D, M, R nằm đường trịn Do ta

được

RMK=RDK=90 hay RM vng góc với MK Lại có PCD∽ KRD nên ta PD CD

KD =RD Hai tam giác KAC LAB vng

góc A nên ta AD2 =KD.DC BD.LD= nên KD ND

BD = CD Do

PD LD

CD =RD, nên

LDR PDB

 ∽  , suy LRD=DBP=LBN

Mặt khác ta có LN2=LA2=LD.LB nên LN tiếp xúc với đường trịn ngoại tiếp tam

(111)

( ) ( )

( ) ( ) ( ) ( )

2 2 2 2

2 2 2 2

IN IM RN RM RK KM RL LN

RK RL AL AK AK AL AL AK

− = − = − − −

= − + − = − + − =

Từ ta IM2 =IN2 hay ta IM IN=

Giả sử đường tròn ngoại tiếp tam giác MND cắt BC Q Ta chứng minh Q điểm cố định tâm đường ngoại tiếp tam giác DMN thuộc đường trung trực đoạn thẳng AQ

Thật vậy, gọi giao điểm đường tròn ngoại tiếp tam giác DMN với IM, INlần lượt E, F khác M, N Khi ta có EM FN= Mà ta có KM2 =KA2 =KD.KC nên

MCK KMD EQD= = nên suy EQ song song với PC Tương tự ta có FQ song song với BP Ta có biến đổi tỉ số sau

QK QK QC QB KE QC FN EK QC KE.KM LA QC

QL QC QB QL EM QB FL FL QB KA LF.LN QB

LA KD.KQ QC QK LA.KD QC

KA LD.LQ QB QL KA.LD QB

= = = =

= =

Từ LA.KD QC

KA.LD QB = nên

LA.KD QB

KA.LD =QC Dẫn đến

2 2

2 2

QB KD LA KD LB

KC LD

QC =KA LD = (1)

Vẽ đường phân giác AJ tam giác ABC, dễ thấy KAJ LAJ=

Khi ta có ABK ACL

S BK

S = CL

ABK ACL

S AB.AK

S = AC.AL nên suy

BK AB.AK

CL =AC.AL

Hoàn toàn tương tự ta BL AB.AL

CK =AC.AK Do ta

2

AB BK.BL

CK.CL

AC = (2)

Mặt khác ta có DB.DL DP.DR DP.DR= = nên DK DB DK DB BK

DL DC DL DC CL

= = =

− (3)

Kết hợp kết (1), (2), (3) ta

2

2

QB AB

QC = AC hay

QB AB

QC=AC, suy AQ

đường phân giác tam giác ABC, hay điểm Q J trùng Điều có nghĩa Q điểm cố định Từ theo suy luận ban đầu ta tâm đường trịn ngoại tiếp tam giác MND ln thuộc đường thẳng cố định

(112)

lượt hình chiếu H CB CA Gọi U, L, Q tâm đường tròn bàng tiếp đỉnh H tam giác AHE, BHF, HEF Chứng minh C tâm đường tròn nội tiếp tam giác QUL

Lời giải

Q

J

T Z

U Y

X

F

E

L

H O

C

B A

Gọi giao điểm LF, UE với HQ X, Y Gọi Z hình chiếu Q HE T hình chiếu L BF Để ý đến tam giác vuông cân HYE HZQ ta có

EY YH YH HE HE CA

EY QY+ = YH QY+ =QH= 2HZ =HE HF EF+ + = AB BC CA+ +

Lại có XH XF HF ( HF ) AC

XH LX+ =2XF LF+ = 2HF FT+ =2HF+ BF BH AB+ − = AB BC CA+ +

Từ hai kết ta EY XH

EY QY+ =XH LX+ hay ta

EY XH

QY= XL Chứng minh

hoàn toàn tương tự ta thu XF YH

XQ = YU Chú ý YE=YH XF=XH

nên ta dễ dàng suy 1

YQ.XQ = XL.YU hay YQ.XQ XL.YU= , điều dẫn đến

hai tam giác vuông YQU XLQ đồng dạng với Từ suy UQL 90= 0 Ta

thấy QU UY YH HE CA

(113)

nhau Gọi J tâm đường tròn nội tiếptam giác ABC Ta thấy hai tam giác HEU LFH đồng dạng với nên ta có CE.CF HE.HF EU.FL= = Mà ta có

0

CFL=UEC 135= nên hai tam giác CFL UCE đồng dạng với Để ý ta lại có hai tam giác EUH AJB đồng dạng với Vậy nên ta

CU UE UE IA

CL = CF =EH = IB Mặt khác ta có

0 0

UCL 90= +UCE LCF 90+ = +45 =135 =AJB

nên hai tam giác CUL JAB đồng dạng với Suy C tâm đường tròn nội tiếp tam giác QUL

Bài 84.Cho tứ giác ABCD khơng phải hình thang nội tiếp đường tâm O Hai

đường chéo AC BD cắt H Gọi giao điểm AB CD, AD BC E, F Chứng minh H hai trực tâm H ; H3 4 hai tam giác FAB, FCD nằm đường thẳng

Lời giải Chứng minh H hai trực tâm H ; H3 4 hai tam giác FAB, FCD nằm đường thẳng Vẽ hai đường cao CC ; DD1 1 cắt trực tâm H4 tam giác FCD Giả sử đường cao hạ từ A tam giác FAB cắt HH4 H’ tac cần chứng minh BH’ vng góc với AF để suy H’ trực tâm tam giác FAB hay H’ trùng với H3

Thật vậy, gọi M giao điểm AC DD1, N

giao điểm CC1 BD Khi dễ thấy tứ giác

1

CDC D nội tiếp đường tròn Suy ta

1 1

AC D =DCD

D

H4 H3

D1 C1

N

M F

H

C B A

Mà ta lại có DCD1=FAB nên ta AC D1 1 =FAB Từ suy ta AB song song với

1

C D Ta có NDC1=MCD1 C DH1 4 =D CH1 4 nên đặt NDC1 =MCD1=

1 4

(114)

1 1

DC N; DC H ; CD M; CD H ta

1

C N tan

C H tan

 

=

1

D M tan

D H tan

 

= Do ta

1

1 4

C N D M

C H = D H , suy MN song song với C D1 Mà ta có AB song song với C D1 nên

ta AB song song với MN Lại có AH’ song song với MH4 Áp dụng định lí

Talets ta có

4

HH' HA HA HB

;

HH =HM HM=HN nên ta 4

HH' HB

HH =HN Từ theo định lí

Talets đảo ta có BH’ song song với NH4, BH’ vng góc với FD Điều có

nghĩa H’ trùng với H3 nên ta H hai trực tâm H ; H3 4 nằm đường thẳng

Bài 85.Cho tứ giác ABCD khơng phải hình thang nội tiếp đường tâm O Hai đường chéo AC BD cắt H Gọi giao điểm AB CD, AD BC E, F Chứng minh F hai trực tâm H ; H1 2 hai tam giác HAB, HCD nằm đường thẳng

Lời giải

Chứng minh F hai trực tâm H ; H1 2 hai tam giác HAB, HCD nằm đường thẳng Vẽ hai đường cao CC ; DD1 1 cắt trực

tâm H2 tam giác HCD Giả sử đường cao AA1

hạ từ A tam giác HAB cắt FH2 H’ ta cần

chứng minh BH’ vng góc với AH để suy H’ trực tâm tam giác HAB hay H’ trùng với H1

Thật vậy, dễ thấy tứ giác CDC D1 1 nội tiếp đường

trịn nên ta có DCD1=BC D1 1 Mà có DCD1 =C BA1 nên BC D1 1=C BA1 , suy AB C D1 song song

với

F

D

A

B

C H

M N

C1

D1 H2

H1

(115)

Đặt NDC1=MCD1 =; C DH1 2 =D CH1 2 = Khi áp dụng tỉ số lượng giác cho

tam giác vuông ta 1 2

C N D M tan

C H D H tan

 

= = Do ta MN song song với

1

C D , nên ta suy MN AB song song với Từ lập luận tương tự ý a ta thu H’ trực tâm tam giác HAB hay hai điểm H’ H1 trùng Từ suy F hai trực tâm H ; H1 nằm đường thẳng

Bài 86 Cho đường tròn ( )O với dây BC khơng phải đường kính Gọi I trung điểm BC điểm A di động cung lớn BC Gọi ( )I1 đường tròn qua I tiếp

xúc với AB B ( )I2 đường tròn qua I tiếp xúc với AC C Hai đường tròn

( )I1 ( )I2 cắt D khác I Chứng minh đường tròn ngoại tiếp tam giác

ADI qua điểm cố định khác I

Lời giải

Trước hết ta phát biểu không chứng minh toán phụ

Bài toán phụ Cho tam giác ABC

nội tiếp đường tròn ( )O có AM là đường trung tuyến, AI đường phân giác Tiếp tuyến đường tròn ( )O B C cắt D Chứng minh AD đối xứng với đường thẳng AM qua qua đường thẳng AI

Bài toán phụ 2.Cho tam giác ABC có đường trung tuyến AI đường phân giác AD Gọi M điểm trên cạnh BC Chứng minh AM đối xứng với AI qua AD

M J

T F

I2 I1

K

E

O

I

D

C B

(116)

chỉ

2

MB AB

MC= AC

Trở lại toán.Để ý BA tiếp tuyến B đường ( )I1 CA tiếp tuyến C đường tròn ( )I2 nên ta

0

BDC=BDI CDI+ =ABC ACB 180+ = −BAC

Từ đósuy tứ giác ABDC nội tiếp đường trịn, ta có IDC=ACB=BDA Điều dẫn đến đường thẳng DA đốixứng với đường thẳng đường trung tuyến DI qua

đường phân giác hạtừ D tam giác DBC Từđógọi T giao điểm hai tiếptuyến B, C củađường tròn ( )O J giao điểmcủa AD với BC theo tốn phụ DA qua điểm T theo toán phụ ta có

2

DB BJ

CJ

DC = Do ta có tam giác TBD đồng dạng với tam giác TAB tam giác TCD đồng dạng với tam giác TAC,

đó suy BD TB BA =TA

CD TC

CA=TA Chú ý ta có TB TC= nên ta suy

BD CD

BA =CA

hay DB AB

DC =AC Đếnđây ta lại có

2

AB BJ

CJ

AC = nên theo tốn phụ hai ta lại có AD đối

xứngvớiđường trung tuyến AI qua đường phân giác hạtừ A tam giác ABC

Điều dẫn đến BID IDC ICD ACB BAD ACB IAC= + = + = + =BIA nên suy IB

đường phân giác góc AID Từ suy IO phân giác ngồi góc AID Gọi M giao điểm thứ gai IB với đường ngoại tiếp tam giác AID, M nằm đường trung trực đoạn thẳng AD Mà ta có điểm O thuộc trung trựccủa đoạn thẳng AD Kếthợpvới IO phân giác ngồi góc AID ta suy O nằm đường tròn ngoại tiếp tam giác AID.Do đường tròn ngoại tiếp tam giác AID qua điểm O cố định

Bài 87 Cho hai đường tròn ( )O1 ( )O2 tiếp xúc với T Một đường thẳng cắt đường tròn ( )O1 hai điểm phân biệt A, B tiếp xúc với đường tròn

( )O2 X (B nằm A X) Đường thẳng XT cắt đường tròn ( )O1 S khác T

(117)

đoạn thẳng CY ST không cắt Gọi I giao điểm đường thẳng XY SC Chứng minh SA SI=

Lời giải

I O2 O1

S

T

Y X

C B

A

Do hai đường tròn ( )O1 ( )O2 tiếp xúc với T hai đường thẳng

1

O O , SX cắt T nên ta có TO S1 =TO X2 Từ suy TAS TYX= =TYI Mặt khác do tứ giác ABTS nội tiếp đường tròn TAS=TCI nên suy TCI TYI= , tứ giác CTIY nội tiếp đường trịn Ta có O ST1 =O TS1 =XTO2 =TXO2 nên O S1 song

song với O X2 Mặt khác ta có O X2 vng góc với AB nên O S1 vng góc với BA Điều dẫn đến O S1 qua trung điểm AB nên tam giác SAB cân S, suy

SA SB= Mặt khác ý đến tứ giác CTIY nội tiếp đường trịn ta có

TIS TIC TYC TXI= = = , mà ta lại có TSI=XSI nên suy tam giác STI tam giác SIX

đồng dạng với nhau, ta SI2=ST.SX Để ý ta có

TBS TAS TCI= = =TYI=BXS, mà ta lại có BST=BSX nên tam giác STB đồng dạng với tam giác SBX , SB2 =ST.SX Từ kết ta SA2=SI2 hay

SA SI=

Bài 88.Cho tam giác ABC nhọn nội tiếp đường tròn ( )O ngoại tiếp đường tròn ( )I

(118)

điểmthứ hai ID với đường tròn ngoại tiếp tam giác ODH Chứng minh ba điểm A, H, K thẳng hàng

Lời giải Vẽ đường kính PQ đường trịn

( )O gọi giao điểm AI BC F Do I tâm đường tròn nội tiếp

tam giác ABC P điểm cung nhỏ BC đường tròn

( )O nên AI qua điểm P Ta có

IBP=IBA CAP+ =IBA IAB+ =BIP

nên tam giác PBI cân P, P tâm đường tròn ngoại tiếp tam

giác BIC

N P

O Q H T

D F

J K

M

I

C B

A

Điều dẫn đến MI vng góc với IP Do J điểm đối xứng với I qua O nên tứ giác PIQJ hình bình hành Do IQ song song với PI nên ta lại suy MI vng góc với QJ Để ý ta thấy IMF APQ= nên hai tam giác vuông MIF PAQ đồng dạng với nên ta có IF IM

AQ =AP hay ta IF.AP IM.AQ= Lại thấy BI phân giác

của tam giác ABF nên ta có FI BF

AI =BA Mặt khác FBP=CAP=BAP nên tam giác

BPF tam giác APB đồng dạng với nhau, suy BF PB PI

BA= PA=PA Từ ta có

FI PI

AI =PA hay FI.PA IA.IP= Kết hợp hai kết lại ta thu IM.AQ IA.IP= hay

AQ PI

AI =MI nên hai tam giác vuông IAQ MIP đồng dạng Điều dẫn đến MPI AQI= Chú ý đến tứ giác PIQJ hình bình hành IPJ IQJ= AQJ 90= Trong tam giác MPJ có MPJ MPI IPJ AOI IQJ AQJ 90= + = + = = 0 nên tam giác MPJ

(119)

Gọi giao điểm thứ hai MA với đường tròn ( )O T Do MI tiếp tuyến đường trịn ngoại tiếp tam giác BIC nên ta có MI2 =MB.MC Mà ta lại có tứ giác

ATBC nội tiếp đường trịn ( )O nên ta lại có MB.MC MT.MA= Do suy

2

MI =MT.MA, mà tam giác MIA vuông I nên IT trở thành đường cao hay IT vng góc với AI Gọi N giao điểm thứ hai TI với đường trịn ( )O , AN đường kính cỉa đường trịn, mà ta có AQJ 90= nên ba điểm Q, J, I thẳng hàng

Do AP song song với QA AQ song song với PN Điều dẫn đến

ITP NTP ANQ ATQ= = = , mà AQT APT= nên tam giác AQT tam giác PIT đồng dạng với Từ suy QA PI QJ

TA =TI = TI nên ta lại hai tam giác vuông AQJ

và ATI đồng dạng với Từ suy AJQ AIT AMI= = hay JAI IMA= nên MA

vng góc với AJ Từ dẫn đến điểm A, J, P, H, M nằm đường trịn đường kính AJ Do ta suy đươc KHO KDO APJ= = =AHJ nên ba điểm H, A, K thẳng hàng

Bài 89.Cho tứ giác lồi ABCDnội tiếp đường trịn tâm Ovà có cặp cạnh đối không song song Gọi M, Ntương ứng giao điểm đường thẳng AB CD, AD

BC Gọi P, Q, S, T tương ứng giao điểm đường phân giác cặp góc

MAN MBN, MBN MCN, MCN MDN, MDN MAN Giả sử bốn điểm

P, Q, S, T đôi phân biệt.Gọi Ilà tâm đường tròn qua bốn điểm P, Q, S Gọi

E giao điểm đường chéo ACvà BD Chứng minh ba điểm E, O, Ithẳng

hàng

(120)

I

E

P

Q S T

O

N M

D

C

B

A

Gọi A, B, C, D góc tứ giác ABCD Ta giả sử MN nằm phía với B đường thẳng AC hình vẽ (Các trường hợp khác chứng minh tương tự)

Xét tam giác ABP ta có

( )

( )

0

0

1

TPQ APB 180 PAB PBA 180 MAN MBN NBA

2

1 1

180 C B C 90 C D

2 2

 

= = − + = − + + 

 

 

= − + + = − +

 

Xét tam giác CDS ta có

( ) ( )

0 1

QST CSD 180 SDC SCD 180 D C A 90 C D

2 2

 

= = − + = − + + = − +

 

So sánh hai đẳng thức trên, ta TPQ QST= hay tứ giác PQTS nội tiếp Ở phần này, vấn đề khó hẳn

việc xác định ví trị điểm E, I khiến ta khó định hướng việc chứng minh điểm thẳng hàng

Tuy nhiên, ta ý E giao hai đường chéo tứ giác ABCD điểm M, N OE vng góc với

MN

K A B C

D

M

N O

(121)

Ta chứng minh điều qua bổ đề: Cho tứ giác ABCD nội tiếp đường trịn ( )O có

M, N giao điểm cặp cạnh đối AB, CD AD, BC Gọi E giao điểm hai đường chéo Khi ta có OE vng góc với MN

Thật vậy, gọi K giao điểm khác E đường tròn ngoại tiếp tam giác ABE, CDE Do tứ giác ABCD nội tiếp đường tròn nên ACD 1AOD

2

= ABD 1AOD

= , suy

ACD ABD+ =AOD Ta có DKE 180= 0−DCE 180= 0−DCA AKE 180= 0−DBA

Từ kết hợp kết ta DKA=3600−DKE AKE− =DOA nên tứ giác

DOKA nội tiếp đường trịn Mặt khác ta có

BKC=BKE CKE EAB EDC+ = + =BOC nên tứ giác OKBC nội tiếp đường trịn Ta có K E giao điểm hai đường tròn ngoại tiếp tam giác AEB CDE, A B hai giao điểm đường tròn ( )O đường tròn ngoại tiếp tam giác ABE, C D là

hai giao điểm đường tròn ( )O đường trịn ngoại tiếp tam giác CDE Từ theo mộ tốn phụ quen thuộc ta có ba điểm K, E, M thẳng hàng Hai tứ giác DOKA OKBC nội tiếp đường trịn nên suy K giao điểm thứ hai khác O hai đường tròn ngoại tiếp tam giác OBC OAD Từ suy luận tương tự ta cũn điểm O, K, N thẳng hàng Mặt khác cách xét góc nội tiếp

trong tứ giác nội tiếp ta có

MKN MKB NKB EAB OCB EDC OBC EKC OKC MKO= + = + = + = + = Hơn hai góc bù nên góc 90 hay ME vng góc với ON Chứng minh tương tự ta có NE vng góc với OM Từ suy E trực tâm tam giác OMN Từ ta OE vng góc với MN Bổ đề chứng minh

Đến đây, cần chứng minh thêm OI vng góc với MN tốn hồn tất có điểm O, I, E nằm đường thẳng vng góc với MN Do O, I tâm đường tròn ngoại tiếp tứ giác ABCD PQTS nên ý tưởng tự nhiên ta chứng minh MN trục đẳng phương hai đường

(122)

Tuy nhiên, thực điều không q khó! Theo xác định điểm Q, T tâm đường tròn bàng tiếp tam giác BCM ADM nên chúng phải nằm phân giác ngồi góc AMB hay M, Q, T thẳng hàng Hơn nữa, tâm đường tròn bàng tiếp nên

0 BCM BAD

MQB 90 90 BAT

2

= − = − =

Từ ta tứ giác ABQT nội tiếp đường tròn Suy MA.MB MQ.MT=

hay M có phương tích đến hai đường tròn ( )O ( )I Hồn tồn tương tự với điểm N Từ suy MN trục đẳng phương hai đường tròn ( )O ( )I

và theo lập luận ta dàng có điều phải chứng minh

Bài 90.Cho tam giác ABC nội tiếp đường trịn ( )O có AB AC I tâm đường tròn nội tiếp tam giác Gọi D, E giao điểm tia AI với BC, đường tròn ( )O

Đường thẳng qua I vng góc với AI cắt BC K KA, KE cắt lại đường tròn ( )O

theo thứ tự M, N Các tia ND, NI cắt lại đường tròn ( )O Q, P Chứng minh PM PQ=

Lời giải

K

O I

N

M Q

P

E

D C B

A

Do I tâm đường tròn nội tiếp nên ta có EIB IAB IBA 1(BAC ABC)

= + = + Mặt khác

(123)

góc BAC, ta có IBE IBE CBE 1(ABC BAC)

= + = + Do ta có BIE EBI= hay

tam giác BEI cân E Chứng minh tương tự ta EB EC EI= = nên E tâm

đường tròn ngoại tiếp tam giác BIC Do KI vng góc với AI nên KI trở thành tiếp tuyến I đường tròn ngoại tiếp tam giác BIC Đồng thời KBC trở thành cát tuyến đường tròn ngoại tiếp tam giác BIC Từ dễ thấy hai tam giác KBI KIC đồng dạng với nên ta KI2 =KB.KC Để ý ta có tứ giác BCEN nội tiếp đường

trịn ( )O nên ta lại có KB.KC KN.KE= Do ta KI2 =KN.KE hay ta có

KI KN

KE= KI nên tam giác KIE đồng dạng với tam giác KNI Điều dẫn đến

0

NKI=KIE 90= hay tam giác INE vng N

Ta có PNE=900 nên PE đường kính đường trịn ( )O Khi hai tam giác BDE

và ABE có góc AEB chung BAD CAE CBE= = nên hai tam giác đồng dạng với Từ ta suy BE DE

AE = BE hay

2

BE =AE.DE Để ý tam giác KIE vuông I có đường cao IN nên ta có BE2 =IE2 =EN.EK, ta suy

EA.ED EN.EK= Điều dẫn đến tứ giác ADNK nội tiếp đường trịn Từ suy

MAE=DNE nên hai cung nhỏ ME QE đường tròn ( )O có số đo Do hai cung nhỏ PM PQ đường trịn ( )O có số đo Đến ta suy PM PQ=

Bài 91.Cho tam giác ABC nhọn có đường phân giác AD, BE, CF cắt O Chứng minh rẳng bán kính đường tròn nội tiếp tam giác AOF, BOD, COE tam giác ABC tam giác

(124)

Gọi O ; O ; O1 2 3 tâm đường tròn nội tiếp tam giác AOF, BOD, COE Giả sử

1

OO ; OO ; OO cắtAB, BC, CA lại

M, N, P

Do bán kính đường trịn nội tiếp tam giác AOF, BOD, COE nên ta O O1 2 song song

với AD; O O1 song song với CE O O2

song song với BE

O3 O2

O1

K

H

O P

F

D

E

N M

C B

A

Mặt khác ta có MO1 NO2 PO3

MO = NO = OP ta O O1 song song với MN; O O1

song song với MP O O2 3 song song vớiCF Từ ta MN song song với AD, NP song song với BE MP song song với CF Gọi giao điểm MO NP H, giao điểm BO MN K Ta có MO MK

OH = NK , mặt khác ta có

MK KN

AO = DO nên ta

được MK AO

KN = DO Do ta

MO AO

HO = DO Theo tính chất đường phân giác

tam giác ta có AO AB AC AB AC

DO BD DC BC

+

= = =

Mặt khác ta có MNO NOD; PNO= =BON mà BON=NOD nên ta

MNO PNO= , suy No tia phân giác MNP Tương tự ta MO phân

giác NMP nên O tâm đường tròn nội tiếp tam giác tam giác MNP

Theo tính chất đường phân giác tam giác ta có MO MN MP MN MP

HO HN HP NP

+

= = =

Từ ta AB AC MN MP

BC NP

+ +

= nên ta AB AC BC MN MP NP

BC NP

+ + + +

=

Hay NP MN MP NP

BC AB AC BC

+ + =

+ + Tương tự ta

MN MP MN MP NP

AB AC AB AC BC

+ +

= =

+ + Do ta

được NP MN MP

(125)

Bài 92 Cho tam giác ABC có AB c; BC a; CA b= = = Chứng minh tam giác ABC vuông B C đẳng thức sau xẩy ra:

a) tanA a

2 = b c+ b)

2 A b c

tan

2 b c

− =

+

Lời giải

+ Trước hết ta chứng minh Nếu tam giác ABC vuông B C ta ln có tanA a

2 = b c+

2 A b c

tan

2 b c

− =

+

Thật vậy, xét tam giác ABC vuông B(trường hợp tam giác ABC vuông C chứng minh tương tự)

Khi gọi AD đường phân giác tam giác ABC

Trong tam giác BAD vng B có tanA BD =BA

Theo tính chất đường phân giác tam giác ta có

BD CD BD CD BC a

AB AC AB AC AB AC b c

+

= = = =

+ + +

K F

E

D C

B A

Do ta tanA a

2 = b c+ Từ

A a

tan

2 = b c+ ( ) ( )

2 2

2

2

A a b c b c

tan

2 b c b c b c

− −

= = =

+

+ +

Do ta tanA a = b c+

2 A b c

tan

2 b c

− =

+

+ Ta chứng minh: Nếu có tanA a

2 = b c+

2 A b c

tan

2 b c

− =

+ tam giác ABC vng

tại B C

Thật vậy, gọi AD đường phân giác tam giác ABC Trên tia đối tia AB lấy điểm E cho AE AC b= =

Khi ta có BAD=DAC=ACE=AEC nên ta AD//EC Kẻ BK vng góc với BE cắt đường thẳng EC K

Ta có tanA tan E BK BK = = BE = b c+

(126)

a) Nếu có tanA a

2 = b c+ từ

A BK BK

tan tan E

2 = = BE =b c+ ta BK a BC= = K

trùng với điểm C Điều có nghĩa tam giác ABC vuông B Vậy tanA a

2 =b c+ tam giác ABC vuông B C

b) Nếutan2 A b c

2 b c

− =

+ , giả sử b c kết hợp với

A BK BK

tan tan E

2 = = BE = b c+ ta

thu

( ) ( ( )( ) )

2

2 2

2

b c b c BK

BK b c

b c b c

− +

=  = −

+ + Mặt khác ta có

2 2

AK =BK +c nên

ta AK2=b2=AC2 Từ suy K trùng với C Điều có nghĩa ABC 90= hay tam giác ABC vuông B

Vậy tan2 A b c

2 b c

− =

+ tam giác ABC vng B C

Bài 93 Cho tam giác ABC có G trọng tâm I giao điểm ba đường phân giác Chứng minh GI song song với BC tan tanB C

2 =3

Lời giải

G I E D M

B C H

A

Hạ AH⊥BC, GD⊥BC, IE⊥BC Do ta GDsong song với IE

Gọi độ dài cạnh tam giác ABC a, b, c đặt p a b c + +

= Gọi r khoảng

cách từ I đến cạnh tam giác ABC Khi ta chứng minh

BE p b; CE p c= − = − Ta có ( )

( )( )( )

2

r p a p

B C IE IE r r

tan tan

2 BE CE p b p c p p a p b p c

= = =

− − − − −

(127)

Ta ( )

( )( )( ) ( )

2

2

a

r p a p S.r p a p.r r.a

B C r.a 2r

tan tan 1

2 p p a p b p c S S S h

− − −

= = = = − = −

− − −

+ Ta chứng minh tan tanB C

2 = GI song song với BC

Thật vậy, tan tanB C

2 = ta suy a

2.r IE

1

h AH

− =  =

Gọi M trung điểm BC, ta có MG

MA=

Do ta MG GD

MA=AH = 3, suy

IE DG

IE GD

AH= AH =

Suy tứ giác GIED hình bình hành Do đóta GI//BC + Ta chứng minh GIsong song với BC tan tanB C

2 =3

Thật vậy, GI song song với BC ta có IE GD= Theo định lí Thales ta có

MG GD IE

MA=AH=Ah =3 Do ta a 2.r 1

h

− = hay tan tanB C 2 =

Vậy toán chứng minh

Bài 94.Cho tam giác nhọn ABC Dựng bên tam giác ABC tam giác cân DAC,

EAB, FBC cho DA DC, EA EC, FB FC= = = ADC=2A, AEA=2B, CFB 2C=

Gọi M giao điểm BD EF,N giao điểm EC DF, P giao điểm FA DE Chứng minh BD CE AF

MD+NE+ PF =

Lời giải

Từ ADC=2A, AEA=2B, CFB 2C= ta suy

0 0

ADC 180 ; AEB 180 ; BFC 180  

Do ta DAC 900 1ADC 900 A

2

= − = − ,

0

BAE 90 AEB 90 B

2

= − = −

Suy 0

DAE=DAC A BAE 180+ + = − B 90

Tương tự ta EBF 90 ; FCD 90  Do

O

F N M

P E

D

C B

(128)

đa giác ADCFBE đa giác lồi

Suy ta có ADC AEB BFC+ + =2 A B C( + + )=3600

Dựng đường tròn (D; AD) (E; EB) cắt O, suy DA DC OD= =

Do ta AOC 360= 0−2ADC 460= 0−4A

Lại có AE EB EO= = nên ta

AOB 360= −4B

Suy ( )

AOC AOB 720+ = −4 A B+ =4C

Do ta ( )

BOC=360 − AOB AOC+ =360 −4C suy FB FC FO= =

Do ba đường trịn (D; AD), (E; EB) (F; FB) cắt O

Từ ta hai điểm O C đối xứng qua DF, hai điểm O A đối xứng qua DE, hai điểm O B đối xứng qua EF

Điều dẫn đến SAED =SOED; SBEF=SOEF; SCDF =SODF

Ta lại có BEF OEF

DEF DEF

S S

BD MD MB MB

1 1

MD MD MD S S

+

= = + = + = +

Hoàn toàn tương tự ta ODF ODE

EDF FDE

S S

CE AF

1 ;

NE= + S PF = + S

Do ta OEF ODF ODE

DEF

S S S

BD CE AF

3

MD NE PF S

+ +

+ + = + = Bài toán chứng minh

Bài 95. Cho ABC nhọn (AB < AC) có AH ⊥ BC H Gọi D, E hình chiếu vng góc H AB AC Đường thẳng DE cắt tia CB S Đường thẳng SA cắt đường trịn đường kính AH M Các đường thẳng BM AC cắt F Chứng minh FA.FC SB.SC SF+ =

(129)

Ta có ADH=AEH 90= nên tứ giác ADEH nội tiếpđường tròn Mặt khác C=AHE AHE ADE= nên C=ADE, ta

0

C EDB 180+ = nên tứ giác BDEC nội tiếp

Ta có

C EDB 180+ = mà

SDB EDB 180+ = nên C SDB 180= = mà lại có S chung,

đó hai tam giác SDB SCE đồng dạng với

nhau Suy SB SD

SE= SChay SE.SD SB.SC=

M

S H

N F

D

E

C B

A

Chứng minh tương tự ta SD.SE SM.SA= Suy SB.SC SM.SA= hay SB SM SA= SC Lại có BSM chung nên hai tam giác SMB SCA đồng dạng với nhau, ta

C SMB= Mà ta có

SMB AMB 180+ = nên

C AMB 180+ = suy tứ giác AMBC nội tiếp đường tròn Chứng minh tương tự ta hai tam giác FMA FBC đồng dạng với nên FA FM

FB = FC hay FA.FC FM.FB=

Trên SF lấy N cho FNM=MBS ta có

FNM MNS 180+ = nên

MBS MNS 180+ =

Lại có MBH MAC 180+ = mà MBH MBS 180+ = nên ta MBS MAC= mà FNM=MBS nên ta FNM MAC= Từ đóta tứ giác FAMN nội tiếp đường

tròn Tương tự ta FNM∽ FBS nên FN.FS FM.FB= SNM ∽ SAF nên SN.SF SM.SA= Từ ta

( )

FA.FC SB.SC FM.FB SM.SA FN.FS SN.SF FS FS SN+ = + = + = + =SF

Bài 96.Cho tam giác ABC nhọn nội tiếp đường tròn (O) Gọi A’, B’, C’ lầ lượt điểm cung nhỏ BC; CA; AB đường tròn (O) Gọi E, Q giao điểm B’C’ với AB, AC; M, F giao điểm A’C’ với BC, AB; P, N giao điểm A’B’ với AC, BC Gọi I giao điểm MQ, NE, PF Chứng minh SIMN+SIPQ+SIEF SINP+SIQE+SIMF

(130)

Gọi I tâm đường trịn nội tiếp tam giác ABC Khi ba đường thẳng AA’, BB’, CC’ đồng quy I

Thật vậy, sử dụng tính chất góc nội tiếp góc có đỉnh nằm bên đường trịn ta

có B' AI 1sdB' A' 1sdB'C 1sdA'C

2 2

= = + B'IB 1sdB' A 1sdA' B

2

= +

Mà ta lại có sdB' C sdB' A; sdA' C sdA' B= =

Từ ta B' AI=B'IA nên tam giác B’IA cân B’, suy B' I=B' A Chứng minh tương tự ta C'I C' A= Từ suy B’C’ đường trung trực đoạn thẳng AI Lặp lại cách chứng minh ta C’A’ A’B’ đường trung trực đoạn thẳng IB, IC Theo tính chất điểm nằm đường trung trực đoạn thẳng ta thu MB MI= FB FI=

O I F

E Q

P

N M

C'

C B'

B

A' A

Từ suy tam giác BMI cân M tam giác FBI cân F Đến ta suy

MBI=MIB; FBI=FIB Mà ta lại có MBI=FBI nên ta FBI=MIB MBI= =FIB, suy IM song song với BF IF song song với BM Kết hợp với MBI=FBI ta suy tứ giác BMIF hình thoi Chứng minh hồn tồn tương tự ta tứ giác AQIE

và CNIP hình thoi Từ ta IFsong song với BC IP song song với CN nên IP song song với BC, suy FP qua điểm I.Chứng minh tương tự ta QM, NE qua điểm I.Vậy đường thẳng MQ, NE, PF đồng quy điểm I

Sử dụng tính chất tỉ số diện tích hai tam giác đồng dạng ta Chứng minh hai tam giác IMN ABC đồng dạng nên

2

IMN IMN

ABC ABC

S

S MN MN

S BC S BC

 

=   =

 

Hai tam giác IPQ BCA đồng dạng với nên IPQ

BCA

S IP NC

BC BC

S = =

Hai tam giác IEF CAB đồng dạng với nên IEF

CAB

S IF BM

BC BC

(131)

Từđó ta IMN IPQ IEF

ABC BCA CAB

S

S S MN NC BM BC

1

BC BC BC BC

S + S + S = + + = =

Hay ta đươch SIMN + SIPQ + SIEF = SABC

Áp dụng bất đẳng thức Bunhiacopxki ta

( ) ( )2

IMN IPQ IEF IMN IPQ IEF ABC

3 S +S +S  S + S + S =S

Lại có SABC =(SIMN+SIPQ+SIEF) (+ SAQIE+SBMIF+SCNIP)

Mà tứ giác AQIE, BMIF, CNIP hình thoi nên ta

( ) ( )

ABC IMN IPQ IEF INP IQE IMF

S = S +S +S +2 S +S +S

Từ ta S( IMN+SIPQ+SIEF) ( SIMN+SIPQ+SIEF) (+2 SINP+SIQE+SIMF)

Hay ta SIMN+SIPQ+SIEFSINP+SIQE+SIMF

Bài 97.Cho tam giác ABC không cân A có góc ABC, ACB góc nhọn Xét điểm D di động cạnh BC cho D khơng trùng với B, C hình chiếu A BC Đường thẳng d vng góc với BC D cắt đường thẳng AB, AC E F Gọi M, N, P tâm đường tròn nội tiếp tam giác AEF, BDE, CDF Chứng minh bốn điểm A, M, N, P nằm đường tròn d qua tâm đường tròn nội tiếp tam giác ABC

Lời giải Để chứng minh toán ta tách chứng minh hai ý

sau:

+ Trước hết ta chứng minh d qua tâm đường tròn nội tiếp tam giác ABC tiếp tuyến chung khác d hai đường tròn ( )N ( )P qua điểm A

• Giả sử đường thẳng d qua điểm tâm đường trịn nội tiếp tam giác ABC, D tà tiếp điểm đường tròn nội tiếp tam giác ABC với BC nên ta

P I F

T E

N d

D C

B

(132)

được ta có BD AB BC AC; CD CA CB AB

2

+ − + −

= =

Từ suy BD DC BA BC AC CA CB AB AB AC

2

+ − + −

− = − = −

Gọi giao điểm tiếp tuyến qua A (khác AB) đường tròn ( )N với đường thẳng T Khi tứ giác TABD ngoại tiếp đường trịn nên AB TD+ =AT BD+ Kết hợp với đẳng thức ta AC TD AT CD+ = + Điều chứng tỏ tứ giác TACD ngoại tiếp đường tròn Vậy AT tiếp xúc với đường tròn ( )P hay nói cánh khác AT

tiếp tuyến chung hai đường tròn ( )N ( )P qua điểm A

• Giả sử tiếp tuyến khác d hai đường tròn ( )N ( )P qua A Gọi giao điểm tiếp tuyến với đườngthẳng d T Khi dễ thấy tứ giác TABD TACD ngoại tiếp đường trịn Do AB TD+ =AT BD+ AC TD AT DC+ = + Từ ta BD DC AB AC− = −

Lại có BD CD BC+ = nên ta BD AB BC AC; CD CA CB AB

2

+ − + −

= =

Từ suy D tiếp điểm đường tròn nội tiếp tam giác ABC với cạnh BC Từ dó suy d qua tâm đường tròn nội tiếp tam giác ABC

+ Tiếp theo ta chứng minh bốn điểm A, M, N, P nằm đường tròn tiếp tuyến chung khác d hai đường tròn ( )N

( )P qua điểm A.Ta chứng minh hai chiều sau: • Giả sử tiếp tuyến chung hai đường tròn ( )N ( )P qua A, ta cần chứng minh bốn điểm A, M, N, P nằm đường tròn

Thật vậy, để ý ba điểm E, M, N thẳng hàng ba điểm F, M, P thẳng hàng

M A

B D C

d

N E

T F

I

P

Do ta có

0

0 0 EAF 180 BAC BAC

NMP 180 EMF 180 90 90

2 2

  −

= − = − + = − =

(133)

Mặt khác tiếp tuyến chung khác d hai đường tròn ( )N ( )P qua điểm A

nên NAP 1BAC

= Mặt khác tiếp tuyến chung khác d hai đường tròn ( )N

( )P qua điểm A nên NAP 1BAC =

Do ta NMP=NAP nên tứ giác AMNP nội tiếp đường trịn

• Giả sử ta bốn điểm A, M, N, P nằm đường tròn Ta cần chứng minh tiếp tuyến chung khác d hai đường tròn ( )N ( )P qua A

Thật vậy, lập luận ta NMP BAC

= Do tứ giác AMNP nội tiếp đường tròn nên ta NMP=NAP Từ suy NAP 1BAC

2

= Qua A vẽ tiếp tuyến Ax

của đường tròn ( )N , ta NAx 1BAx =

Do ta APx NAP NAx 1BAC 1BAx 1CAx PAC

2 2

= − = − = =

Từ suy Ax đối xứng với AC qua AP, mà AC tiếp xúc với đường tròn (P) nên Ax tiếp xúc với đườn tròn ( )P Suy tiếp tuyến chung khác d hai dường tròn

( )N ( )P qua điểm A

Vậy từ hai ý ta bốn điểm A, M, N, P nằm đường tròn d qua tâm đường trònnội tiếp tam giác ABC

Bài 98.Cho tam giác ABC có trực tâm H nội tiếp đường tròn ( )O Một tiếp tuyến thay đổi đường tròn ( )O cắt HB, HC theo thứ tự E, F Điểm K đối xứng với H qua EF Chứng minh đường tròn ngoại tiếp tam giác KEF tiếp xúc với đường tròn cố định

Lời giải

(134)

của cặp đường thẳng AB DE, BC EF, CD AF Chứng minh ba điểm G, H, K thẳng hàng điểm F nằm đường tròn ( )O

Chứng minh toán phụ.

+ Điều kiện cần.Giả sử điểm F nằm trênđường tròn ( )O

Gọi I giao điểm hai đường ngoại tiếp tam giác GBD tam giác KDF (I khác D) Khi ta có

( ) ( )

( ) ( )

BIF BID DIF BGD DKF

1

sdAFE sdBCD sdABC sdCDF

2

1

sdCAE sdBCF sdBAF sdCDE

2

= + = +

= − + −

= − = −

Mà ta có BHF 1(sdBAF sdCDE)

= −

O

I H K

G F

E D

C B A

Do BIF BHF= nên tứ giác BIHF nội tiếp đường tròn, suy BIH BFH 180+ = Mặt

khác ta có BIG BDG 1sdBG

= = Mà BDG=BFE nên

BIH BIG+ =BIH BFH 180+ = ,

suy

HIG 180= Do ba điểm H, I, G thẳng hàng Chứng minh tương tự ta I, H, K thẳng hàng Như bốn điểm H, I, G, K thẳng hàng Do ba điểm G, H, K thẳng hàng

+ Điều kiện đủ. Giả sử ba điểm H, G, K thẳng hàng Gọi T giao điểm EH với

(135)

Trở lại toán. Gọi P tiếp điểm EF

với đường tròn ( )O Gọi M, N theo thứ tự giao điểm BH, CH với đường tròn ( )O

Khi dễ thấy M, N đối xứng với H theo thứ tự qua AC AB Do đường trịn tâm A bán kính AH qua điểm M N Do A H cố đinh nên đường tròn ( )A

cố định Gọi V, U theo thứ tự giao điểm

thứ hai PN, PM với đường tròn ( )A

Q

H

P N

M V

U T

K

F O

E

C B

A

Gọi T giao điểm EV FU Ta có điểm H, M, N, U, V nằm đường tròn ( )A để ý ba điểm E, P, F thẳng hàng nên theo tốn phụ T nằm đường tròn ( )A Do EF tiếp tuyến P với đường tròn ( )O nên ta có NPE=NMP=NVU suy UV song song với EF Để ý đến tứ giác TVNH

TMUH nội tiếp ta có

TVP 180= −THN 0

TUP 180= −TUM 180= −THM Do tính

đối xứng ta có MAN=2BAC 180= ( −MHN) Từ ta có biến đổi góc

( ) ( )

( ) ( )

( )

( ) ( )

0

0 0

0

0 0

0 0

ETF 180 TVU TUV 180 TVP UVP TUP VUP

180 180 THN UVP 180 THM VUP

MHN 180 UVP VUP MHN 180 UVP VUP MHN MPN

MHN 180 MAN MHN MAN 180 MHN 180 MHN 180

180 MHN 180 BHC 180 EKF

= − − = − − − −

= − − + − − +

= − + + = − − − = −

= − − = + − = + − −

= − = − = −

Suy tứ giác TEKF nội tiếp đường tròn Giả sử TQ tiếp tuyến với đường tròn ( )A ta có QTV QTE TFE TUV= = = nên TQ tiếp tuyến đường tròn ngoại tiếp tam giác EKF Vậy đường tròn ngoại tiếp tam giác EKF ln tiếp xúc với đường trịn tâm A bán kính AH cố định

(136)

tam giác PCE đồng dạngvới tam giác BAO tam giác PBF đồng dạng với tam giác CAO Tiếp tuyến P đường tròn ( )O cắt đường tròn ngoại tiếp tam

giác PCE PBF theo thứ tự M N khác P Gọi Q giao điểm EM FN

Chứng minh đường trịn ngoại tiếp tam giác QMN ln tiếp xúc đường tròn cố định P thay đổi

Lời giải

Trước hết ta phát biểu chứng minh toán phụ

Bài toán phụ 1.Cho tứ giác ABCD có cạnh AB CD cắt M, cạnh AD BC cắt N Chứng minh đường tròn ngoại tiếp tam giác ABN, CDN, ADM, BCM qua điểm

Chứng minh Giả sử hai đường tròn ngoại tiếp tam giác AD CDN cắt điểm E khác D Khi tứ giác AMED CDNE nội tiếp đường tròn nên ta có MDE CNE=

MAE MDE=

Từ ta MAE CNE= =BNE nên tứ giác ABNE nội tiếp đường tròn Mặt khác từ tứ giác nội tiếp đường trịn ta có

ECN=EDN EDN=AME

E

N M

D

C B

A

Do ta AME ECN ECM= = nên tứ giác BCEM nội tiếp đường tròn Vậy

(137)

Chứng minh. Gọi S giao điểm thứ hai đường tròn ngoại tiếp tam giác BFM đường tròn ngoại tiếp tam giác CEM Khi để ý đến tứ giác nội tiếp đường tròn tiếp tuyến EF đường trịn ngoại tiếp tam

giác BMC ta có biến đổi góc

( )

( )

0

0

0

0

ERF QRF QRE 180 ACQ 180 ABQ

360 ACQ ABQ BAC BQA

BAC 180 BMC BAC MBC MCB

180 MSE PSF 180 ESF

= + = − + −

= − + = +

= + − = + +

= − + = −

R

S

Q L

F

E M

O

C B

A

Điều dẫn đến tứ giác SERF nội tiếp đường trịn

Mặt khác ta lại có

BSC=BSM CSM+ =AEF AFE 180+ = −BAC nên tứ giác ABSC nội tiếp đường tròn Gọi SL tiếp tuyến S đường trịn ( )O Khi ta có biến đổi

góc ESL=BSL ESB+ =BCS EMB+ =BCS MCB MCS+ = =MES Từ SL tiếp tuyến S đường tròn ngoại tiếp tam giác ESF, điều kéo theo SL tiếp tuyến đường tròn ngoại tiếp tam giác REF Vậy đường tròn ngoại tiếp tam giác REF tiếp

(138)

Trở lại tốn. Do có tam giác PCE đồng dạngvới tam giác BAO tam giác PBF đồng dạngvới tam giác CAO nên ta suy

0

EPF BPF BPC CPE OAC BPC OAB BAC BPC 180

= + + = + +

= + =

Gọi K L giao điểm BN với

CM BF với CE Do có tam giác PCE đồng dạngvới tam giác BAO tam giác PBF đồng dạngvới tam giác CAO nên ta có

( ) ( )

( )

0

0

0

BLC 180 EFL FEL 180 PBF PCE

180 BAO CAO 180 2BAC

180 BOC

= − + = − +

= − + = −

= −

O

X

L N

M Q

K F

E P

C B

A

Do ta tứ giác tứ giác BOCL nội tiếp đường tròn Để ý đến tứ giác BFNP CMEP nội tiếp đường trịn ta lại có ECM EPM= =FPN FBN= nên tứ giác BCKL nội tiếp đường tròn hay điểm K thuộc đường trònngoại tiếp tam giác BOC Xét tứ giác PDQN có F giao điểm EP với QN M giao điểm NP với QE, nên

theo bài tốn phụ 1ta có đường trịn ngoại tiếp tam giác FNP, EMP QMN

đi qua mộtđiểm, ta gọiđiểmđó X Khi đótừ tứ giác BXPF, CXPE, BOPF, COPE nội tiếp đường trịn Do ta thu biến đổi góc

0

BXC=BXP CXP 180+ = −BFP 180+ −CEP=BOP COP+ =BOC nên X thuộc đường

tròn ngoạitiếp tam giác BOC Xét tam giác KBC có P điểmnằm tam giác, tiếp tuyếntại P củađường tròn ngoạitiếp tam giác PBC cắt KB, KC N, M Lại có điểm

X nằm đường trịn ngoại tiếp tam giác KBC, hai đường tròn ngoại tiếp hai tam giác XBN XCM cắt Q Khi theo bài tốn phụ 2 đường trịn ngoại tiếp tam giác QMN ln tiếp xúc với đường ngịn ngoại tiếp tam giác KBC hay

tiếp xúc vớiđường trịn ngoạitiếp tam giác BOC Mà ta có tam giác BOC cốđịnh nên

đường tròn ngoại tiếp tam giác BOC cố định Vậy đường tròn ngoại tiếp tam giác

QMN ln tiếp xúc đường trịn cố định

(139)

CN Một đường thẳng qua H cắt AC, BD theo thứ tự K, L Trên BC lấy điểm Q, R cho KR song song với BM LQ song song với CN Gọi P giao điểm KR với QL Chứng minh đường tròn ngoại tiếp tam giác PQR tiếp xúc với đường tròn ngoại tiếp tam giác EBC

Lời giải Trước hết ta phát biểu chứng minh

bài toán phụ.Cho tứ giác ABCD nội tiếp đường trịn ( )O có cạnh AB CD cắt nhau M, cạnh AD BC cắt N Khi đường tròn ngoại tiếp tam giác ABN, CDN, ADM, BCM qua một điểm E ba điểm M, E, N thẳng hàng.

O

E

N M

D

C B

A

Chứng minh Giả sử hai đường tròn ngoại tiếp tam giác MAD CDN cắt điểm E khác D Khi tứ giác AMED CDNE nội tiếp đường trịn nên ta có

MDE CNE= MAE MDE= , từ ta MAE CNE= =BNE nên tứ giác ABNE nội tiếp đường tròn Mặt khác từ tứ giác nội tiếp đường trịn ta có

ECN EDN= EDN=AME nên ta AME ECN ECM= = nên tứ giác BCEM nội tiếp đường tròn Vậy đường tròn ngoại tiếp tam giác ABN, CDN, ADM, BCM qua điểm E Do tứ giác ADEM CDEN nội tiếp đường trịn nên ta có

0

MAD MED 180+ = DCN DEN 180+ = Mà ta lại có tứ giác ABCD nội tiếp đường

trịn nên MAD DCN 180+ = Từ ta MED NED 180+ = hay ba điểm M, E, N thẳng hàng

(140)

T S

R

Y

X

Q P D

E

H

K L

N

M

C B

A

Xét tứ giác XKYL có E giao điểm XK với YK P giao điểm XY với LK, theo tốn phụ đường trịn ngoại tiếp tam giác XEL, XKP, KYE YLP qua điểm S ma điểm P, S, E thẳng hàng Từ dễ thấy

XKS=XPS=LPS=LYS KSX=KPX=YPL=YSL nên suy hai tam giác SKX

SYL đồng dạng với Mặt khác theo KR song song với BM LQ song song với CN nên theo định lý Thales ta có BL HL CX

BY=HK =CK nên ta có hai tam giác SCK

SBY đồng dạng với nhau, hai tam giác SCX SBL đồng dạng với Đến ta thu hai tam giác SCB SKY đồng dạng với BSC KSY= Để ý tứ giác SEYK nội tiếp đường trịn nên ta có BEC=YEK=YSK=BSC hay từ giác BESC nội tiếp đường tròn Cũng từ hai tam giác SCB SKY đồng dạng với ta

SBC SYL= Do CSK=BSY=CRK nên suy tứ giác SCKR nội tiếp đường tròn Lập luận chứng minh hồn tồn tương tự ta có tứ giác SXCQ nội tiếp đường trịn Đến ta có XQS XCS PRS= = nên tứ giác PQSR nội tiếp đường trịn Điều có

nghĩa hai đường tròn ngoại tiếp hai tam giác PQR BEC qua điểm S Giả sử ST tiếp tuyến đường tròn ngoại tiếp tam giác PQR Để ý đến tứ giác nội

Ngày đăng: 24/02/2021, 11:40

Từ khóa liên quan

Tài liệu cùng người dùng

Tài liệu liên quan